Sunteți pe pagina 1din 199

ACCOUNTI NG STANDARDS

Accounting Standards are the defined accounting policies issued by Government or expert institute. These
standards are issued to bring harmonization in follow up of accounting policies.

Presently, Institute of Chartered Accountants of India has issued 29 Accounting Standards as listed below.

AS 1. Disclosure of Accounting Policies
AS 2. Valuation of Inventories
AS 3. Cash Flow Statements
AS 4. Contingencies and Events Occurring After the Balance Sheet Date
AS 5. Net Profit or Loss for the Period, Prior Period Items and Changes in Accounting Policies
AS 6. Depreciation Accounting
AS 7. Construction Contracts
AS 8. Accounting for Research and Development (Not Applicable now)
AS 9. Revenue Recognition
AS 10. Accounting for Fixed Assets
AS 11. Accounting for the Effects of Changes in Foreign Exchange Rates
AS 12. Accounting for Government Grants
AS 13. Accounting for Investments
AS 14. Accounting for Amalgamation
AS 15. Accounting for Retirement Benefits in the financial Statements of Employers
AS 16. Borrowing Costs
AS 17. Segment Reporting
AS 18. Related Party Disclosure
AS 19. Leases
AS 20. Earning Per Share
AS 21. Consolidated Financial Statements
AS 22. Accounting for Taxes on Income
AS 23. Accounting for Investments in Associates in Consolidated Financial Statements
AS 24. Discontinuing Operations
AS 25. Interim Financial Reporting
AS 26. Intangible Assets
AS 27. Financial Reporting of Interests in Joint Ventures
AS 28. Impairment of Assets
AS 29. Provisions, Contingent Liabilities & Contingent Assets

Procedure for I ssuing Accounting Standards
1. Accounting Standard Board (ASB) determines the broad areas in which Accounting Standards
need to be formulated.
2. In the preparation of AS, ASB is assisted by Study Groups.
3. ASB also holds discussions with representative of Government, Public Sector Undertakings,
Industry and other organizations (ICSI/ICWAI) for ascertaining their views.
4. An exposure draft of the proposed standard is prepared and issued for comments by members of
ICAI and the public at large.
5. After taking into consideration the comments received, the draft of the proposed standard will be
finalized by ASB and submitted to the council of the Institute.
6. The council of the Institute will consider the final draft of the proposed Standard and If found
necessary, modify the same in consultation with ASB. The AS on the relevant subject will then be
issued under the authority of the council.
AS 1
DISCLOSURE OF ACOUNTING POLICY


Accounting policies are the specific accounting principles and the methods of applying those principles
adopted by an enterprise in the preparation and presentation of financial statements.

- All significant accounting policies should be disclosed.
- Such disclosure form part of financial statements.
- All disclosures should be made at one place.
- Specific disclosure for the adoption of fundamental accounting assumptions is not
required.
- Disclosure of accounting policies cannot remedy a wrong or inappropriate
treatment of the item in the accounts.

Any change in accounting policies which has a material effect in the current period or which is reasonably
expected to have material effect in later periods should be disclosed.
In the case of a change in accounting policies, which has a material effect in the current period, the amount by
which any item in the financial statements is affected by such change should also be disclosed to the extent
ascertainable. Where such amount is not ascertainable, the fact should be indicated.

Fundamental Accounting Assumption: (GCA) :

1] Going Concern

2] Consistency

3] Accrual


Major considerations governing the selection of accounting policies:

1] Prudence

2] Substance over form (Logic over Law)

3] Materiality


The following are examples of the areas in which different accounting policies may be adopted by different
enterprises:

- Methods of depreciation
- Methods of translation of foreign currency
- Valuation of inventories
- Valuation of investments
- Treatment of retirement benefits
- Treatment of contingent liabilities etc.


AS 2

VALUATI ON OF I NVENTORY

I nventories are assets:

(a) held for sale in ordinary course of business;
(b) in the process of production fro such sale (WIP);
(c) in the form of materials or supplies to be consumed in the production process or in the rendering
of services.

However, this standard does not apply to the valuation of following inventories:

(a) WIP arising under construction contract (Refer AS 7);
(b) WIP arising in the ordinary course of business of service providers;
(c) Shares, debentures and other financial instruments held as stock in trade; and
(d) Producers inventories of livestock, agricultural and forest products, and mineral oils, ores and
gases to the extent that they are measured at net realizable value in accordance with well
established practices in those industries.


Inventories should be valued at the lower of cost and net realizable value.

The cost of inventories should comprise

(a) all costs of purchase
(b) costs of conversion
(c) other costs incurred in bringing the inventories to their present location and condition.

The costs of purchase consist of

(a) the purchase price
(b) duties and taxes ( other than those subsequently recoverable by the enterprise from the taxing
authorities like CENVAT credit)
(c) freight inwards and other expenditure directly attributable to the acquisition.

Trade discounts (but not cash discounts), rebates, duty drawbacks and other similar items are
deducted in determining the costs of purchase.

The costs of conversion include direct costs and systematic allocation of fixed and variable
production overhead.

Allocation of fixed overheads is based on the normal capacity of the production facilities. Normal
capacity is the production, expected to be achieved on an average over a number of periods or
seasons under normal circumstances, taking into account the loss of capacity resulting from planned
maintenance.





Under Recovery: Unallocated overheads are recognized as an expense in the period in which they
are incurred.
Example: Normal capacity = 20000 units
Production = 18000 units
Sales = 16000 units
Closing Stock = 2000 units
Fixed Overheads = Rs. 60000

Then, Recovery rate = Rs60000/20000 = Rs 3 per unit
Fixed Overheads will be bifurcated into three parts:
Cost of sales : 16000*3 = 48000
Closing stock : 2000 *3 = 6000
Under recovery : Rs 6000 ( to be charged to P/L)

(Apparently it seems that fixed cost element in closing stock should be
60000/18000*2000 =Rs 6666.67. but this is wrong as per AS-2)

Over Recovery: In period of high production, the amount of fixed production overheads is
allocated to each unit of production is decreased so that inventories.
Example: Normal capacity = 20000 units
Production = 25000 units
Sales = 23000 units
Closing Stock = 2000 units
Fixed Overheads = Rs 60000

Recovery Rate = Rs 60000/20000 = Rs 3 per unit
But, Revised Recovery rate = Rs 60000/25000 = Rs. 2.40 per unit

Cost of sales : 23000*2.4 = Rs 55200
Closing Stock : 2000 *2.4 = Rs. 4800

Joint or by products:
In case of joint or by products, the costs incurred up to the stage of split off should be allocated on
a rational and consistent basis. The basis of allocation may be sale value at split off point or sale
value at the completion of production. In case of the by products of negligible value or wastes,
valuation may be taken at net realizable value. The cost of main product is then joint cost minus
net realizable value of by product or waste.

The other costs are also included in the cost of inventory to the extent they contribute in bringing
the inventory to its present location and condition.

Interest and other borrowing costs are usually not included in cost of inventory. However, AS-16
recommends the areas where borrowing costs are taken as cost of inventory.

Certain costs are strictly not taken as cost of inventory.
(a) Abnormal amounts of wasted materials, labour, or other production costs;
(b) Storage costs, unless those costs are necessary in the production process prior to a further
production stage;
(c) Administrative overheads that do not contribute to bringing the inventories to their present
location and condition; and
(d) Selling and Distribution costs.

Cost Formula:
Specific identification method for determining cost of inventories
Specific identification method means directly linking the cost with specific item of
inventories. This method has application in following conditions:
In case of purchase of item specifically segregated for specific project and is
not ordinarily interchangeable.
In case of goods of services produced and segregated for specific project.


Where Specific Identification method is not applicable
The cost of inventories is valued by the following methods;
FIFO ( First In First Out) Method
Weighted Average Cost

Cost of inventories in certain conditions:
The following methods may be used for convenience if the results approximate actual cost.
Standard Cost: It takes into account normal level of consumption of material and
supplies, labour, efficiency and capacity utilization. It must be regularly reviewed taking
into consideration the current condition.
Retail Method: Normally applicable for retail trade
Cost of inventory is determined by reducing the gross margin from the
sale
value of inventory.

Net Realisable Value means the estimated selling price in ordinary course of business, at the time of
valuation, less estimated cost of completion and estimated cost necessary to make the sale.

Comparison between net realizable value and cost of inventory

The comparison between cost and net realizable value should be made on item-by-item basis. (In
some cases, group of items-by-group of item basis)

For Example:
Cost NRV Inventory Value as per AS-2
Item A 100 90 90
Item B 100 115 100
Total 200 205 200 190

Raw material valuation
If the finished goods to which raw material is applied, is sold at profit, RAW MATERIAL is valued at
cost irrespective of its NRV level being lower to its costs.





AS 3
CASH FLOW STATEMENT
Definitions:
Cash comprises cash on hand and cash at bank. (Demand Deposits with bank)

Cash Equivalents are
Short Term
Highly Liquid Investments (Maturity around 3 months)
Subject to insignificant risk of changes in value.

Cash Flows are inflows and outflows of cash and cash equivalents.

Cash Flow Statement represents the cash flows during the specified period by operating, investing
and financing activities.

Operating Activities are the principal revenue-producing activities of the enterprise and other
activities that are not investing activities and financing activities.
Example:
1] Cash receipts from sales of goods/services
2] Cash receipts from royalties, fees and other revenue items
3] Cash payments for salaries, wages and rent
4] Cash payment to suppliers for goods
5] Cash payments or refunds of Income Tax unless they can be specifically identified with financing
or investing activities
6] Cash receipts and payments to future contracts, forward contracts when the contracts are held for
trading purposes.

Cash from operating activities can be disclosed either by DIRECT METHOD OR BY INDIRECT
METHOD.

I nvesting Activities are the acquisition and disposal of long-term assets and other investments not
included in cash equivalents.
Example:
1] Cash payments/receipts to acquire/sale of fixed assets including intangible assets
2] Cash payments to acquire shares or interest in joint ventures (other than the cases where
instruments are considered as cash equivalents)
3] Cash advances and loans made to third parties (Loan sanctioned by a financial enterprise is
operating activity)
4] Dividends and Interest received
5] Cash flows from acquisitions and disposal of subsidiaries

Financing Activities are activities that result in changes in the size and composition of the owners
capital (including preference share capital in the case of a company) and borrowing of the
enterprise.
Example:
1] Cash proceeds from issue of shares and debentures
2] Buy back of shares
3] Redemption of Preference shares or debentures
4] Cash repayments of amount borrowed.
5] Dividend and Interest paid


An enterprise should report separately major classes of gross cash receipts and gross cash payments
arising from investing and financing activities.
However, cash flows from following activities may be reported on a net basis.
Cash receipts and payments on behalf of customers
For example: Cash collected on behalf of, and paid over to, the owners of properties.
Cash flows from items in which turnover is quick, the amounts are large and the maturities
are short.
For example: Purchase and sale of investments
For financial enterprise: Cash receipts and payments for the acceptance and repayment of
deposits with a fixed maturity date.
For financial enterprise: Deposits placed/withdrawn from other financial enterprises
For financial enterprise: Cash advances and loans made to customers and the repayment of
those advances and loans.

Foreign Currency Cash Flows:

Cash flows arising in foreign currency should be recorded in enterprise reporting currency
applying the exchange conversion rate existing on the date of cash flow.

The effect of changes in exchange rates of cash and cash equivalents held in foreign currency should
be reported as separate part of the reconciliation of the changes in cash and cash equivalents during
the period.

Extraordinary Items: These items should be separately shown under respective heads of cash from
operating, investing and financing activities.

Investing and financing transactions that do not require the use of cash and cash equivalents should
be excluded from a cash flow statement. For Example
A] The conversion of debt to equity
B] Acquisition of an enterprise by means of issue of shares

Other Disclosure:
Components of cash and cash equivalents.
Reconciliation of closing cash and cash equivalents with items of balance sheet.
The amount of significant cash and cash equivalent balances held by the enterprise, which
are not available for use by it.












AS - 4
CONTINGENCIES AND EVENTS OCCURRING AFTER THE BALANCE SHEET DATE


Contingency : A contingency is a condition or situation, the ultimate outcome of which, gain or loss, will be
known or determined only on the occurrence, or non-occurrence, of one or more uncertain
future events.

Accounting Treatment:
If it is likely that a contingency will result in

LOSS: It is prudent to provide for that loss in the financial statements.

PROFI T: Not recognized as revenue (However, when the realization of a gain is virtually certain, then such
gain is not a contingency and accounting for the gain is appropriate.)

The estimates of the outcome and of the financial effect of contingencies are determined
- by the judgement of the management
- by review of events occurring after the balance sheet date
- by experience of the enterprise in similar transaction
- by reviewing reports from independent experts.

If estimation cannot be made, disclosure is made of the existence and nature of the contingency.

Provision for contingencies are not made in respect of general or unspecified risks.

The existence and amount of guarantees and obligations arising from discounted bills of exchange are
generally disclosed by way of note even though the possibility of loss is remote.

The amount of a contingent loss should be provided for by a charge in the statement of profit and loss if:
(a) it is probable that future events will confirm that, after taking into account any related probable
recovery, an asset has been impaired or a liability has been incurred as at the balance sheet date, and
(b) a reasonable estimate of the amount of the resulting loss can be made.


If either of aforesaid two conditions are not met, e.g where a reasonable estimate of the loss is not practicable,
the existence of the contingency should be disclosed by way of note unless the possibility of loss is
remote.Such disclosure should provide following information:

(a) the nature of the contingency;
(b) the uncertainities which may affect the future outcome;
(c) an estimate of the financial effect, or a statement that such an estimate cannot be made.






Events Occurring after the Balance Sheet Date:
Events occurring after the balance sheet date are those significant events, both favourable and unfavourable,
that occur between the balance sheet date and the date on which the financial statements are approved by the
Board of Directors in case of a company, and, by the corresponding approving authority in the case of any
other entity.

Two types of events can be identified:

Adjusting Event:
Those, which provide further evidence of conditions that, existed at the balance sheet date

Actual adjustments in financial statements are required for adjusting event.

Exceptions:
1] Although, not adjusting event, Proposed dividend are adjusted in books of account.
2] Adjustments are required for the events, which occur after balance sheet date that indicates that
fundamental accounting assumption of going concern is no longer, appropriate.

Non-Adjusting Events:

Those, which are indicative of conditions that arose subsequent to the balance sheet date.

No adjustments are required to be made for such events. But, disclosures should be made in the report of the
approving authority of those events occurring after the balance sheet date that represent material changes
and commitments affecting the financial position of the enterprise. Such disclosure should provide following
information:

(a) the nature of the events
(b) an estimate of the financial effect, or a statement that such an estimate cannot be made.
























AS-5
NET PROFIT OR LOSS FOR THE PERIOD, PRIOR PERIOD ITEMS AND CHANGES IN ACCOUNTING
POLICIES

All items of income and expense, which are recognized in a period, should be included in the determination of
net profit or loss for the period unless an Accounting Standard requires or permits otherwise.

The net profit or loss for the period comprises the following components, each of which should be disclosed on
the face of the statement of profit and loss:

(a) profit or loss from ordinary activities; and
(b) extraordinary items.

Ordinary Activities are any activities, which are undertaken by an enterprise as part of its business, and such
related activities in which the enterprise engages in furtherance of, incidental to, or arising from, these
activities.

When items of income and expenses within profit or loss from ordinary activities are of such size, nature that
their disclosure is relevant to explain the performance of the enterprise for the period, the nature and amount
of such items should be disclosed properly. Examples of such circumstances are:
(Exceptional Items)
- disposal of items of fixed assets
- litigation settlements
- legislative changes having retrospective application
- disposal of long term investments
- reversal of provisions

Extraordinary items are income or expense that arise from events or transactions that are clearly distinct
from the ordinary activities of the enterprise and, therefore, are not expected to recur frequently or regularly.
Examples of events or transactions that generally give rise to extraordinary items for most enterprises are:
- attachment of property of the enterprise;
- an earthquake

However, claims from policyholders arising from an earthquake do not qualify as an extraordinary item for
an insurance enterprise that insures against such risks.

Extraordinary items should be disclosed in the statement of profit and loss as a part of net profit or loss for
the period. The nature and the amount of each extraordinary item should be separately disclosed in the
statement of profit and loss in a manner that its impact on current profit or loss can be perceived.



Prior Period I tems:
Prior period items are income or expenses that arise in the current period as a result of ERROR or
OMMISSIONS in the preparation of the financial statements of one or more prior periods.

The nature and amount of prior period items should be separately disclosed in the statement of profit and loss
in a manner that their impact on the current profit or loss can be perceived.






Changes in Accounting Policy:
Accounting policies are the specific accounting principles and the methods of applying those principles
adopted by an enterprise in the preparation and presentation of financial statements.

A change in an accounting policy should be made only if the adoption of a different accounting policy is
required:
(a) by statute
(b) for compliance with an accounting standard
(c) if it is considered that the change would result in a more appropriate presentation of the financial
statements of the enterprise.

Any change in accounting policy which has a material effect, should be disclosed. Such changes should be
disclosed in the statement of profit and loss in a manner that their impact on profit or loss can be perceived.

Where the effect of such change is not ascertainable, the fact should be indicated.

If a change is made in the accounting policies which has no material effect on the financial statements for the
current period but which is reasonably expected to have material effect in later periods, the fact of such
change should be appropriately disclosed in the period in which the change is adopted.

The following are not changes in accounting policies:

(a) the adoption of an accounting policy for events which differ in substance from previously occurring events
e.g. introduction of a formal retirement gratuity scheme by an employer in place of ad hoc ex-gratia
payments to employees on retirement; and
(b) the adoption of a new accounting policy for events or transactions which did not occur previously or that
were immaterial.

Change in Accounting Estimate:

The nature and amount of a change in an accounting estimate which has a material effect in the current
period, or which is expected to have a material effect in subsequent periods, should be disclosed. If it is
impracticable to quantify the amount, this fact should be disclosed.

The effect of a change in an accounting estimate should be classified using the same classification in the
statement of profit and loss as was previously for the estimate.
For example, the effect of a change in an accounting estimate that was previously included as an
extraordinary item is reported as an extraordinary item.

Clarifications:
(a) Change in accounting estimate does not bring the adjustment within the definitions of an extraordinary
item or a prior period item.
(b) Sometimes, it is difficult to distinguish between a change in an accounting policy and a change in
accounting estimate. In such cases, the change is treated as a change in an accounting estimate, with
appropriate disclosures.







AS 6
DEPRECI ATI ON ACCOUNTI NG

Depreciation is a measure of the wearing out, consumption or other loss of value of a
depreciable asset arising from use, passage of time or obsolescence through technology
and market changes. Depreciation is allocated so as to charge a fair proportion of the
depreciable amount in each accounting period during the expected useful life of the asset.
Depreciation includes amortisation of assets whose useful life is predetermined.


The depreciable amount of a depreciable asset should be allocated on a systematic
basis to each accounting period during the useful life of the asset.

Depreciable assets are assets which
[1] are expected to be used during more than one accounting period; and
[2] have a limited useful life; and
[3] are held by an enterprise for use in the production or supply or for administrative
purposes

Depreciable amount of a depreciable asset is its historical cost, or other amount
substituted for historical cost less the estimated residual value.

Useful life is the period over which a depreciable asset is expected to be used by the
enterprise.
The useful life of a depreciable asset is shorter than its physical life.

There are two method of depreciation:
1] Straight Line Method (SLM)
2] Written Down Value Method (WDVM)
Note: A combination of more than one method may be used.

The depreciation method selected should be applied consistently from period to
period. The change in method of depreciation should be made only if;
The adoption of the new method is required by statute; or
For compliance with an accounting standard; or
If it is considered that change would result in a more appropriate
preparation of financial statement; or

When there is change in method of depreciation, depreciation should be
recalculated in accordance with the new method from the date of the assets coming
into use. (i.e RETROSPECTIVELY)
The deficiency or surplus arising from such recomputation should be adjusted in the
year of change through profit and loss account.
Such change should be treated as a change in accounting policy and its effect
should be quantified and disclosed.

The useful lives of major depreciable assets may be reviewed periodically. Where
there is a revision of the estimated useful life, the unamortised depreciable amount
should be charged over the revised remaining useful life. (i.e. PROSPECTIVELY)

Any addition or extension which becomes an integral part of the existing asset
should be depreciated over the remaining useful life of that asset.
The depreciation on such addition may also be applied at the rate applied to the
existing asset.
Where an addition or extension retains a separate identity and is capable of being
used after the existing asset is disposed of, depreciation should be provided
independently on the basis of estimate of its own useful life.

Where the historical cost of a depreciable asset has undergone a change due to
increase or decrease in the long term liability on account of exchange fluctuations,
price adjustments, changes in duties or similar factors, the depreciation on the revised
unamortised depreciable amount should be provided prospectively over the residual
useful life of the asset.

This accounting standard is not applied on the following items.
Forests and plantations
Wasting assets
Research and development expenditure
Goodwill
Live stock

Disclosure requirements
1] the historical cost
2] total depreciation for each class charged during the period
3] the related accumulated depreciation
4] depreciation method used ( Accounting policy)
5] depreciation rates if they are different from those prescribed by the statute
governing the enterprise




























AS 7
CONSTRUCTI ON CONTRACT

A Construction contract is a contract specifically negotiated for the construction of an asset or a
combination of assets that are closely interrelated or interdependent in terms of their design,
technology and function or their ultimate purpose or use.

Recognition of contract revenue and contract cost

When the outcome of a construction contract can be estimated reliably, contract revenue and contract
cost should be recognized as revenue and expenses by reference to the stage of construction. (This
accounting standard recommends the use of percentage of completion method)

When the outcome of a construction contract cannot be estimated reliably,
Revenue should be recognized only to the extent of contract costs incurred of which recovery is
probable. (i.e. Revenue recognized = Costs Incurred )
Contract costs should be recognized as an expense in the period in which they are incurred.

The outcome of a construction contract can be estimated reliably when all the following conditions
are satisfied:
(a) total contract revenue can be measured reliably;
(b) the receipt of revenue is probable;
(c) the contract costs to complete the contract can be measured reliably;
(d) the stage of completion at the reporting date can be measured reliably;
(e) the contract costs attributable to the contract can be clearly identified.

Contract revenueshould comprise:

(a) the initial amount of revenue agreed in the contract; and
(b) variations in amount to be received
- to the extent that it is probable that they will result in revenue; and
- they are capable of being reliably measured.

( Contract can of two kinds: Fixed Price contract and Cost Plus contract)

Contract costs should comprise:

(a) costs that relate directly to the specific contract;
(b) costs that are attributable to contract activity in general and can be allocated to the contract.


At any stage of contract, when it is probable that total contract costs will exceed total contract
revenue, the expected loss should be recognized as an expense immediately. The amount of
such loss is determined irrespective of:

- whether or not work has commenced on the contract;
- the stage of completion of contract activity; or
- whether outcome of contract is estimated or not



When an uncertainty arises about the collectability of an amount already included in contract
revenue, and already recognized in the statement of profit and loss, the uncollectable amount
or the amount in respect of which recovery has ceased to be probable is recognized as an
expense rather than an adjustment of the amount of contract revenue.

Contract costs that relate to future activity, are recognized as an asset provided it is probable
that they will be recovered. Such asset is classified as Contract WIP.

The stage of completion of a contract may be determined by following ways;
- surveys of work done
- completion of physical proportion of the contract work
- the proportion that contract costs incurred for work performed upto
the reporting date bear to the estimated total contract costs

When a contract covers a number of assets, the construction of each asset should be treated as
a separate construction of each asset should be treated as separate construction contract when
- separate proposals have been submitted for each asset;
- each asset has been subject to separate negotiation
- the costs and revenues of each asset can be identified.

A group of contracts, whether with a single customer or with several customers, should be
treated as a single construction contract when
- the group of contracts is negotiated as a single package;
- the contracts are very closely interrelated
- the contracts are performed concurrently or in a continuous sequence.

The recognition of revenue and expenses in construction contract is based on reliable estimate.
This estimate may vary from one accounting year to another accounting year. The effect of
change in estimate should be treated as per AS-5. i.e. It should not be treated as prior period
item or extraordinary item.

Disclosure:
- Contract Revenue recognized as revenue
- Method used to determine the contract revenue
- Method used to determine the stage of completion
- Contract costs incurred + Recognised Profit Recognised Loss
- Amount of advances received
- Amount due from customers
- Amount due to customers














AS 9
REVENUE RECOGNI TI ON

Revenue is the gross inflow of cash, receivables or other consideration arising in the course of the ordinary
activities of an enterprise from the sale of goods, from the rendering of services, and from the use by others of
enterprise resources yielding interest, royalties and dividends.

Revenue includes: - Proceeds from sale of goods
- Proceeds from rendering of services
- Interest, royalty and dividends.

Sale of goods

Revenue from sales should be recognized when
All significant risks and rewards of ownership have been transferred to the buyer from the seller.
Ultimate realisability of receipt is reasonably certain.

Rendering of Services

Revenue from service transactions is usually recognized as the service is performed, either by proportionate
completion method or by the completed service contract method.

1) Proportionate Completion method This is a method of accounting, which recognises revenue in the
statement of profit and loss proportionately with degree of completion of services under a contract.

Revenue is recognised by reference to the performance of each act. The revenue recognised under this
method would be determined on the basis of contract value, associated costs, number of acts or other
suitable basis.

2) Completed service contract method This is a method of accounting, which recognises revenue in the
statement of profit and loss only when the rendering of services under a contract is completed or
substantially completed.

Revenue under this method is recognised on completion or substantial completion of the job.

Revenue from I nterest : Recognised on time proportion basis

Revenue from Royalties: Recognised on accrual basis in accordance with the terms of the relevant
agreement.

Revenue from Dividends: Recognised when right to receive is established

Subsequent uncertainty in collection: When the uncertainty relating to collectability arises subsequent to the
time of sale or the rendering of services, it is more appropriate to make a separate provision to reflect the
uncertainty rather than to adjust the amount of revenue originally recorded.

Disclosure: An enterprise should disclose the circumstances in which revenue recognition has been postponed
pending the resolution of significant uncertainties.




EXAMPLES

1] On sale, buyer takes title and accepts billing but delivery is delayed at buyers request
- Revenue should be recognised notwithstanding that physical delivery has not been completed.

2] Delivery subject to installations and inspections
- Revenue should not be recognised until the customer accepts delivery and installation and inspection are
complete. However, when installation process is very simple, revenue should be recognised. For example.
Television sale subject to installation.

3] Sale on approval
- Revenue should not be recognised until the goods have been formally accepted or time for rejection has
elapsed or where no time has been fixed, a reasonable time has elapsed.

4] Sales with the condition of money back if not completely satisfied
- It may be appropriate to recognize the sale but to make suitable provision for returns based on previous
experience.

5] Consignment sales
- Revenue should not be recognised until the goods are sold to a third party.

6] Installment sales
- Revenue of sale price excluding interest should be recognised on the date of sale.

7] Special order and shipments
- Revenue from such sales should be recognized when the goods are identified and ready for delivery.

8] Where seller concurrently agrees to repurchase the same goods at a later date
- The sale should not be recognised, as this is a financial arrangement.

9] Subscriptions received for publications
- Revenue received or billed should be deferred and recognised either on a straight-line basis over time or
where the items delivered vary in value from period to period, revenue should be based on the sales value
of the item delivered.

10] Advertisement commission received
- It is recognised when the advertisement appears before public.

11] Tution fees received
- Should be recognised over the period of instruction.

12] Entrance and membership fees
- Entrance fee is generally capitalized
- If the membership fee permits only membership and all other services or products are paid for
separately, the fee should be recognised when received. If the membership fee entitles the member to
services or publications to be provided during the year, it should be recognised on a systematic and
rational basis having regard to the timing and nature of all services.

13] Sale of show tickets
- Revenue should be recognised when the event takes place.



14] Guaranteed sales of agricultural crops
- When sale is assured under forward contract or government guarantee, the crops can be recognised at
net realizable value although it does not satisfy the criteria of revenue recognition.


The above accounting standard is not applicable for:
- Revenue arising from construction contracts
- Revenue arising from hire purchase, lease agreements
- Revenue arising from Government grants and subsidies
- Revenue of Insurance companies arising from insurance contracts
- Profit or loss on sale of fixed assets
- Realised or unrealized gains resulting from changes in foreign exchange rates























AS-10
ACCOUNTI NG FOR FIXED ASSETS


Definitions:
Fixed Asset is an asset held with the intention of being used for the purpose of producing or providing goods
or services and is not held for sale in the normal course of business. (It is expected to be used for more than
one accounting period.)

The cost of fixed asset includes:
Purchase price
Import Duties and other non-refundable taxes
Direct cost incurred to bring the asset to its working condition
Installation cost
Professional fees like fees of architects
General overhead of enterprise when these expenses are specifically attributable to
acquisition/preparation of fixed assets
Any expenses before the commercial production, including cost of test run and experimental
production
Any expenses before the asset is ready for use not put to use
Loss on deferred payment arising out of foreign currency liability
Price adjustment, changes in duties and similar factors

The cost of fixed asset is deducted with:
Trade discounts and rebates
Sale proceeds of test run production
Amount of government grants received/receivable against fixed assets (See AS- 12)
Gain on deferred payment arising out of foreign currency liability

Similarly, historical cost of self constructed fixed assets will include:
All cost which are directly related to the specific asset
All costs that are attributable to the construction activity should be allocated to fixed assets
Any internal profit included in the cost should be eliminated.

Any expenses incurred on asset between date of ready for use and put to use is either charged to P&L
A/c or treated as deferred revenue expenditure to be amortised in 3-5 years after commencement of
production.

When fixed asset is acquired in exchange for another asset, the cost of the asset acquired should be
recorded
- either at, fair market value
- or at, the net book value of the assets given up

For this purpose, fair market value may be determined by reference either to the asset given up or to
the asset acquired, whichever is more clearly evident.

Fixed asset acquired in exchange for shares or other securities should be recorded at FMV of assets
given up or asset acquired, whichever is more clearly evident. (i.e the option of recording the asset at
net book value of asset given up is closed)

Fair market value is the price that would be agreed to in an open and unrestricted market between
knowledgeable and willing parties dealing at arms length distance.
Subsequent expenditures related to an item of fixed asset should be added to its book value only if
they increase the future benefits from the existing asset beyond its previously assessed standard of
performance.
Material items retired from active use and held for disposal should be stated at the lower of their
net book value and net realizable value and shown separately. Fixed assets should be eliminated
from the financial statements on disposal or when no further benefit is expected from its use and
disposal. Profit/loss on such disposal or writing off is recognized in the profit and loss account.
REVALUATI ON
When the fixed assets are revalued, these assets are shown at revalued price. Revaluation of fixed
assets should be restricted to the net recoverable amount of fixed asset.
When a fixed asset is revalued, an entire class of assets should be revalued or selection of assets
for revaluation should be made on a systematic basis. That basis must be disclosed.
Accounting treatment of revaluation under different situation:

When revaluation is made upward

Fixed Assets A/c Dr
To Revaluation Reserve


When revaluation is made downward

P&L A/c Dr
To Fixed Assets

When revaluation is made upward subsequent to previous upward revaluation

Fixed Assets A/c Dr
To Revaluation Reserve


When revaluation is made downward subsequent to previous upward revaluation

Revaluation Reserve A/c Dr (To the extent of carrying amount of R.R)
P&L A/c Dr (Balancing Figure)
To Fixed assets


When revaluation is made upward subsequent to previous downward revaluation

Fixed assets A/c Dr
To P&L A/c (To the extent of previous downward
revaluation)
To Revaluation Reserve (Balancing Figure)


When revaluation is made downward subsequent to previous downward revaluation

P& L A/c Dr
To Fixed Assets



Accounting treatment on disposal of Fixed Assets:
On sale of fixed assets
Bank A/c Dr
P & L A/c Dr (If Loss)
To Fixed Assets
To P & L A/c (If Profit)

On sale of fixed assets where upward revaluation has taken place

On disposal of a previously revalued item of fixed asset, the difference between net disposal
proceeds and the net book value is normally charged or credited to the profit and loss account
except that, to the extent such a loss is related to an increase which was previously recorded as a
credit to revaluation reserve and which has not been subsequently reversed or utilized, it is
charged directly to that account. The amount standing in revaluation reserve following the
retirement or disposal of an asset which relates to that asset may be transferred to general reserve.

If Loss If Profit

Bank A/c Dr Bank A/c Dr
Revaluation Reserve A/c Dr To Fixed Assets A/c
P& L A/c Dr To P/L A/c
To Fixed Assets

Revaluation Reserve A/c Dr Revaluation Reserve A/c Dr
To General Reserve To General Reserve


In the case of fixed assets owned by the enterprise jointly with others, the extent of the enterprises
share in such assets, and the proportion of the original cost, accumulated depreciation and WDV
should be stated in the B/S.
Alternatively, the pro rata cost of such jointly owned assets may be grouped together with similar fully
owned assets with an appropriate disclosure thereof.


Only purchased goodwill should be recorded in books.

Disclosure:
Gross and net book value of fixed assets at the beginning and end of period showing additions and
disposals
Revalued amounts substituted for historical costs of fixed assets, the method adopted to compute
the same and whether an external valuer was involved.


AS - 12
ACCOUNTI NG FOR GOVERNMENT GRANTS

Applicability: Mandatory for all enterprises with respect from 01/04/1994.

Government Grants are assistance by government in cash or kind for past or future compliance with certain
conditions.

Government grants may be received in following ways.
Grants related to acquisition of fixed assets
Grants related to revenue
Grants related to promoters contribution
Grants related to compensation for expenses

Government Grants should be recognised
Where there is reasonable assurance that the enterprise will comply with the conditions attached to
them; and
The grants will be received.

Amount of Grant:
Monetary Grant: Amount earned should be the value of grant.

Non- Monetary Grant:
Where grants are given at concessional rate, then such assets are accounted for at their acquisition
cost.
Where grants are given free of cost, then such assets are recorded at nominal value.

Accounting Treatment:

Grants related to Depreciable assets:
EITHER, Grants are shown as deduction from Gross value of assets

Bank A/c Dr
To Government Grant

Government Grant Dr
To Fixed Assets
(When grant is equal to book value of asset, fixed asset is shown at nominal value.)

OR, Grants are treated as deferred income

Bank A/c Dr
To Grant
[In this case, Grants are recognised as profit in P&L A/c on a systematic and rational basis over the useful
life of assets (i.e. in proportion to the amount of depreciation charged over period)]

{Net effect on Profit & Loss A/c will remain same in both cases}


Grants related to Non-Depreciable assets:
EITHER, Grants are shown as deduction from Gross value of assets

Bank A/c Dr
To Government Grant

Government Grant Dr
To Fixed Assets
(When grant is equal to book value of asset, fixed asset is shown at nominal value.)

OR, shown as reserves
When no future obligations are to be fulfilled
Bank A/c Dr
To Gov. Grant

Gov. Grant Dr
To Capital Reserve

When grant requires fulfillment of certain obligations:
Bank A/c Dr
To Gov. Grant

Gov. Grant Dr (Should be credited to income over the same period over which
the cost of
To P&L A/c meeting such expense is charged to revenue)
(In respective years)
The deferred income balance should be separately disclosed in the financial statement.

Grants related to revenue
Government grants related to revenue should be recognised on a systematic basis in the profit and loss
account over the periods necessary to match with the related costs, which they are intended to compensate.

Grants related to promoters contribution
Grant should be treated as Capital Reserve.

Bank A/c Dr
To Gov Grant

Gov Grant A/c Dr
To Capital Reserve

Grants related to compensation for expenses
Government grants receivable as compensation for expenses or losses (with no further costs) should be
recognised as an income in the year of receivable as an Extra-ordinary item.

REFUND OF GOVERNMENT GRANT
Government grants sometimes become refundable because certain conditions are not fulfilled. The grant
refundable is treated as an extraordinary item.

The amount refundable in respect of a government grant related to a specific asset is recorded by increasing
the book value of the asset or by reducing the capital reserve or the deferred income balance, as appropriate,
by the amount refundable. (Where the book value of asset is increased, the depreciation should be provided on
new asset value prospectively)

Where the amount refundable is in respect of a government grant related to revenue, the refund is applied first
against any unamortised deferred credit remaining in respect of the grant. Rest amount of refund should be
charged to profit and loss account.

Where, the amount refundable is in respect of promoters contribution, the capital reserve should be reduced
by the amount refundable.

Contingency related to Govt. Grant
A contingency related to Govt. grant receivable and refundable should be treated in accordance with AS-4.

Disclosures:
The accounting policy adopted
The nature and extent of gov. grants recognised in the financial statements.

































AS - 13
ACCOUNTI NG FOR I NVESTMENTS (Revised in 2003)
Applicability: Mandatory for all enterprises.

Investments are classified as Long Term Investments and Short Term Investments.

Current Investment is intended to be held for not more than one year and readily realisable.

Long term Investment is an investment other than a current investment.

The carrying amount of current investments is lower of cost and fair value.

It is prudent to carry investments individually at the lower of cost and fair value. But, such
comparison can also be made category-wise.

The carrying amount of long-term investments is carried at cost. However, when there is
permanent decline in the value of a long-term investment, the carrying amount is reduced to
recognize the decline. The carrying amount of long-term investments should be determined on
individual basis.

Any reduction or reversal of reduction in value of investment is adjusted through P&L A/c.

Cost of I nvestments:

The cost of an investment should include acquisition charges such as brokerage, fees and duties.

If an investment is acquired-
- by issue of shares or other securities; then the investments should be valued at the
fair value of the issued security. (i.e. Issue price determined by statutory authority)
- By exchange of another asset; then the investments should be valued at fair value of
the asset given up or asset acquired, whichever is more clearly evident.


Investment property is investment in land or buildings that is not intended to be occupied
substantially for use by, or in the operations of, the investing enterprise. An investment
property is classified as long-term investment.

Disposal of Investments : On disposal, the difference between the carrying amount and
the disposal proceeds, net of expenses, is recognized in the profit and loss statement.

Reclassification of investments:
Long-term to short-term: Transfers from one class to another class are made at lower of
cost and carrying amount at the date of transfer.
Current to long-term: Transfers are made at lower of cost and fair value at the date of
transfer.


Disclosure:

1] Accounting policies for determination of carrying amount
2] Classification of Investments
3] The amounts included in Profit and loss statement
- profits or losses on disposal and changes in carrying amount of current and
longterm investments
- interest, dividends (showing separately dividends from subsidiary) and rentals on
investments showing separately such income from current and long term
investments.
- Gross Income should be disclosed (i.e. The amount of TDS should be shown under
advance taxes paid)
4] Aggregate amount of quoted and unquoted investments giving the aggregate market
value of quoted investments.


































AS- 15
ACCOUNTI NG FOR RETI REMENT BENEFITS
(Revised in 2005 & titled as Employees Benefit)

Applicability: It is mandatory for all enterprises.
Retirement Benefits consists of :
1. Provident Fund
2. Superannuation / Pension
3. Gratuity
4. Leave Encashment Benefit
5. Other Retirement Benefits

Accounting treatment under Defined Contribution Scheme/ Provident Fund

Contribution payable by the employer in a year is charged to profit & loss account.


Accounting treatment under Defined Benefit Scheme/ Gratuity/ Leave Encashment

Payment of Retirement Benefit out of its own fund

Appropriate provision for accruing liability is created through profit & loss account. Accruing
liability is calculated by actuarial method.

Note: Actuarial valuation is the process used by an actuary (expert) to estimate the present value
of benefits to be paid under a retirement benefit scheme. Actuarial valuation should normally be
conducted at least once in every three years. Differences arising after fresh actuary valuation
should be adjusted through Profit & Loss account in the year in which fresh actuary valuation is
conducted.

Benefits funded through creation of a trust

Amount to be contributed to the trust every year is provided through profit & loss account. The
amount to be contributed is calculated by actuarial valuation.

Benefits funded through a scheme administrated by the insurer

The premium paid to the insurer is charged to profit & loss account. Such premium is calculated
through actuarial valuation.

Review of Actuarial Method/Assumption

Any alterations in the retirement benefit costs, arising due to change in method/ assumption, are

EITHER, charged to credited to profit and loss account in the year of change in accordance with
Accounting Standard 5. Prior Period and extra ordinary item and changes in accounting
policies.
OR, spread over a period not more than the expected remaining working lives of the participating
Employees.
Disclosure:

Method by which retirement benefit costs for the period have been defined
When accounting is made as per actuarial valuation, date on which such valuation was conducted.












































AS-17

SEGMENTAL REPORTI NG

A BUSI NESS SEGMENT is a distinguishable component of an enterprise that is engaged in providing an
individual product or service or a group of related products or services and that is subject to risks and
returns that are different from those of other business segments.

A GEOGRAPHI CAL SEGMENT is a distinguishable component of an enterprise that is engaged in
providing products or services within a particular economic environment and that is subject to risks and
returns that are different from those of components operating in other economic environments.

The risks and returns of an enterprise are both by the geographical
(1) location of production or service facilities and other assets of an enterprise and
(2) location of its customers.
The definition allows geographical segments to be based on any of the two.

A REPORTABLE SEGMENT is a business segment or a geographical segment identified on the basis of
foregoing definitions for which segment information is required to be disclosed by the standard.

ENTERPRI SE REVENUE is revenue is revenue from sales to external customers as reported in the
statement of profit and loss. (i.e. Sales made to external customers by all segments)

SEGMENT REVENUE is the aggregate of
(f) revenue directly attributable to segments
(g) revenue reasonably allocated to segment; and
(h) revenue from transactions with other segments.

SEGMENT EXPENSE is the aggregate of
(a) operating expense directly attributable to segment
(b) expenses reasonably allocated to segment; and
(c) expenses relating to transactions with other segments.

However, SEGMENT REVENUE/EXPENSE does not include
(a) Extraordinary items as defiened in AS-5
(b) Interest or dividend ( including earned/incurred on loans to other segment) unless the operations of
the segment are primarily of a financial nature
(c) Gains on sales of investments or on extinguishments of debt (Capital gain/loss) unless the operations
of the segment are primarily of a financial nature.
(d) General administration expenses, head office expenses and other expenses that arise at the enterprise
level and relate to the enterprise as a whole.

SEGMENT RESULT is segment revenue less segment expenses.

SEGMENT ASSETS are those operating assets that are employed by a segment in its operating activities
and that either are directly attributable the segment or can be allocated to the segment on a reasonable
basis.

SEGMENT LI ABI LI TIES are those operating liabilities that result from operating activities and that
either are directly attributable the segment or can be allocated to the segment on a reasonable basis.

( If the segment result of a segment includes interest expense, its segment liabilities include the related
interest-bearing liabilities and vice versa.)
(Segment liabilities do not include income tax liabilities and vice versa.)
Similarly, if depreciation segment expenses then related assets comes under segment assets.
Primary segment and Secondary segment

One among the two, Business Segment and Geographical Segment, is primary segment and other becomes
secondary segment. The reporting requirements for the primary and secondary segments are different.

Basis for identifying primary and secondary segments

Risks and returns are the main criteria for identifying primary and secondary segments.
If the risks and returns of an enterprise are affected predominantly by differences in the products,
business segments are recognized as primary segments and geographical segments as secondary
segments and vice versa.
If the risks and returns of an enterprise are affected both by differences in the products as well as
differences in the locations in which it operates, then the enterprise should use business segments
as its primary segment and geographical segment as its secondary segment.
If risks and returns of an enterprise are affected neither by differences in products/services nor by
differences in geographical areas of operations, the management may elect any of the two as
primary with other being secondary segment.

(Internal organization and management structure of an enterprise and its system of internal financial
reporting to the board of directors and the CEO should normally be the basis for identifying the
predominant source and nature of risks and differing rates of return facing the enterprise.)

Reportable Segments
A business segment or geographical segment should be identified as reportable segment if:

(a) its revenue from sales to external customers and from transactions with other segments is 10% or
more of the total revenue, external and internal, of all segments; or

(b) its segment result, whether profit or loss, is 10% or more of-
(1) the combined result of all segments in profit, or
(2) the combined result of all segments in loss,
whichever is greater in absolute amount; or

(c ) its segment assets are 10% or more of the total assets of all segments.


A business/reportable segment that is not a reportable segment as per above, may be recognized as
reportable segment despite its size at the discretion of the management of the enterprise.

If total external revenue attributable to reportable segments constitutes less than 75% of the total
enterprise revenue, additional segments should be identified as reportable segments, even if they do not
meet 10% thresholds as above, until at least 75% percent of the total enterprise revenue is included in
reportable segments.

A segment identified as a reportable segment in the immediately preceding period because it satisfied the
relevant 10% thresholds should continue to be a reportable segment for the current period
notwithstanding that its revenue, result, and assets no longer meet the 10% thresholds.





Primary Reporting Format
The following disclosures are required for each reportable segment of primary segment:
(1) Segment revenue, (with classification of external and internal)
(2) Segment result
(3) Total carrying amount of segment assets
(4) Total amount of segment liabilities
(5) Total cost incurred during the period to acquire segment assets
(6) Depreciation and amortisation recognized as expense, and
(7) Total non cash expenses other than Dep. And amortisation recognized as expense.

Disclosers required pursuant to clause (6) and (7) above, need not be made in respect of a segment, if the
enterprise reports cash flows arising from operating, investing and financing activities for such segment.

An enterprise should present a reconciliation between the information disclosed for reportable segments
and aggregated information in the enterprise financial statements (in respect of clause 1 to 4 above)

Secondary Reporting Format
Where primary segments are business segments
(1) Segment revenue from external customers for each geographical segment whose revenue from
sales to external customers is 10% or more of enterprises revenue;
(2) The total carrying amount of segment assets for each geographical segment whose segment
assets is 10% or more of the total assets of all geographical segments;
(3) New assets acquired for each geographical segment whose segment assets is 10% or more of
the total assets of all geographical segments.
Where primary segments are geographical segments based on location of assets
The following information (point 1 to 3)should be disclosed for each business segment whose
revenue from sales to external customers is 10% or more of enterprise revenue or whose segment
assets are 10% or more of the total assets of all business segments
(1) segment revenue from external customers;
(2) the total carrying amount of segment assets; and
(3) new segment assets acquired;
(4) if location of customers is different from location of its assets, then the enterprise should
also report revenue from sales to external customers for each customers based
geographical segment whose revenue from sales to external customers is 10% or more of
enterprise revenue.
Where primary segments are geographical segments based on location to customers
Points 1 to 3 as above.
(5) If locations of assets are different from locations of customers, then the enterprise is
required to report the following segment information for each asset based geographical
segment whose revenue from sales to external customers is 1% or more of enterprise
revenue or whose segment assets are 10% or more of total enterprise assets
I. The total carrying amount of segment assets by geographical location of the assets;
and
II. New segment assets acquired by location of assets.

Other Disclosures:
In measuring and reporting segment revenue from transactions with other segments, inter-segment
transfers should be measured on the basis that the enterprise actually used to price those transfers.
The transfer-pricing basis should be disclosed in the financial statements.

An enterprise should indicate the types of products and services included in each reported business
segment and indicate the composition of each reported geographical segment, primary and secondary,
if not otherwise disclosed in the financial statements.































AS-16
BORROWI NG COSTS
Applicability: Mandatory for all enterprises w.e.f. 01/04/2000.

Borrowing Costs include:
1. Interest and commitment charges on borrowings
2. Amortization of discounts or premiums relating to borrowings
3. Amortisation of ancillary costs incurred in connection with the arrangement of
borrowings
4. Exchange difference arising from borrowings to the extent it amounts to interest costs.

Borrowing costs should be recognized as an expense in the period in which they are incurred.

Borrowing costs that are directly attributable to the acquisition, construction or production of a
qualifying asset should be capitalized as part of that asset.

Qualifying Asset is an asset that necessarily takes a substantial period of time to get ready for its
intended use or sale. e.g. Heavy Plant & Machinery.

BORROWING COST ELIGIBLE FOR CAPITALISATION

Specific Borrowing for acquisition of qualifying asset: Borrowing cost to be capitalised

Amount of borrowing cost = Specific Borrowing Cost Income from temporary investment

General Borrowing and used for acquisition of qualifying asset: Borrowing cost should be capitalised
with the following amount;

Amount of Borrowing Cost = Expenditure cost on asset or Asset cost * Capitalisation rate

Capitalisation Rate = Weighted Average Borrowing costs on general borrowing
(i.e. Excluding cost of specific borrowing)

Note: When with the capitalization of borrowing cost, the cost exceeds the net recoverable amount, the
carrying amount is written down to net recoverable amount as per the recommendation of other
accounting standards.

COMMENCEMENT OF CAPITALISATION

Capitalisation of Borrowing should commence when all the following conditions are satisfied

1. Expenditure for the acquisition of a qualifying asset is being incurred
2. Borrowing costs are being incurred; and
3. Activities that are necessary to prepare the asset for its intended use or sale are in progress

SUSPENSION OF CAPITALISATION

Capitalisation of borrowing costs should be suspended during extended periods in which active
development is interrupted.

However capitalization should not be suspended when a temporary delay is a necessary part of the
process of getting an asset ready for its intended use or sale.

Example:
Borrowing costs incurred while land is under development are capitalized during the period in
which activities related to the development are being undertaken. However, it should not be
capitalized when land acquired for building purposes is held without use.

CESSATION OF CAPITALISATION

Capitalisation of borrowing costs should cease when substantially all the activities necessary to
prepare the qualifying asset for its intended use or sale are complete.


When the construction of a qualifying asset is completed in parts
and a completed part is capable of being used while construction continues for the other parts,
capitalization of borrowing costs in relation to that part should cease
when substantially all the activities necessary to prepare that part for its intended use or sale are
complete.
Example:
Housing complex comprising several buildings: If individual building can be used separately, its
capitalisation should cease.
Disclosure:
1. The Accounting Policies adopted for borrowing costs.
2. The amount of borrowing costs capitalized during the period.






AS-18




RELATED PARTY DI SCLOSURES











(revised in 2003)





Applicability: Mandatory for all enterprises with respect from 01/04/2004

Related party is considered to be related if at any time during the reporting period

one party has the ability to control the other party or exercise significant influence over the
other party in making financial and/or operating decisions. The following related party
relationship are covered under AS-18:

1. Enterprises that directly, or indirectly through one or more intermediaries, control, or are
controlled by, or are under common control with the reporting enterprise ( e.g. Holding
companies, subsidiaries
and fellow subsidiaries)
2. Associates and joint ventures of the reporting enterprise
3. Individuals owning, directly or indirectly, an interest in the voting power of the reporting
enterprise that gives them control or significant influence over the enterprise, and relatives of any
such individual;
4. Key management personnel and relatives of such personnel; and
5. Enterprises over which any person described in (4) or (5) is able to exercise significant influence.

Key Management Personnel
Those persons who have the authority and responsibility for planning, directing and controlling the
activities of the reporting enterprise.

The following are not deemed to be related party:

1. Two companies simply because they have common director
2. A single customer/supplier with whom an enterprise transacts a significant volume of business
merely by virtue of the resulting economic dependence
3. Providers of finance
4. Trade Unions
5. Public Utilities
6. Government departments and Government sponsored bodies

DI SCLOSURE
If there have been transactions between related parties, during the existence of a related party
relationships, the reporting enterprise should disclose the followings:
1. Name of the transacting related party
2. Description of the relationship
3. Description of nature of transaction
4. Volume of transaction ( Amount wise or proportion wise)
5. Any other information necessary for understanding financial statements
6. Outstanding ( Amount wise or proportion wise) and any provision for doubtful debt due
from such party
7. Amounts written off or written back in respect of debts due from or to related parties.

The followings are the examples of the related party transactions
Purchase or sale of goods
Rendering or receiving services
Purchases or sales of fixed assets
Licence agreements
Leasing or hire purchase agreements
Guarantees and collaterals
Management contracts including for deputation of employees.

















AS-19
LEASES

Applicability:- Mandatory for all enterprises w.e.f. 1.04.2001.

It should be applied in accounting for all leases other than:

a) lease agreements to explore for or use natural resources;
b) licensing agreements for items such as plays, manuscripts, patents and copyrights; and
c) lease agreements to use lands.
Lease : A lease is an agreement whereby the lessor conveys to the lessee in return for a payment or
series of payments the right to use an asset for an agreed period of time.
Finance Lease : All risks and rewards incident to ownership of an asset is transferred.
Operating Lease : Lease other than finance lease; i.e. which does not transfer all the risk and
reward incidental to ownership.

Minimum Lease Payments :
For lessor Total Lease rent to be paid over the lease term
+
Any Guaranteed Residual Value by or on behalf of Lessee
+
Residual Value Guaranteed by Third Party
(-)
Contingent Rent
(-)
Cost for Service and tax to be paid by and reimbursed to lessor

For lessee - Total Lease rent to be paid over the lease term
+
Any Guaranteed Residual Value by or on behalf of Lessee
(-)
Contingent Rent
(-)
Cost for Service and tax to be paid by and reimbursed to lessor

Accounting for Finance Lease I n the books of lessee
The lessee should recognize the lease as an asset at lower of the following
- Fair Value of the leased asset
- Present value of minimum lease payments
(In calculating the present value of the minimum lease payments, the discount rate is the interest rate
implicit in the lease. If implicit rate is not known, the lessees incremental borrowing rate should be
used.) Entry required to be passed:
Lease Assets A/c Dr
To Lessor

All lease payments should then be apportioned between the finance charge and the reduction of the
outstanding liability. Finance charge should be debited to P&L A/c.
Lessor A/c Dr
P&L A/c Dr (With the amount of finance charge)
To Bank A/c (With the amount of lease payment)

The lessee as per AS-6 should depreciate the leased asset.

Accounting for Finance Lease I n the books of lessor

The lessor should recognize the transaction as sale with the cash price. If artificially low rates of
interest are quoted, profit on sale should be restricted to that which would apply if a commercial rate
of interest were charged.
The cost of sale recognized at the commencement of the lease term is the cost/carrying amount less the
present value of the unguaranteed residual value.

Accounting for Operating Lease I n the books of lessee

Lease payments (excluding costs for services such as insurance and maintenance) are recognized as
an expense in the statement of profit or loss on a straight-line basis unless another systematic basis is
more appropriate.

Accounting for Operating Lease I n the books of lessor

Lease receipts are recognized as an income in the statement of profit or loss on a straight-line basis
unless another systematic basis is more appropriate. The lessor should present an asset given under
operating lease in its balance sheet under fixed assets.

Initial direct costs incurred specifically to earn revenues from an operating lease are
Either, deferred and allocated to income over the lease term in proportion of income
Or, recognized as an expense in the statement of current year profit and loss.


SALE AND LEASEBACK TRANSACTI ONS
A sale and leaseback transaction involves the sale of an asset by the vendor and the leasing of the
same asset back to the vendor.

If sale and leaseback transaction results in finance lease:

Excess or deficiency of sale proceeds over the carrying amount should be deferred and amortised over
the lease term in proportion to the depreciation of the leased asset. It should not be immediately
recognized as income or loss in the financial statements.

If sale and leaseback transaction results in operating lease:

If the sale price is equal to fair value
- Any profit or loss should be recognized immediately.

If the sale price is below fair value
- Any profit or loss should be recognized immediately, except that, if the loss is
compensated by future lease payments at below market price
- If the loss is compensated by future lease payments at below market price, the profit
or loss should be deferred and amortised in proportion to the lease payments.

If the sale price is above fair value
- The excess over fair value should be deferred and amortised over the period for
which the asset is expected to be used.

Further, if the fair value at the time of a sale and leaseback transaction is less than the carrying
amount of the asset, a loss equal to the amount of the difference between the carrying amount and fair
value should be recognized immediately.

Disclosure Requirements:

In the books of lessee in case of financial lease
1. Assets acquired under finance lease
2. Reconciliation between the total of minimum lease payments and their present value as at the
balance sheet date with following segregation
- not later than one year
- later than one year and not later than five years
- later than five years
3. Contingent rents recognized as expense
4. Future minimum sublease payments expected to be received under non-cancellable subleases
5. General description of the leasing arrangements

In the books of lessor in case of financial lease
1. General description of the significant leasing arrangement
2. Accounting policy for initial direct cost
3. Reconciliation of total gross investment in lease and present value of minimum lease payment
(MLP) receivable at the balance sheet date
4. MLP receivable in following categories
- not later than one year
- later than one year and not later than five years
- later than five years

In the books of lessee in case of operating lease
1. General description of the significant leasing arrangement
2. Total of future minimum lease payments in the following period
- not later than one year
- later than one year and not later than five years
- later than five years
3. Lease payments recognized in profit & loss account for the period


In the books of lessor in case of operating lease
1. General description of the significant leasing arrangement
2. Accounting policy for the initial direct payment
3. Future lease payments in aggregate classified as :
- not later than one year
- later than one year and not later than five years
- later than five years











AS-20
EARNI NGS PER SHARE
(Revised in 2004)

Applicability:- Mandatory w.e.f. 1.04.2001 in respect of enterprises whose equity shares or potential
equity shares are listed on a recognized stock exchange in India.

An enterprise should present BASIC & DILUTED EPS on the face of the statement of profit and loss
account for each class of equity shares that has a different right to share in the net profit for the period.
EPS to be calculated & presented even in case of losses.


Basic EPS = Net profit/loss for the period attributable to equity shareholders
Weighted Average No. of Equity Shares


Diluted EPS =
Adjusted Net profit/loss for the period attributable to equity shareholders
Weighted Average No. of (Equity Shares + Dilutive Potential
Equity Shares)

Where net profit/loss for equity shareholders = PAT less Preference Dividend including CDT
(Preference Dividend should be deducted whether or not provided in case of Cumulative Preference
Shares).

Date from which the shares are included for calculation of weighted no. of shares:

Equity shares issued for cash

Date on which cash is received
Debentures converted to cash

Date of conversion
Equity shares issued in exchange for settlement
Of a liability
Date when settlement becomes
effective
Equity shares issued for rendering of services

Date on which services are
rendered
Equity Shares issued in course of
Amalgamation in the nature of Purchase

Date of the Acquisition
Equity Shares issued in course of
Amalgamation in the nature of Purchase

Beginning of the reporting
period

Partly paid equity shares are treated as a fraction of an equity share to the extent that they were entitled
to participate in dividends relative to a fully paid equity shares.
Where an enterprise has equity shares of different nominal values but with the same dividend rights, the
number of equity shares is calculated by converting all such equity shares into equivalent number of
shares of the same nominal value.


BONUS ISSUE, SHARE SPLIT, REVERSE SHARE SPLIT etc.
In these cases, shares are issued to existing shareholders for no additional consideration. Therefore, the
number of equity shares outstanding is increased without an increase in resources.

RIGHTS ISSUE
In rights issue, the exercise price is often less than the fair value of the shares. Therefore, a right issue
generally includes a bonus element. Hence, number of equity shares to be taken for calculating Basic
EPS should be:
Right Shares + (Equity Shares prior to right * conversion factor)
Where, conversion factor = Fair value per share immediately prior to the exercise of
rights
Theoretical ex-rights fair value per share
Where, Theoretical ex-rights fair value = (Fair value of Prior shares + Right Proceeds)
Post Right total no. of equity shares


DI LUTED EARNI NGS PER SHARE

Diluted EPS =
Adjusted Net profit/loss for the period attributable to equity shareholders
Weighted Average No. of (Equity Shares + Dilutive Potential Equity
Shares)
In calculating diluted EPS, the net profit (considered for BASIC EPS) is adjusted with the corresponding
changes in profits that shall arise when dilutive potential shares are issued. For example: When
debentures are converted to shares, the net profit should be added with interest amount and further
adjusted with related tax expense.

Potential equity shares should be treated as dilutive when, and only when, their conversion to equity
shares would decrease net profit per share from continuing ordinary operations.

DI SCLOSURE

Basic & Diluted EPS
Amount used as numerator & reconciliation with PAT
Number used as denominator for Basic & Diluted EPS & reconciliation thereon
Nominal value of shares along with EPS figures







AS 22
ACCOUNTI NG FOR TAXES ON I NCOME

Applicability :- a) Mandatory w.e.f. 1-04-2001 in respect of the following:
1.Enterprises whose equity or debt securities are listed on a recognized
stock exchange in India;
2.All the enterprises of a group, if the parent presents consolidated
financial statements.
b) Mandatory w.e.f. 1.04.2002, in respect of companies not covered by a);
c) Mandatory w.e.f. 1.04.2006 in respect of all other enterprises.

Accounting income (loss) is the net profit or loss for a period, as reported in the statement of profit and
loss, before deducting income tax expense or adding income tax saving. (i.e. PBT as per P/L A/c)
Taxable income (tax loss) is the amount of income (loss) for a period, determined in accordance with the
tax laws, based upon which income tax payable (recoverable) is determined. (i.e. GTI)

Tax expense (tax saving) is the aggregate of current tax and deferred tax charged or credited to the
statement of profit and loss for the period. (i.e. tax which is to be debited or credited to P/L A/c)

Current tax is the amount of income tax determined to be payable (recoverable) in respect of the taxable
income (tax loss) for a period. (i.e. tax as per Income tax Act)

Deferred tax is the tax effect of timing differences. Model journal entries to be passed in books of
account should be as under:

Current Tax A/c ..Dr
To Provision for Current Tax

Deferred Tax A/c Dr
To Deferred Tax Liability A/c

OR
Deferred Tax Assets A/c .Dr
To Deferred Tax A/c

Tax Expense A/cDr
Deferred Tax A/cDr (In case DTA is created)
To Current Tax A/c
To Deferred Tax A/c (In case DTL is created)

P/L A/cDr
To Current Tax A/c

Permanent differences are the differences between taxable income and accounting income for a period
that originate in one period and do not reverse subsequently.
Examples:
- Expenditure disallowed as per Income Tax Act (Forever)
- Excess expenditure allowed by Income Tax Act, 1961 in respect of Scientific Expenditure

Timing differences are the differences between taxable income and accounting income for a period that
originate in one period and are capable of reversal in one or more subsequent periods.
Examples:
- Depreciation rate/method different as per Accounts and Income tax Calculation
- Expenditure of the nature mentioned in Section 43B (e.g. sales tax charged in account on
accrual basis but not paid; such sales tax will be an allowable expenditure in the year of
payment and a disallowable expenditure in the year in which accrued)

Hints for creation of DTL or DTA

When accounting profit/ loss is higher than taxable profit/loss: Deferred Tax liability is created or
Deferred tax asset is reversed.

When accounting profit/loss is less than taxable profit/loss: Deferred tax asset is created or Deferred
Tax Liability is reversed.

When taxable loss is carried forward for set off: Deferred Tax Asset is created.

When carried forward taxable loss is set off : Deferred Tax Asset is reversed.

However, Deferred Tax Asset (DTA) should be recognized and carried forward only to the extent that
there is a reasonable certainty that sufficient future taxable income will be available against which such
deferred tax assets can be reversed/ realized.
Example: Deferred Tax Asset should be created in respect of taxable loss being carried forward, when
there is reasonable certainty that carried forward taxable loss will be set off. (i.e. Adequate taxable
profit is expected in future)

The carrying amount of deferred tax assets should be reviewed at each balance sheet date. An enterprise
should write down the carrying amount of deferred tax asset to the extent that it is no longer reasonably
certain that sufficient profits will be available.

Such, written down value can be re-stated if it becomes virtually certain that sufficient profits will be
available (for set off).

Also at each balance sheet date, an enterprise re-assesses unrecognized deferred tax assets. The
enterprise recognizes previously unrecognized deferred tax assets to the extent that it has become
reasonably certain that sufficient future taxable income will be available against which such deferred
tax assets can be realized.

Presentation & Disclosure

In the Balance Sheet, a Deferred Tax Asset should be shown after the head INVESTMENT and
Deferred Tax Liability should be shown after the head UNSECURED LOAN.

Current Tax assets and liabilities should be separately shown with Deferred Tax assets and liabilities.

Deferred Tax asset is set-off with deferred tax liabilities when
- the enterprise has a legally enforceable right to set-off assets against liabilities
representing current tax; and
- the deferred tax assets and the deferred tax liabilities relate to taxes on income levied by
the same governing taxation laws.

The nature of the evidence supporting the recognition of deferred tax assets should be disclosed, if an
enterprise has unabsorbed depreciation or carry forward of losses under tax laws.






AS 24
DI SCONTI NUI NG OPERATI ONS


A discontinuing operation is a component of an enterprise:

(a) that the enterprise, pursuant to a single plan, is:
- disposing of substantially in its entirety (example demerger)
- disposing of piecemeal (selling and settling assets and liabilities one by one)
- terminating through abandonment; and

(b) that represents a separate major line of business or geographical area of operations; and

( c) that can be distinguished operationally and for financial reporting purposes.

Examples of activities that may not satisfy criteria (a) above but that can be discontinuing operations in
combination with other circumstances include:

- gradual or evolutionary phasing out of a product line or class of service;
- discontinuing, even if relatively abruptly, several products within an ongoing line of
business;
- shifting of some production or marketing activities for a particular line of business from
one location to another; and
- closing of a facility to achieve productivity improvements or other cost savings;
- Selling shares of subsidiary whose activities are similar to those of the parent or other
subsidiaries. (In case of Consolidated Financial Statements)

A reportable business segment or geographical segment as defined in AS-17 would normally satisfy
criteria (b) above.

A component that can be distinguished operationally and for financial reporting purposes {criteria [c]
above} if all the following conditions are met:

1. the operating assets and liabilities of the component can be directly attributed to it;
2. its revenue can be directly attributed to it;
3. at least a majority of its operating expenses can be directly attributed to it.


Presentation and Disclosure

An enterprise should include the following information relating to a discontinuing operation in its
financial statements beginning with the financial statements for the period in which the initial disclosure
event occurs and up to and including the period in which discontinuance is completed.:

I NI TI AL DI SCLOSURE:

1. A description of the discontinuing operation(s);
2. the business or geographical segment(s) in which it is reported as per AS 17
3. the date and nature of the initial disclosure event;
4. the date or period in which the discontinuance is expected to be completed if known or
determinable;
5. the carrying amounts, as of the balance sheet date, of the total assets to be disposed of and the
total liabilities to be settled;
6. revenue and expenses from such discontinuing operation in current reporting period;
7. pre-tax profit/loss from discontinuing operation during the current financial reporting period,
and income tax expense.
8. net cash flows attributable to the operating, investing, and financing activities of the
discontinuing operation during the current financial reporting period.


With respect to a discontinuing operation, the initial disclosure event is the occurrence of one of the
following, whichever occurs earlier

(a) the enterprise has entered into a binding sale agreement for substantially all of the assets
of the discontinuing operation; or
(b) the enterprises board of directors or similar governing body has both
(i) approved a formal plan; and
(ii) made an announcement of the plan.

Other Disclosures

When an enterprise disposes of assets or settles liabilities attributable to a discontinuing operation or
enters into binding agreements for the sale of such assets or the settlement of such liabilities, it should
include, in its financial statements, the following information when the event occurs.

(a) Gain or loss recognized on such disposal.
(b) Net selling prices (or range of prices) of those assets for whih the enterprise has
entered into binding contract, the expected timing of receipt of cash flow and the
carrying amount of those assets.

The disclosures required above should be presented in the notes to the financial statements except the
following, which should be shown on the face of the statements of profit or loss;

(a) pre-tax profit/loss from ordinary activities of discontinuing operation and related
income tax expense
(b) pre-tax gain or loss recognized on disposal of assets or settlement of liabilities.

If an enterprise abandons or withdraws from a plan that was previously reported as a discontinuing
operation, that fact, reason therefor and its effect should be disclosed.

Comparative information for prior periods in respect of discontinuing operations should also be deemed
as discontinuing operations.














AS-28
I MPAI RMENT OF ASSETS

Impairment of Assets means weakening in the value of asset. An enterprise should assess at
each balance sheet date whether there is any indication that an asset may be impaired. If
any such indication exists, the enterprise should estimate the recoverable amount of the
asset.

I ndication of Impairment of an asset
- External Sources of I nformation
Market value has declined significantly more than that would be expected as
a result of depreciation.
Adverse effect on the enterprise due to change in technology, market
conditions, etc.
Change in interest rates.
The carrying amount of the net assets of the reporting enterprise is more
than its market capitalization.
- I nternal Sources of I nformation
Physical damage of asset
Significant change in style or extent of use of asset.
Internal Reporting indicates that the economic performance of an asset is, or
will be, worse than expected.

Assets - This AS does not apply to
1. Inventories (AS-2)
2. Assets arising from Const. Contracts (AS-7)
3. Financial Assets including Investments (AS-13)

A Financial Asset is any asset that is :-
Cash ;
A contractual right to receive cash or another financial asset
from another enterprise;
A contractual right to exchange financial instruments with
another enterprise under conditions that are potentially
favorable; or
An ownership interest in another enterprise.
4. Deferred tax Asset (AS-22)



RECOVERABLE AMOUNT : (of asset or cash generating unit)
HIGHER OF
- NET SELLING PRICE OF ASSET
- VALUE IN USE

Net Selling Price : It is the amount obtainable from sale of an asset in an arms length
transaction between knowledgeable, willing parties less the costs of disposal.

Value in use: It is the present value of estimated future Cash flow expected to arise from
continue use of an asset and from its disposal at the end of its useful life.

Estimating the value in use of an asset involves the following steps :-
- Estimation of future cash inflows & Outflows.
- Application of appropriate discount rate.
Projection of cash flow should be based on
- Most recent financial budgets/forecasts.
- Reasonable and supportable assumptions on the economic conditions. Giving more
weights to external evidence.
- Steady or declining growth rate for the period beyond the period covered by most
recent budgets/forecasts.

Increasing growth rate can be taken when it can be properly justified. However, growth rate
should not exceed the long-term average growth rate for the products/industries/countries in
which the enterprise operates.

Future cash flows should be estimated for the asset in its current condition.
Estimates of future cash flows should not include estimated future cash inflows or
outflows that are expected to arise from :-
a future restructuring to which an enterprise is not yet committed ; or
future capital expenditure that will improve or enhance the asset in excess of its
originally assessed standard of performance.

Estimates of future cash flows should not include;
Cash inflows or outflows from financing activities ; or
I ncome tax receipts or payments.

Foreign Currency Future Cash Flows : Future cash flows are estimated in the currency in
which they will be generated and then discounted using a discount rate appropriate for the
currency. An enterprise translates the present value obtained using the exchange rate at the
balance sheet date (describe in Accounting standard (AS)11, Accounting for the Effects of
changes in Foreign Exchange rates, as the closing rate).

DISCOUNT RATE : The discount rate should be PRE TAX RATE. That reflects current
market assessments of the time value of money & the risk specific to the asset.
As a starting point the enterprise may take into account the following rates :-
- Weighted average cost of capital.
- Market borrowing rate.
- Enterprises incremental Borrowing rates.


I MPAI RMENT LOSS
An Impairment Loss is the amount by which the carrying amount of an asset exceeds its
recoverable amount i.e.

Impairment loss = Carrying amount (-) Recoverable amount.

An impairment loss or a revalued asset is recognized as an expense in the statement
of Profit or Loss. However, an impairment loss on a revalued asset is recognized directly
against any revaluation surplus for the asset to the extent that the impairment loss does not
exceed the amount held in the revaluation surplus for the same asset.
When the amount estimated for an impairment loss is greater than the carrying
amount of the asset to which it relates, an enterprise should recognize a liability if, and
only if, that is required by another Accounting Standards.

Accounting entries required to be passed on recognition of Impairment loss :-
Impairment Loss A/c - Dr.
To Assets A/c.

P/L A/c / revaluation reserve A/c - Dr.
To Impairment Loss.

After such recognition Depreciation should be calculated prospectively considering such
loss :

CASH GENERATING UNIT
Cash generating unit is the
- smallest identifiable group of assets.
- that generates cash inflows.
- That are largely independent of the cash inflows from other assets.

If there is any indication that an asset may be impaired, the recoverable amount should be
estimated for the individual asset.
If it is not possible to estimate the recoverable amount of the individual asset, an enterprise
should determine the recoverable amount of the cash generating unit to which the asset
belongs (the assets cash generating unit).

The recoverable amount of an individual asset cannot be determined if:
1) The assets value in use cannot be estimated (to be close to its net selling price) ; and
2) The asset does not generate cash inflow from continuing use that is largely
independent from others.

Example : -
A bus company provides services under contract with a municipality that requires minimum
service on each of five separate routes. Assets devoted to each route and the cash flows
from each route can be identified separately. One of the routes operates at a significant
loss.
Because the enterprise does not have the option to curtail any one bus route, the lowest
level of identifiable cash inflows from continuing use that are largely independent of the
cash inflows from other assets or group of assets is the cash inflows generated by the five
routes together. The cash-generating unit for each route is the bus company as a whole


If an active market exists for the output produced by an asset or a group of asset, this asset
on group of assets should be identified as a separate cash-generating unit, even if output is
used internally.
In such case, future market price should be considered while calculating future cash
inflows and cash outflows.

Cash generating units should be identified consistently from period to period for the same
asset or types of assets; unless a change is justified.

Allocation of Goodwill & Corporate Assets

Corporate Assets ; are assets other than goodwill that contribute to the future cash flows of
both the cash generating unit under review and other cash generating units.
The Goodwill and Corporate Assets are allocated to cash generating unit on a Reasonable
and Consistent basis to the CGU under review.
Then, the recoverable amount of the CGU under review is compared with the carrying
amount of CGU (including allocated goodwill & corporate assets). (BOTTOM UP TEST).

The impairment loss should be allocated to reduce the carrying amount of the assets of the
unit in the following order :

(a) first, to allocated goodwill, and
(b) then, to the other asset of the CGU on a pro-rata basis based on carrying amount of
individual assets.

Entry to be passed :-
P/L A/c Dr. (with the amount of impairment loss)
To Goodwill (up to the allocated amount)
Other Assets (on pro-rata basis)

The carrying amount should not be reduced below to zero. The excess loss (i.e. beyond
zero) should be allocated to other assets on pro-rata basis.
A liability should be recognized for any remaining amount of an impairment loss for a
CGU that is required by another Accounting Standard.
If, in performing the bottom up test the enterprise could not allocate the carrying amount
of goodwill/Corporate on a reasonable and consistent basis to the CGU under review, the
enterprise should also perform a top-down test :- i.e.

1) Identify the smallest cash generating unit that includes the cash-generating unit
under review and to which the carrying amount of goodwill/Corporate assets can be
allocated on a reasonable and consistent basis (i.e. the large CGU is identified)
2) Then, compare the RA of the larger CGU with CA of larger CGU.
If; CA > RA, identify impairment loss and allocate it first to goodwill and then to
other assets on pro-rata basis.


Example Application of the Bottom-Up and Top-Down Tests to Goodwill

In this example, tax effects are ignored.

At the end of 20X0, enterprise M acquired 100% of enterprise Z for Rs. 3,000 lakhs. Z has 3
cash-generating units A, B and C with net fair values of Rs. 1,200 lakhs, Rs. 800 lakhs and
Rs. 400 lakhs respectively. M recognizes goodwill of rs. 600 lakhs (Rs. 3,000 lakhs less Rs.
2,400 lakhs) that relates to Z.

At the end of 20X4, A makes significant losses. Its recoverable amount is estimated to be
Rs. 1,350 lakhs. Carrying amounts are detailed below.

Carrying amounts at the end of 20X4 (Amount in Rs. lakhs)

End of 20X4 A B C Goodwill
Total
Net carrying amount 1,300 1,200 800 120
3,420

A- Goodwill Can be Allocated on a reasonable and Consistent Basis

At the date of acquisition of Z, the net fair values of A, B and C are considered a
reasonable basis for a pro-rata allocation of the goodwill to A, B and C.




Allocation of goodwill at the end of 20X4
A B C
Total
End of 20X0
Net fair values 1,200 800 400
2,400
Pro-rata 50% 33% 17%
100%
End of 20X4
Net carrying amount 1,300 1,200 800
3,300
Allocation of goodwill
(using the pro-rata above) 60 40 20
120

_______________________________________________
Net carrying amount
(after allocation of goodwill) 1,360 1,240 820
3,420

========================================

In accordance with the bottom-up test , M computes As recoverable amount to its
carrying amount after the allocation of the carrying amount of goodwill.

Application of bottom-up test (Amount in Rs. lakhs) End of 20X4
Carrying amount after allocation of goodwill (Schedule 2) 1,360
Recoverable amount 1,350
Impairment loss 10
====
M recognizes an impairment loss of Rs. 10 lakhs for A. The impairment loss is fully
allocated to the goodwill.
B- Goodwill Cannot Be Allocated on a Reasonable and Consistent Basis

There is no reasonable way to allocate the goodwill that arose on the acquisition of Z to A,
B and C. At the end of 20X4, Zs recoverable amount is estimated to be Rs. 3,400 lakhs.
At the end of 20X4, M first applies the bottom-up test. It compares As recoverable
amount to its carrying amount excluding the goodwill.

Application of bottom-up test (Amount in Rs. lakhs)
End of 20X4 A
Carrying amount 1,300
Recoverable amount 1,350
Impairment loss 0
Therefore, no impairment loss is recognized for A as a result of the bottom-up test.

Since the goodwill could not be allocated on a reasonable and consistent basis to A, M also
performs a top-down test. It compares the carrying amount of Z as a whole to its
recoverable amount (Z as a whole is the smallest cash-generating unit that includes A and
to which goodwill can be allocated on a reasonable and consistent basis).



Application of the top-down test (Amount in Rs. lakhs)

End of 20X4 A B C Goodwill Z
Carrying amount 1,300 1,200 800 120 3,420
Impairment loss arising from the bottom-up test 0 - - - 0
Carrying amount after the bottom-up test 1,300 1,200 800 120 3,420
Recoverable amount
3,400
Impairment loss arising from top-down test
20
Therefore, M recognizes an impairment loss of Rs. 20 lakhs that it allocates fully to
goodwill.

Reversal of an Impairment Loss
An enterprise should assess at each balance sheet date whether there is any indication that
an impairment loss recognized in prior accounting period may no longer exist or may have
decreased.

If any such indication exists, the enterprise should estimate the recoverable amount of that
asset.
Indications of such change are known through External & Internal sources of information.

Impairment loss recognized earlier is reversed.
Such reversal is recognized as income in Profit/loss statement. For Revalued asset, it
should be credited to revaluation reserve(As per AS-10) if earlier revaluation reserve A/c
has been debited on recognition of impairment loss.

Reversal of an Impairment Loss for a Cash Generating Unit
- First, reversal should be made to increase the carrying amount of assets other than
goodwill; and
- Then, to goodwill allocated to cash generating unit.

Impairment in case of Discontinuing Operations
If the enterprise wants to sell the discontinuing operation in its entirety, then the
recoverable amount for the entire unit is compared with the carrying amount of entire unit.

If the enterprise wants to dispose the discontinuing operation in piecemeal, then the
recoverable amount of individual assets are determined and compared with carrying
amount of individual assets.

If the enterprise abandons the discontinuing operation, the recoverable amount of
individual assets are determined and compared with carrying amount of individual assets.














AS-29
PROVI SI ONS, CONTI GENT LI ABI LI TI ES AND CONTI NGENT ASSETS

PROVI SI ON:
A provision is a liability which can be measured only by using a substantial degree of estimation.

Treatment : A provision should be recognized when:

(a) An enterprise has a present obligation as a result of past event
(b) It is probable that an outflow of resources embodying economic benefits will be
required to settle the obligation; and
(c) A reliable estimate can be made of the amount of the obligation.

Present Obligation: An obligation is a present obligation if, based on the evidence available, its
existence at the balance sheet date is considered Probable, i.e. more
likely than not.


Past Event: A Past event that leads to a present obligation is called an obligating event.


CONTI NGENT LI ABI LI TY:

1] A contingent liability is
A possible obligation that arises from past events
And; existence of which will be confirmed by the occurrence or non
occurrence of future events not wholly within the control of the enterprise

2] A contingent liability is
A present obligation that arises from past events
And; not recognized because of lower probability of outflow of resources or
non-availability of reliable estimate

Possible Obligation: An obligation is a present obligation if, based on the evidence
available, its existence at the balance sheet date is considered Not
Probable.

Treatment: An enterprise should not recognize a contingent liability.
It should be disclosed in financial statements unless the possibility of
outflow is remote.


CONTI NGENT ASSETS:

A contingent assets is a possible asset that arises from past events of the existence of which
will be confirmed only by the occurrence or non-occurrence of one or more uncertain
future events not wholly within the control of the enterprise.

Treatment: An enterprise should not recognize a contingent asset.
An enterprise should not be disclosed in financial statements.
It may be disclosed in the report of approving authority, where an inflow
is probable.
Other I mportant I ssues:

1. Provisioning is required for only those liabilities that exist at the balance sheet date.
( i.e. No provision is required for costs that need to be incurred to operate in
future.)
2. Where details of a proposed new law have yet to be finalized, an obligation arises
only when the legislation is virtually certain to be enacted. For example, huge
penalty shall be imposed on the enterprise if the proposed law is enacted. No
provisioning is required unless the virtual certainty of the enactment of the law is
established.
3. Where there are a number of similar obligations (e.g. product warranties) the
probability that an outflow will be required in settlement is determined by
considering the class of obligations as a whole.
4. If the reliable estimate of the liability cannot be made, it should be disclosed as a
contingent liability.
5. Where an enterprise is jointly & severally liable for an obligation:
Provision should be made for the portion on which enterprise has direct
liability.
The balance amount should be disclosed as contingent liability.
6. Gains from the expected disposal of assets should not be taken into account in
measuring a provision.
7. Reimbursement for expenditure of which provision is created, should be recognized
when and only when it is virtually certain that the reimbursement shall be received
on settlement of liability.

Such Reimbursement may be shown as a net figure in Profit & Loss statement but
should be presented in balance sheet as a separate asset (i.e. net provision not to be
shown)

8. A provision should be used only for expenditures for which the provision was
originally recognized. Provisions should also be reviewed at each balance sheet
date and if no longer required, it should be reversed.
9. Provision should not be recognized for future operating losses as it neither meets
the criteria of liability nor meets the criteria for recognition of provision.

RESTRUCTURI NG:

A restructuring is a program that is planned and controlled by management and materially
changes either:
(a) the scope of a business undertaken by an enterprise; or
(b) the manner in which the business is expected.

Restructuring may include the following:
(a) sale or termination of a line of business;
(b) the closure of business location in a region
(c) eliminating a layer of management;

Treatment: A provision for recognition criteria is recognized only when the recognition
criteria for provision is met.

A restructuring provision should include only the direct expenditures arising from the
restructuring, which are those that are both;
(a) necessarily entailed by the restructuring; and
(b) not associated with the ongoing activities of the enterprise.

Restructuring provision does not include costs like
(a) retraining or relocating continuing staff
(b) marketing expenses
(c) investments in new systems and distribution networks.

Identifiable future operating losses up to the date of a restructuring and gains on disposal
of assets (even if it is included as part of restructuring) are not included in provisions.


DI SCLOSURES:
The enterprise should disclose for each class of provision:
(a) the carrying amount at the beginning & end of the period
(b) additional provision made during the period
(c) amount used during the period
(d) amount reversed during the period
(e) nature of obligation & and expected time of incurrence
(f) indication about the uncertainties attached to the provisions

The enterprise should disclose for each class of contingent liabilities:
(a) an estimate of its financial effects
(b) an indication of the uncertainties relating to any outflow
(c) the possibility of any reimbursement

Where any of the information required is not disclosed because it is not practicable to do
so, that fact should be stated.

In extremely rare cases, disclosures can be expected to seriously harm the enterprise in a
dispute with other parties. In such cases, instead of detailed information, general nature of
dispute together with the reason of non-disclosures should be disclosed.



Example 1: Warranties

A manufacturer gives warranties et the time of sale to purchasers of its product. Under the
terms of the contract for the manufacturer undertakes to make good, by repairs or
replacement, manufacturer defects that become apparent within three years from the date
of sale. On past experience, it is probable (i.e. more likely than not) that there will be some
claims under the warranties.

Present obligation as a result of a past obligating event- The obligating event is the sale of
the product with a warranty, which gives rise to an obligation

An outflow of resources embodying economics benefits in settlement- Probable for the
warranties as a whole.

Conclusion A provision is recognized for the best estimate of the costs of making good
under the warranty products sold before the balance sheet date.




Example 2 Contaminated Land- Legislation Virtually Certain to be Enacted

An enterprise in the oil industry causes contaminated but does not clean up because there is
no legislation requiring cleaning up, and the enterprise has been contaminating land for
several years. At 31 March 2005 it is virtually certain that a law requiring a clean up of
land already contaminated will be enacted shortly after the year end.

Present obligation as a result of a past obligating event- he obligating event is the
contamination of the land because of the virtually certainty of legislation requiring
cleaning up.

An outflow of resources embodying economics benefits in settlement- Probable.

Conclusion - A provision is recognized for the best estimate of the costs of the clean up.




Example 3: Offshore Oilfield

An enterprise operates an offshore oilfield where its licensing agreement requires it to
remove the oil rig at the end of production and restore the seabed. Ninety percent of the
eventual cost related to the removal of the oil rig and restoration of damage caused by
building it, and ten percent arise through the extraction of oil. At the balance sheet date,
the rig has been constructed but no oil has been extracted.

Present obligation as a result of past obligating event- The construction of the oil rig
created an obligation under the terms of the license to remove the rig and restore the
seabed and is thus as obligating event. At the balance sheet date, however, there is no
obligation to rectify the damage that will be caused by extraction of oil.

An outflow of resources embodying economics benefits in settlement- Probable.

Conclusion- A provision is recognized for the best estimate of ninety percent of the eventual
costs that relate to the removal of the oil rig and restoration of damage caused by building
it. There coasts are included as part of the cost of the oil rig. The ten percent of costs that
arise through the extraction of oil are recognized as a liability when the oil is extracted.

Example 4: Refunds Policy

A retail store has a policy of refunding purchases by dissatisfied customers, even though it
is under no legal obligation to do so. Its policy of making refunds is generally known.
Present obligation as a result of a past obligating event- The obligating event is the sale of
the product, which gives rise to an obligation because obligating also arise from normal
business practice, custom and a desire to a maintain good business relations or act in an
equitable manner
Outflows of resources embodying economic benefit in settlementProbable, a proportion of
goods are returned for refund.

Conclusion A provision is recognized for the best estimate of the costs of refunds.


Example 5: Legal Requirement to Fit Smoke Filters

Under new legislation, an enterprise is required to fit smoke filters to its factories by 30
September 2005. The enterprise has not fitted the smoke filters.

(a) At the balance sheet date of 31 March 2005

Present obligation as a result of a past obligating event There is no obligation because
there is no obligating event either for the costs of fitting smoke filters or for fines under the
legislation

Conclusion No provision is recognized for the cost of fitting the smoke filters.
(b) At the balance sheet date of 31 March 2006

Present obligations as a result of a past obligating event There is still no obligation for
the costs of fitting smoke filters because no obligating event has occurred (the fitting of the
filters). However, an obligation might arise to pay fines o penalties under the legislation
because the obligating event has occurred (the non-complaint operation of the factory).

An outflow of resources embodying economic benefits in settlement - Assessment of
probability of incurring fines and fines and penalties by non-compliant operation depends
on the details of the legislation and the stringency of the enforcement regime.

Conclusion No provision is recognized for the costs of fitting smoke filters. However, a
provision is recognized for the best estimate of any fines and penalties that are more likely
than not to be imposed.

Example 6: Staff retraining as a Result of Changes in the Income Tax System

The government introduces a number of changes to the income tax system. As a result of
these changes, an enterprise in the financial services sector will need to retrain a large
proportion of its administrative and sales workforce in order to ensure continued
compliance with financial services regulation. At the balance sheet date, no retraining of
staff has taken place.

Present obligation as a result of past obligating event There is no obligation because no
obligating event (retraining) has taken place.

Conclusion No provision is recognized.

Example 7: A Single Guarantee

During 2004-05, Enterprise A gives a guarantee of certain borrowing of Enterprise B,
whose financial condition at that time is sound. During 2005-06, the financial condition of
Enterprise B deteriorates and at 30 September 2005 Enterprise B goes into liquidation..
(a) At 31 March 2005

Present obligation as a result of a past obligating event The obligating event is the giving
of the guarantee, which gives rise to an obligation.

An outflow of resources embodying economics benefits in settlement No outflow of
benefits is probable at 31 March 2005.
Conclusion No provision is recognized. The guarantee is disclosed as a contingent
liability unless the probability of any is regarded as remote At 31 March 2006

Present obligation as a result of a past obligating event The obligating event is the giving
of the guarantee, which gives to a legal obligation.

Conclusion A provision is recognized for the best estimate of the obligation.
Note: This example deals with a single guarantee. If an enterprise has a portfolio of similar
guarantee, it will assess that portfolio as a whole in determining whether an outflow of
resources embodying economic benefit is probable. Where an enterprise gives guarantees
in exchange for a fee, revenue is recognized under AS 9, Revenue recognition.

Example 8: A Court Case

After a wedding in 2004-05, ten people died, possibly as a result of food poisoning from
products sold by the enterprise. Legal proceedings are started seeking damages from the
enterprise but it disputes liability. Up to the date of approval of the financial statements for
the year 31 March 2005, the enterprises lawyers advice that it is probable that the
enterprise will not be found liable. However, when the enterprise prepares the financial
statements for the year 31 March 2006, its lawyers advice that, owing to developments in
the case, it is probable that the enterprise will be found liable.

(a) At 31 March 2005

Present obligation as a result of a past obligating event On the basis of the evidence
available when the financial statements were approved, there is no present obligation as a
result of past events.
Conclusion No provision is recognized. The matter is disclosed as a contingent liability
unless the probability of any outflow is regarded as remote

(b) At 31 March 2006

Present obligation as a result of a past obligating event On the basis of the evidence
available, there is a present obligation.
An outflow of resources embodying economic benefits in settlement Probable.

Conclusion A provision is recognized for the best estimate of the amount to settle the
obligation.

Example 9A: Refurbishment Costs No Legislative Requirement

A furnace has a lining that needs to be replaced every five years for technical reasons. At
the balance sheet date, the lining has been in use for three years.
Present obligation as a result of a past obligating event- There is no present obligation.
Conclusion No provision is recognized.

The cost of replacing the lining is not recognized because, at the balance sheet date, no
obligation to replace the lining exits independently of the companys future actions even
the intention to incur the expenditure depends on the company deciding to continue
operating the furnace or to replace the lining.


Example 9B: Refurbishment Costs Legislative Requirement

An Airline is required by law to overhaul its aircraft once every three years.

Present obligation as a result of a past obligating event There is no present obligation.
Conclusion No provision is recognized.

The costs of overhauling aircraft are not recognized as a provision for the same reason as
the cost of replacing the lining is not recognized as a provision in example 9A. Even a legal
requirement to overhaul does not make the cost of the overhaul a liability, because no
obligation exits to overhaul the aircraft independently of the enterprises future actions
the enterprise could avoid the future expenditure by its future actions, for example by
selling the aircraft.









Accounting Standard 1: Disclosure of Accounting Policies
Significant Accounting Policies followed in preparation and
presentation of financial statements should form part thereof and be
disclosed at one place in the financial statements.
Any change in the accounting policies having a material effect in the
current period or future periods should be disclosed. The amount by which
any item in financial statements is affected by such change should be
disclosed to the extent ascertainable. If the amount is not ascertainable
the fact should be indicated.
If fundamental assumptions (going concern, consistency and accrual)
are not followed, fact to be disclosed.
Major considerations governing selection and application of accounting
policies are i) Prudence, ii) Substance over form and iii) Materiality.
The ICAI has made an announcement that till the issuance of
Accounting Standards on (i) Financial Instruments : Presentation, (ii)
Financial Instruments : Disclosures and (iii) Financial Instruments :
Recognition and Measurement, an enterprise should provide information
regarding the extent of risks to which an enterprise is exposed and as a
minimum, make following disclosures in its financial statements:
a. category-wise quantitative data about derivative instruments that
are outstanding at the balance sheet date,
b. the purpose, viz. hedging or speculation, for which such derivative
instruments have been acquired, and
c. the foreign currency exposures that are not hedged by a derivative
instrument or otherwise.
This announcement is applicable in respect of financial statements for the
accounting period(s) ending on or after March 31, 2006.
Accounting Standard 2: Valuation of Inventories
This standard should be applied in accounting for inventories other than
WIP arising under construction contracts, WIP of service providers, shares,
debentures and financial instruments held as stock in trade, producers
inventories of livestock, agricultural and forest products and mineral oils,
ores and gases to the extent measured at net realisable value in
accordance with well established practices in those industries.
Inventories are assets held for sale in ordinary course of business, in
the process of production of such sale, or in form of materials to be
consumed in production process or rendering of services.
Inventories do not include machinery spares which can be used with an
item of fixed asset and whose use is irregular.
Net realisable value is the estimated selling price less the estimated
costs of completion and estimated costs necessary to make the sale.
fb.com//SUPERWHIZZ4U CACWACS.WORDPRESS.COM
Cost of inventories should comprise all costs incurred for bringing the
inventories to their present location and condition.
Inventories should be valued at lower of cost and net realisable value.
Generally, weighted average cost or FIFO method is used in cases where
goods are ordinarily interchangeable.
Specific Identification Method to be used when goods are not ordinarily
interchangeable or have been segregated for specific projects.
Disclose the accounting policies adopted including the cost formula
used, total carrying amount of inventories and its classification.
Also refer ASI 2 deals with accounting of machinery spares
Accounting Standard 3: Cash Flow Statements
Prepare and present a cash flow statement for each period for which
financial statements are prepared.
A cash flow statement should report cash flows during the period
classified by operating, investing and financial activities.
Operating activities are the principal revenue producing activities of the
enterprise other than investing or financing activities.
Investing activities are the acquisition and disposal of long term assets
and other investments not included in cash equivalents.
Financing activities are activities that result in changes in the size and
composition of the owners capital and borrowings of the enterprise.
A cash flow statement for operating activities should be prepared by
using either the direct method or the indirect method. For investing and
financing activities cash flows should be prepared using the direct method.
Cash flows arising from transactions in a foreign currency should be
recorded in enterprises reporting currency by applying the exchange rate
at the date of the cash flow.
Investing and financing transactions that do not require the use of cash
and cash equivalent balances should be excluded.
An enterprise should disclose the components of cash and cash
equivalents together with reconciliation of amounts as disclosed to
amounts reported in the balance sheet.
An enterprise should disclose together with a commentary by the
management the amount of significant cash and cash equivalent balances
held by it that are not available for use.
Accounting Standard 4: Contingencies and Events Occurring after the Balance
Sheet Date
A contingency is a condition or situation the ultimate outcome of which
will be known or determined only on the occurrence or non-occurrence of
uncertain future event/s.
fb.com//SUPERWHIZZ4U CACWACS.WORDPRESS.COM
Events occurring after the balance sheet date are those significant
events both favourable and unfavourable that occur between the balance
sheet date and the date on which the financial statements are approved.
Amount of a contingent loss should be provided for by a charge in P & L
A/c if it is probable that future events will confirm that an asset has been
impaired or a liability has been incurred as at the balance sheet date and a
reasonable estimate of the amount of the loss can be made.
Existence of contingent loss should be disclosed if above conditions are
not met, unless the possibility of loss is remote.
Contingent Gains if any, not to be recognised in the financial
statements.
Material change in the position due to subsequent events be accounted
or disclosed.
Proposed or declared dividend for the period should be adjusted.
Material event occurring after balance sheet date affecting the going
concern assumption and financial position be appropriately dealt with in
the accounts.
Contingencies or events occurring after the balance sheet date and the
estimate of the financial effect of the same should be disclosed.

Note: The underlined paras/words have been withdrawn on issuance of AS
29 effective for accounting periods commencing on or after 1-4-2004.
Accounting Standard 5: Net Profit/Loss for the Period, Prior Period Items and
Changes in Accounting Policies
All items of income and expense, which are recognised in a period,
should be included in determination of net profit or loss for the period
unless an accounting standard requires or permits otherwise.
Prior period, extraordinary items be separately disclosed in a manner
that their impact on current profit or loss can be perceived. Nature and
amount of significant items be provided. Extraordinary items should be
disclosed as a part of profit or loss for the period.
Effect of a change in the accounting estimate should be included in the
determination of net profit or loss in the period of change and also future
periods if it is expected to affect future periods.
Change in accounting policy, which has a material effect, should be
disclosed. Impact and the adjustment arising out of material change
should be disclosed in the period in which change is made. If the change
does not have a material impact in the current period but is expected to
have a material effect in future periods then the fact should be disclosed.
fb.com//SUPERWHIZZ4U CACWACS.WORDPRESS.COM
Accounting policy may be changed only if required by the statute or for
compliance with an accounting standard or if the change would result in
appropriate presentation of the financial statements.
A change in accounting policy on the adoption of an accounting
standard should be accounted for in accordance with the specific
transitional provisions, if any, contained in that accounting standard.
Accounting Standard 6: Depreciation Accounting
Standard does not apply to depreciation in respect of forests,
plantations and similar regenerative natural resources, wasting assets
including expenditure on exploration and extraction of minerals, oils,
natural gas and similar non-regenerative resources, expenditure on
research and development, goodwill and livestock. Special considerations
apply to these assets.
Allocate depreciable amount of a depreciable asset on systematic basis
to each accounting year over useful life of asset.
Useful life may be reviewed periodically after taking into consideration
the expected physical wear and tear, obsolescence and legal or other
limits on the use of the asset.
Basis for providing depreciation must be consistently followed and
disclosed. Any change to be quantified and disclosed.
A change in method of depreciation be made only if required by statute,
for compliance with an accounting standard or for appropriate
presentation of the financial statements. Revision in method of
depreciation be made from date of use. Change in method of charging
depreciation is a change in accounting policy and be quantified and
disclosed.
In cases of addition or extension which becomes integral part of the
existing asset depreciation to be provided on adjusted figure prospectively
over the residual useful life of the asset or at the rate applicable to the
asset.
Where the historical cost undergoes a change due to fluctuation in
exchange rate, price adjustment etc. depreciation on the revised
unamortised amount should be provided over the balance useful life of the
asset.
On revaluation of asset depreciation should be based on revalued
amount over balance useful life. Material impact on depreciation should be
disclosed.
Deficiency or surplus in case of disposal, destruction, demolition etc. be
disclosed separately, if material.
Historical cost, amount substituted for historical cost, depreciation for
the year and accumulated depreciation should be disclosed.
fb.com//SUPERWHIZZ4U CACWACS.WORDPRESS.COM
Depreciation method used should be disclosed. If rates applied are
different from the rates specified in the governing statute then the rates
and the useful life be also disclosed.
Accounting Standard 7 : Accounting for Construction Contracts (Revised 2002)
Applicable to accounting for construction contract.
Construction contract may be for construction of a single/combination
of interrelated or interdependent assets.
A fixed price contract is a contract where contract price is fixed or per
unit rate is fixed and in some cases subject to escalation clause.
A cost plus contract is a contract in which contractor is reimbursed for
allowable or defined cost plus percentage of these cost or a fixed fee.
In a contract covering a number of assets, each asset is treated as a
separate construction contract when there are:
separate proposal;
subject to separate negotiations and the contractor and
customer is able to accept/reject that part of the contract;
identifiable cost and revenues of each asset
A group of contracts to be treated as a single construction contract
when
they are negotiated as a single package;
contracts are closely interrelated with an overall profit margin;
and
contracts are performed concurrently or in a continuous
sequence.
Additional asset construction to be treated as separate construction
contract when
assets differs significantly in design/technology/function from
original contract assets.
a price negotiated without regard to original contract price
Contract revenue comprises of
initial amount and
variations in contract work, claims and incentive payments that
will probably result in revenue and are capable of being reliably
measured.
Contract cost comprises of
costs directly relating to specific contract
costs attributable and allocable to contract activity
fb.com//SUPERWHIZZ4U CACWACS.WORDPRESS.COM
other costs specifically chargeable to customer under the terms
of contracts.
Contract Revenue and Expenses to be recognised, when outcome can
be estimated reliably up to stage of completion on reporting date.
In Fixed Price Contract outcome can be estimated reliably when
total contract revenue can be measured reliably.
it is probable that economic benefits will flow to the enterprise;
contract cost and stage of completion can be measured reliably
at reporting date; and
contract costs are clearly identified and measured reliably for
comparing actual costs with prior estimates.
In cost plus contract outcome is estimated reliably when
it is probable that economic benefits will flow to the enterprise;
and
contract cost whether reimbursable or not can be clearly
identified and measured reliably.
When outcome of a contract cannot be estimated reliably
revenue to the extent of which recovery of contract cost is
probable should be recognised;
contract cost should be recognised as an expense in the period
in which they are incurred; and
An expected loss should be recognised as expense.
When uncertainties no longer exist revenue and expenses to be
recognised as mentioned above when outcomes can be estimated reliably.
When it is probable that contract costs will exceed total contract
revenue, the expected loss should be recognised as an expense
immediately.
Change in estimate to be accounted for as per AS 5.
An enterprise to disclose
contract revenue recognised in the period.
method used to determine recognised contract revenue.
methods used to determine the stage of completion of contracts
in progress.
For contracts in progress an enterprise should disclose
the aggregate amount of costs incurred and recognised profits
(less recognised losses) up to the reporting date.
amount of advances received and
fb.com//SUPERWHIZZ4U CACWACS.WORDPRESS.COM
amount of retention.
An enterprise should present
gross amount due from customers for contract work as an asset
and
the gross amount due to customers for contract work as a
liability.
Accounting Standard 8: Accounting for Research and Development
Note: In view of operation of AS 26, this Standard stands withdrawn.
Accounting Standard 9: Revenue Recognition
Standard does not deal with revenue recognition aspects of revenue
arising from construction contracts, hire-purchase and lease agreements,
government grants and other similar subsidies and revenue of insurance
companies from insurance contracts. Special considerations apply to these
cases.
Revenue from sales and services should be recognised at the time of sale
of goods or rendering of services if collection is reasonably certain; i.e.,
when risks and rewards of ownership are transferred to the buyer and when
effective control of the seller as the owner is lost.
In case of rendering of services, revenue must be recognised either on
completed service method or proportionate completion method by relating
the revenue with work accomplished and certainty of consideration
receivable.
Interest is recognised on time basis, royalties on accrual and dividend
when owners right to receive payment is established.
Disclose circumstances in which revenue recognition has been postponed
pending significant uncertainties.
Also refer ASI 14 (withdrawing GC 3/2002) deals with the manner of disclosure of
excise duty in presentation of revenue from sales transactions (turnover).
Accounting Standard 10: Accounting for Fixed Assets
Fixed asset is an asset held for producing or providing goods and/or
services and is not held for sale in the normal course of the business.
Cost to include purchase price and attributable costs of bringing asset to
its working condition for the intended use. It includes financing cost for
period up to the date of readiness for use.
Self-constructed assets are to be capitalised at costs that are specifically
related to the asset and those which are allocable to the specific asset.
Fixed asset acquired in exchange or part exchange should be recorded at
fair market value or net book value of asset given up adjusted for balancing
payment, cash receipt etc. Fair market value is determined with reference
to asset given up or asset acquired.
fb.com//SUPERWHIZZ4U CACWACS.WORDPRESS.COM
Revaluation, if any, should be of class of assets and not an individual asset.
Basis of revaluation should be disclosed.
Increase in value on revaluation be credited to Revaluation Reserve while
the decrease should be charged to P & L A/c.
Goodwill should be accounted only when paid for.
Assets acquired on hire purchase be recorded at cash value to be shown
with appropriate note about ownership of the same. (Not applicable for
assets acquired after 1st April, 2001 in view of AS 19 Leases becoming
effective).
Gross and net book values at beginning and end of year showing additions,
deletions and other movements, expenditure incurred in course of
construction and revalued amount if any be disclosed.
Assets should be eliminated from books on disposal/when of no utility
value.
Profit/Loss on disposal be recognised on disposal to P & L statement.
Also refer ASI 2 which deals with accounting for machinery spares.
Accounting Standard 11: The Effects of Changes in Foreign Exchange Rates
(Revised 2003)
The Statement is applied in accounting for transactions in foreign currency
and translating financial statements of foreign operations. It also deals
with accounting of forward exchange contract.
Initial recognition of a foreign currency transaction shall be by applying the
foreign currency exchange rate as on the date of transaction. In case of
voluminous transactions a weekly or a monthly average rate is permitted, if
fluctuation during the period is not significant.
At each Balance Sheet date foreign currency monetary items such as cash,
receivables, payables shall be reported at the closing exchange rates
unless there are restrictions on remittances or it is not possible to effect
an exchange of currency at that rate. In the latter case it should be
accounted at realisable rate in reporting currency. Non monetary items
such as fixed assets, investment in equity shares which are carried at
historical cost shall be reported at the exchange rate on the date of
transaction. Non monetary items which are carried at fair value shall be
reported at the exchange rate that existed when the value was determined.

Note: Schedule VI to the Companies Act, 1956, provides that any increase
or reduction in liability on account of an asset acquired from outside India
in consequence of a change in the rate of exchange, the amount of such
increase or decrease, should added to, or, as the case may be, deducted
from the cost of the fixed asset.

fb.com//SUPERWHIZZ4U CACWACS.WORDPRESS.COM
Therefore, for fixed assets, the treatment described in Schedule VI will be
in compliance with this standard, instead of stating it at historical cost.
Exchange differences arising on the settlement of monetary items or on
restatement of monetary items on each balance sheet date shall be
recognised as expense or income in the period in which they arise.
Exchange differences arising on monetary item which in substance, is net
investment in a non integral foreign operation (long term loans) shall be
credited to foreign currency translation reserve and shall be recognised as
income or expense at the time of disposal of net investment.
The financial statements of an integral foreign operation shall be
translated as if the transactions of the foreign operation had been those of
the reporting enterprise; i.e., it is initially to be accounted at the exchange
rate prevailing on the date of transaction.
For incorporation of non integral foreign operation, both monetary and non
monetary assets and liabilities should be translated at the closing rate as
on the balance sheet date. The income and expenses should be translated
at the exchange rates at the date of transactions. The resulting exchange
differences should be accumulated in the foreign currency translation
reserve until the disposal of net investment. Any goodwill or capital reserve
on acquisition on non-integral financial operation is translated at the
closing rate.
In Consolidated Financial Statement (CFS) of the reporting enterprise,
exchange difference arising on intra group monetary items continues to be
recognised as income or expense, unless the same is in substance an
enterprises net investment in non integral foreign operation.
When the financial statements of non integral foreign operations of a
different date are used for CFS of the reporting enterprise, the assets and
liabilities are translated at the exchange rate prevailing on the balance
sheet date of the non integral foreign operations. Further adjustments are
to be made for significant movements in exchange rates upto the balance
sheet date of the reporting currency.
When there is a change in the classification of a foreign operation from
integral to non integral or vice versa the translation procedures applicable
to the revised classification should be applied from the date of
reclassification.
Exchange differences arising on translation shall be considered for
deferred tax in accordance with AS 22.
Forward Exchange Contract may be entered to establish the amount of the
reporting currency required or available at the settlement date of the
transaction or intended for trading or speculation. Where the contracts are
not intended for trading or speculation purposes the premium or discount
arising at the time of inception of the forward contract should be amortized
as expense or income over the life of the contract. Further, exchange
fb.com//SUPERWHIZZ4U CACWACS.WORDPRESS.COM
differences on such contracts should be recognised in the P & L A/c in the
reporting period in which there is change in the exchange rates. Exchange
difference on forward exchange contract is the difference between
exchange rate at the reporting date and exchange difference at the date of
inception of the contract for the underlying currency.
Profit or loss arising on the renewal or cancellation of the forward contract
should be recognised as income or expense for the period. A gain or loss on
forward exchange contract intended for trading or speculation should be
recognised in the profit and loss statement for the period. Such gain or loss
should be computed with reference to the difference between forward rate
on the reporting date for the remaining maturity period of the contract and
the contracted forward rate. This means that the forward contract is
marked to market. For such contract, premium or discount is not
recognised separately.
Disclosure to be made for:
o Amount of exchange difference included in Profit and Loss
statement.
o Net exchange difference accumulated in Foreign Currency
Translation Reserve.
o In case of reclassification of significant foreign operation, the
nature of the change, the reasons for the same and its impact on
the shareholders fund and the impact on the Net Profit and Loss for
each period presented.
Non mandatory Disclosures can be made for foreign currency risk
management policy.
Accounting Standard 12: Accounting for Government Grants
Grants can be in cash or in kind and may carry certain conditions to be
complied.
Grants should not be recognised unless reasonably assured to be realized
and the enterprise complies with the conditions attached to the grant.
Grants towards specific assets should be deducted from its gross value.
Alternatively, it can be treated as deferred income in P & L A/c on rational
basis over the useful life of the depreciable asset. Grants related to non-
depreciable asset should be generally credited to Capital Reserves unless
it stipulates fulfilment of certain obligations. In the latter case the grant
should be credited to the P & L A/c over a reasonable period. The deferred
income balance to be shown separately in the financial statements.
Grants of revenue nature to be recognised in the P & L A/c over the period
to match with the related cost, which are intended to be compensated.
Such grants can be treated as other income or can be reduced from related
expense.
fb.com//SUPERWHIZZ4U CACWACS.WORDPRESS.COM
Grants by way of promoters contribution is to be credited to Capital
Reserves and considered as part of shareholders funds.
Grants in the form of non-monetary assets, given at concessional rate,
shall be accounted at their acquisition cost. Asset given free of cost be
recorded at nominal value.
Grants receivable as compensation for losses/expenses incurred should be
recognised and disclosed in P & L A/c in the year it is receivable and
shown as extraordinary item, if material in amount.
Grants when become refundable, be shown as extraordinary item.
Revenue grants when refundable should be first adjusted against
unamortised deferred credit balance of the grant and the balance should
be charged to the P & L A/c.
Grants against specific assets on becoming refundable are recorded by
increasing the value of the respective asset or by reducing Capital Reserve
/ Deferred income balance of the grant, as applicable. Any such increase in
the value of the asset shall be depreciated prospectively over the residual
useful life of the asset.
Accounting policy adopted for grants including method of presentation,
extent of recognition in financial statements, accounting of non-monetary
assets given at concession/ free of cost be disclosed.
Accounting Standard 13: Accounting for Investments
Current investments and long term investments be disclosed distinctly with
further sub-classification into government or trust securities, shares,
debentures or bonds, investment properties, others unless it is required to
be classified in other manner as per the statute governing the enterprise.
Cost of investment to include acquisition charges including brokerage,
fees and duties.
Investment properties should be accounted as long term investments.
Current investments be carried at lower of cost and fair value either on
individual investment basis or by category of investment but not on global
basis.
Long term investments be carried at cost. Provision for decline (other than
temporary) to be made for each investment individually.
If an investment is acquired by issue of shares/securities or in exchange of
an asset, the cost of the investment is the fair value of the securities
issued or the assets given up. Acquisition cost may be determined
considering the fair value of the investments acquired.
Changes in the carrying amount and the difference between the carrying
amount and the net proceeds on disposal be charged or credited to the P &
L A/c.
fb.com//SUPERWHIZZ4U CACWACS.WORDPRESS.COM
Disclosure is required for the accounting policy adopted, classification of
investments; profit / loss on disposal and changes in carrying amount of
such investment.
Significant restrictions on right of ownership, realisability of investments
and remittance of income and proceeds of disposal thereof be disclosed.
Disclosure should be made of aggregate amount of quoted and unquoted
investments together with aggregate value of quoted investments.
Accounting Standard 14: Accounting for Amalgamations
Amalgamation in nature of merger be accounted for under Pooling of
Interest Method and in nature of purchase be accounted for under
Purchase Method.
Under the Pooling of the Interest Method, assets, liabilities and reserves of
the transferor company be recorded at existing carrying amount and in the
same form as it was appearing in the books of the transferor.
In case of conflicting accounting policies, a uniform policy be adopted on
amalgamation. Effect on financial statement of such change in policy be
reported as per AS5.
Difference between the amount recorded as share capital issued and the
amount of capital of the transferor company should be adjusted in
reserves.
Under Purchase Method, all assets and liabilities of the transferor
company be recorded at existing carrying amount or consideration be
allocated to individual identifiable assets and liabilities on basis of fair
values at date of amalgamation. The reserves of the transferor company
shall lose its identity. The excess or shortfall of consideration over value of
net assets be recognised as goodwill or capital reserve.
Any non-cash item included in the consideration on amalgamation should
be accounted at fair value.
In case the scheme of amalgamation sanctioned under the statute
prescribes a treatment to be given to the transferor company reserves on
amalgamation, same should be followed. However a description of
accounting treatment given to reserves and the reasons for following a
treatment different from that prescribed in the AS is to be given. Also
deviations between the two accounting treatments given to the reserves
and the financial effect, if any, arising due to such deviation is to be
disclosed. (Limited Revision to AS 14 w.e.f 1-4-2004)
Disclosures to include effective date of amalgamation for accounting, the
method of accounting followed, particulars of the scheme sanctioned.
In case of amalgamation under the Pooling of Interest Method the
treatment given to the difference between the consideration and the value
of the net identified assets acquired is to be disclosed. In case of
amalgamation under the Purchase Method the consideration and the
fb.com//SUPERWHIZZ4U CACWACS.WORDPRESS.COM
treatment given to the difference compared to the value of the net
identifiable assets acquired including period of amortization of goodwill
arising on amalgamation is to be disclosed.
Accounting Standard 15: Accounting for Retirement Benefits in the Financial
Statement of Employers
For retirement benefits of provident fund and other defined contribution
schemes, contribution payable by employer and any shortfall on collection
from employees if any for a year be charged to P & L A/c. Excess payment
be treated as pre-payment.
For gratuity and other defined benefit schemes, accounting treatment will
depend on the type of arrangements, which the employer has entered into.
If payment for retirement benefits out of employers funds, appropriate
charge to P & L to be made through a provision for accruing liability,
calculated according to actuarial valuation.
If liability for retirement benefit funded through creation of trust, cost
incurred be determined actuarially. Excess/ shortfall of contribution paid
against amount required to meet accrued liability as certified by actuary be
treated as pre-payment or charged to P & L account
If liability for retirement benefit is funded through a scheme administered
by an insurer, an actuarial certificate or confirmation from insurer to be
obtained. The excess/ shortfall of the contribution paid against the amount
required to meet accrued liability as certified by actuary or confirmed by
insurer should be treated as pre-payment or charged to P & L account.
Any alteration in the retirement benefit cost should be charged or credited
to P & L A/c and change in actuarial method should be disclosed as per AS
5.
Financial statements to disclose method by which retirement benefit cost
have been determined.
Accounting Standard 15 - Employee Benefits Effective from accounting period
commencing on or after 1 April, 2006.
Applicable to Level II & III enterprises (subject to certain relaxation
provided), if number of persons employed is 50 or more.
For Enterprises employing less than 50 persons, any method of accrual
for accounting long-term employee benefits liability is allowed.
Employee benefits are all forms of consideration given in exchange of
services rendered by employees. Employee benefits include those provided
under formal plan or as per informal practices which give rise to an
obligation or required as per legislative requirements. These include
performance bonus (payable within 12 months) and non-monetary benefits
such as housing, car or subsidized goods or services to current employees,
post-employment benefits, deferred compensation and termination
fb.com//SUPERWHIZZ4U CACWACS.WORDPRESS.COM
benefits. Benefits provided to employees spouses, children, dependents,
nominees are also covered.
Short-term employee benefits should be recognised as an expense
without discounting, unless permitted by other AS to be included as a cost
of an asset.
Cost of accumulating compensated absences is accounted on accrual
basis and cost of non-accumulating compensated absences is accounted
when the absences occur.
Cost of profit sharing and bonus plans are accounted as an expense
when the enterprise has a present obligation to make such payments as a
result of past events and a reliable estimate of the obligation can be made.
While estimating, probability of payment at a future date is also
considered.
Post employment benefits can either be defined contribution plans,
under which enterprises obligation is limited to contribution agreed to be
made and investment returns arising from such contribution, or defined
benefit plans under which the enterprises obligation is to provide the
agreed benefits. Under the later plans if actuarial or investment experience
are worse then expected, obligation of the enterprise may get increased at
subsequent dates.
In case of a multi-employer plans, an enterprise should recognise its
proportionate share of the obligation. If defined benefit cost can not be
reliably estimated it should recognise cost as if it were a defined
contribution plan, with certain disclosures (in para 30)
State Plans and Insured Benefits are generally Defined Contribution
Plan.
Cost of Defined contribution plan should be accounted as an expense
on accrual basis. In case contribution does not fall due within 12 months
from the balance sheet date, expense should be recognised for discounted
liabilities.
The obligation that arises from the enterprises informal practices
should also be accounted with its obligation under the formal defined
benefit plan.
For balance sheet purpose, the amount to be recognised as a defined
benefit liability is the present value of the defined benefit obligation
reduced by (a) past service cost not recognised and (b) the fair value of the
plan asset. An enterprise should determine the present value of defined
benefit obligations (through actuarial valuation at intervals not exceeding
three years) and the fair value of plan assets (on each balance sheet date)
so that amount recognised in the financial statements do not differ
materially from the liability required. In case of fair value of plan asset is
higher than liability required, the present value of excess should be treated
as an asset.
fb.com//SUPERWHIZZ4U CACWACS.WORDPRESS.COM
For determining Cost to be recognised in the profit and loss account for
the Defined benefit plan, following should be considered :
Current service cost
Interest cost
Expected return of any plan assets
Actuarial gains and losses
Past service cost
Effect of any curtailment or settlement
Surplus arising out of present value of plan asset being higher
than obligation under the plan.
Actuarial Assumptions comprise of following :
Mortality during and after employment
Employee Turnover
Plan members eligible for benefits
Claim rate under medical plans
The discount rate, based on market yields on Government bonds
of relevant maturity.
Future salary and benefits levels
In case of medical benefits, future medical costs (including
administration cost, if material)
Rate of return expectation on plan assets.
Actuarial gains / losses should be recognised in profit and loss account as
income / expenses.
o Past Service Cost arises due to introduction or changes in the defined
benefit plan. It should be recognised in the profit and loss account over the
period of vesting. Similarly, surplus on curtailment is recognised over the
vesting period. However, for other long term employee benefits, past
service cost is recognised immediately.
o The expected return on plan assets is a component of current service cost.
The difference between expected return and the actual return on plan
assets is treated as an actuarial gain / loss, which is also recognised in the
profit and loss account.
o An enterprise should disclose information by which users can evaluate the
nature of its defined benefit plans and the financial effects of changes in
those plans during the period. For disclosures requirement refer to para
120 to 125 of the standard.
o Termination benefits are accounted as a liability and expense only when
the enterprise has a present obligation as a result of a past event, outflow
fb.com//SUPERWHIZZ4U CACWACS.WORDPRESS.COM
of resources will be required to settle the obligation and a reliable estimate
of it can be made. Where termination benefits fall due beyond 12 months
period, the present value of liability needs to be worked out using the
discount rate. If termination benefit amount is material, it should be
disclosed separately as per AS 5 requirements. As per the transitional
provisions expenses on termination benefits incurred up to 31 March,
2009 can be deferred over the pay-back period, not beyond 1 April, 2010.
o Transitional Provisions
When enterprise adopts the revised standard for the first time, additional
charge on account of change in a liability, compared to pre-revised AS
15, should be adjusted against revenue reserves and surplus.
Accounting Standard 16: Borrowing Costs
Statement to be applied in accounting for borrowing costs.
Statement does not deal with the actual or imputed cost of owners
equity/preference capital.
Borrowing costs that are directly attributable to the acquisition,
construction or production of any qualifying asset (assets that takes a
substantial period of time to get ready for its intended use or sale. should
be capitalized.) Generally, a period of 12 months is considered as a
substantial period of time (ASI-1).
Income on the temporary investment of the borrowed funds be deducted
from borrowing costs.
In case of funds obtained generally and used for obtaining a qualifying
asset, the borrowing cost to be capitalized is determined by applying
weighted average of borrowing cost on outstanding borrowings, other than
borrowings for obtaining qualifying asset.
Capitalization of borrowing costs should be suspended during extended
periods in which development is interrupted. When the expected cost of
the qualifying asset exceeds its recoverable amount or Net Realizable
Value, the carrying amount is written down.
Capitalization should cease when activity is completed substantially or if
completed in parts, in respect of that part, all the activities for its intended
use or sale are complete.
Financial statements to disclose accounting policy adopted for borrowing
cost and also the amount of borrowing costs capitalized during the period.
In case exchange difference on foreign currency borrowings represent
saving in interest, compared to interest rate for the local currency
borrowings, it should be treated as part of interest cost for AS 16 (ASI-10).
Accounting Standard 17: Segment Reporting
Requires reporting of financial information about different types of
products and services an enterprise provides and different geographical
areas in which it operates.
fb.com//SUPERWHIZZ4U CACWACS.WORDPRESS.COM
A business segment is a distinguishable component of an enterprise
providing a product or service or group of products or services that is
subject to risks and returns that are different from other business
segments.
A geographical segment is distinguishable component of an enterprise
providing products or services in a particular economic environment that is
subject to risks and returns that are different from components operating
in other economic environments.
Internal organizational management structure, internal financial reporting
system is normally the basis for identifying the segments.
The dominant source and nature of risk and returns of an enterprise should
govern whether its primary reporting format will be business segments or
geographical segments.
A business segment or geographical segment is a reportable segment if (a)
revenue from sales to external customers and from transactions with other
segments exceeds 10% of total revenues (external and internal) of all
segments; or (b) segment result, whether profit or loss, is 10% or more of (i)
combined result of all segments in profit or (ii) combined result of all
segments in loss whichever is greater in absolute amount; or (c) segment
assets are 10% or more of all the assets of all the segments. If there is
reportable segment in the preceding period (as per criteria), same shall be
considered as reportable segment in the current year.
If total external revenue attributable to reportable segment constitutes
less than 75% of total revenues then additional segments should be
identified, for reporting.
Under primary reporting format for each reportable segment the enterprise
should disclose external and internal segment revenue, segment result,
amount of segment assets and liabilities, cost of fixed assets acquired,
depreciation, amortization of assets and other non cash expenses.
Interest expense (on operating liabilities) identified to a particular segment
(not of a financial nature) will not be included as part of segment expense.
However, interest included in the cost of inventories (as per AS 16) is to be
considered as a segment expense (ASI-22).
Reconciliation between information about reportable segments and
information in financial statements of the enterprise is also to be provided.
Secondary segment information is also required to be disclosed. This
includes information about revenues, assets and cost of fixed assets
acquired.
When primary format is based on geographical segments, certain further
disclosures are required.
Disclosures are also required relating to intra-segment transfers and
composition of the segment.
AS disclosure is not required, if more than one business or geographical
segment is not identified (ASI-20).
Accounting Standard 18: Related Party Disclosures
fb.com//SUPERWHIZZ4U CACWACS.WORDPRESS.COM
Applicability of AS 18 has been restricted to enterprises whose debt or
equity securities are listed in any stock exchange in India or are in the
process of listing and all commercial enterprises whose turnover for the
accounting period exceeds Rs 50 crores.
The statement deals with following related party relationships: (i)
Enterprises that directly or indirectly control (through subsidiaries) or are
controlled by or are under common control with the reporting enterprise;
(ii) Associates, Joint Ventures of the reporting entity; Investing party or
venturer in respect of which reporting enterprise is an associate or a joint
venture; (iii) Individuals owning voting power giving control or significant
influence; (iv) Key management personnel and their relatives; and (v)
Enterprises over which any of the persons in (iii) or (iv) are able to exercise
significant influence. Remuneration paid to key management personnel
falls under the definition of a related party transaction (ASI-23).
Parties are considered related if one party has ability to control or exercise
significant influence over the other party in making financial and/or
operating decisions.
Following are not considered related parties: (i) Two companies merely
because of common director, (ii) Customer, supplier, franchiser, distributor
or general agent merely by virtue of economic dependence; and (iii)
Financiers, trade unions, public utilities, government departments and
bodies merely by virtue of their normal dealings with the enterprise.
Disclosure under the standard is not required in the following cases (i) If
such disclosure conflicts with duty of confidentially under statute, duty
cast by a regulator or a component authority; (ii) In consolidated financial
statements in respect of intra-group transactions; and (iii) In case of state-
controlled enterprises regarding related party relationships and
transactions with other state-controlled enterprises.
Relative (of an individual) means spouse, son, daughter, brother, sister,
father and mother who may be expected to influence, or be influenced by,
that individual in dealings with the reporting entity.
Standard also defines inter alia control, significant influence, associate,
joint venture, and key management personnel.
Where there are transactions between the related parties following
information is to be disclosed: name of the related party, nature of
relationship, nature of transaction and its volume (as an amount or
proportion), other elements of transaction if necessary for understanding,
amount or appropriate proportion outstanding pertaining to related parties,
provision for doubtful debts from related parties, amounts written off or
written back in respect of debts due from or to related parties.
Names of the related party and nature of related party relationship to be
disclosed even where there are no transactions but the control exists.
Items of similar nature may be aggregated by type of the related party. The
type of related party for the purpose of aggregation of items of a similar
nature implies related party relationships. Material transactions; i.e., more
than 10% of related party transactions are not to be clubbed in an
aggregated disclosure. The related party transactions which are not
fb.com//SUPERWHIZZ4U CACWACS.WORDPRESS.COM
entered in the normal course of the business would ordinarily be
considered material (ASI-13).
A non-executive director is not a key management person for the purpose
of this standard. Unless,
o he is in a position to exercise significant influence
by virtue of owning an interest in the voting power or,
o he is responsible and has the authority for directing and controlling
the activities of the reporting enterprise. Mere participation in the
policy decision making process will not attract AS 18. (ASI-21).
Accounting Standard 19: Leases
Applies in accounting for all leases other than leases to explore for or use
natural resources, licensing agreements for items such as motion pictures
films, video recordings plays etc. and lease for use of lands.
A lease is classified as a finance lease or an operating lease.
A finance lease is one where risks and rewards incident to the ownership
are transferred substantially; otherwise it is an operating lease.
Treatment in case of finance lease in the books of lessee:
At the inception, lease should be recognised as an asset and a liability at lower of
fair value of leased asset and the present value of minimum lease payments
(calculated on the basis of interest rate implicit in the lease or if not
determinable, at lessees incremental borrowing rate).
Lease payments should be appropriated between finance charge and the
reduction of outstanding liability so as to produce a constant periodic rate of
interest on the balance of the liability.
Depreciation policy for leased asset should be consistent with that for other
owned depreciable assets and to be calculated as per AS 6.
Disclosure should be made of assets acquired under finance lease, net carrying
amount at the balance sheet date, total minimum lease payments at the balance
sheet date and their present values for specified periods, reconciliation between
total minimum lease payments at balance sheet date and their present value,
contingent rent recognised as income, total of future minimum sub lease
payments expected to be received and general description of significant leasing
arrangements.
Treatment in case of finance lease in the books of lessor:
The lessor should recognize the asset as a receivable equal to net investment in
lease.
Finance income should be based on pattern reflecting a constant periodic return
on net investment in lease.
fb.com//SUPERWHIZZ4U CACWACS.WORDPRESS.COM
Manufacturer/dealer lessor should recognize sales as outright sales. If artificially
low interest rates quoted, profit should be calculated as if commercial rates of
interest were charged. Initial direct costs should be expensed.
Disclosure should be made of total gross investment in lease and the present
value of the minimum lease payments at specified periods, reconciliation between
total gross investment in lease and the present value of minimum lease
payments, unearned finance income, unguaranteed residual value accruing to the
lessor, accumulated provision for uncollectible minimum lease payments
receivable, contingent rent recognised, accounting policy adopted in respect of
initial direct costs, general description of significant leasing arrangements.
Treatment in case of operating lease in the books of the
lessee :
Lease payments should be recognised as an expense on straightline basis or
other systematic basis, if appropriate.
Disclosure should be made of total future minimum lease payments for the
specified periods, total future minimum sub lease payments expected to be
received, lease payments recognised in the P & L statement with separate
amount of minimum lease payments and contingent rents, sub lease payments
recognised in the P & L statement, general description of significant leasing
arrangements.
Treatment in case of operating lease in the books of the lessor:
Lessors should present an asset given on lease under fixed assets and lease
income should be recognised on a straight-line basis or other systematic basis, if
appropriate.
Costs including depreciation should be recognised as an expense.
Initial direct costs are either deferred over lease term or recognised as expenses.
Disclosure should be made of carrying amount of the leased assets, accumulated
depreciation and impairment loss, depreciation and impairment loss recognised
or reversed for the period, future minimum lease payments in aggregate and for
the specified periods, general description of the leasing arrangement and policy
for initial costs.
Sale and leaseback transactions
If the transaction of sale and lease back results in a finance lease, any excess or
deficiency of sale proceeds over the carrying amount should be amortized over
the lease term in proportion to depreciation of the leased assets.
fb.com//SUPERWHIZZ4U CACWACS.WORDPRESS.COM
If the transaction results in an operating lease and is at fair value, profit or loss
should be recognised immediately. But if the sale price is below the fair value any
profit or loss should be recognised immediately, however, the loss which is
compensated by future lease payments should be amortized in proportion to the
lease payments over the period for which asset is expected to be used. If the sales
price is above the fair value the excess over the fair value should be amortised.
In a transaction resulting in an operating lease, if the fair value is less than the
carrying amount of the asset, the difference (loss) should be recognised
immediately.
Note : Leases applies to all assets leased out after 1st April, 2001 and is
mandatory.
Accounting Standard 20: Earnings Per Share
Focus is on denominator to be adopted for earnings per share (EPS)
calculation.
In case of enterprises presenting consolidated financial statements EPS to
be calculated on the basis of consolidated information, as well as
individual financial statements.
Requirement is to present basic and diluted EPS on the face of Profit and
Loss statement with equal prominence to all periods presented.
EPS required being presented even when negative.
Basic EPS is calculated by dividing net profit or loss for the period
attributable to equity shareholders by weighted average of equity shares
outstanding during the period. Basic & Diluted EPS to be computed on the
basis of earnings excluding extraordinary items (net of tax expense).
(Limited Revision w.e.f 1-4-2004)
Earnings attributable to equity shareholders are after
the preference dividend for the period and the attributable tax.
The weighted average number of shares for all the periods presented is
adjusted for bonus issue, share split and consolidation of shares. In case of
rights issue at price lower than fair value, there is an embedded bonus
element for which adjustment is made.
For calculating diluted EPS, net profit or loss attributable to equity
shareholders and the weighted average number of shares are adjusted for
the effects of dilutive potential equity shares (i.e., assuming conversion
into equity of all dilutive potential equity).
Potential equity shares are treated as dilutive when their conversion into
equity would result in a reduction in profit per share from continuing
operations.
Effect of anti-dilutive potential equity share is ignored in calculating
diluted EPS.
In calculating diluted EPS each issue of potential equity share is
considered separately and in sequence from the most dilutive to the least
dilutive.
fb.com//SUPERWHIZZ4U CACWACS.WORDPRESS.COM
This is determined on the basis of earnings per incremental potential
equity.
If the number of equity shares or potential equity shares outstanding
increases or decreases on account of bonus, splitting or consolidation
during the year or after the balance sheet date but before the approval of
financial statement, basic and diluted EPS are recalculated for all periods
presented. The fact is also disclosed.
Amounts of earnings used as numerator for computing basic and diluted
EPS and their reconciliation with Profit and Loss statement are disclosed.
Also, the weighted average number of equity shares used in calculating the
basic EPS and diluted EPS and the reconciliation between the two EPS is
to be disclosed.
Nominal value of shares is disclosed along with EPS.
It has been clarified that if an enterprise discloses EPS for complying with
requirements of any source or otherwise, should calculate and disclose
EPS as per AS 20. Disclosure under Part IV of Schedule VI to the
Companies Act, 1956 should be in accordance with AS 20 (ASI-12).
Note: Earnings Per Share apply to the enterprise whose equity shares and
potential equity shares are listed on a recognised stock exchange. If the
enterprise is not so covered but chooses to present EPS, then it should
calculate EPS in accordance with the standard.
Accounting Standard 21: Consolidated Financial Statements
To be applied in the preparation and presentation of consolidated financial
statements (CFS) for a group of enterprises under the control of a parent.
Consolidated Financial Statements is recommendatory. However, if
consolidated financial statements are presented, these should be prepared
in accordance with the standard. For listed companies mandatory as per
listing agreement.
Control means, the ownership directly or indirectly through subsidiaries, of
more than one-half of the voting power of an enterprise or control of the
composition of the board of directors or such other governing body, to
obtain economic benefit. Subsidiary is an enterprise that is controlled by
parent.
Control of composition implies power to appoint or remove all or a majority
of directors.
When an enterprise is controlled by two enterprises definitions of control,
both the enterprises are required to consolidate the financial statements of
the first mentioned enterprise (ASI-24).
Consolidated financial statements to be presented in addition to separate
financial statements.
All subsidiaries, domestic and foreign to be consolidated except where
control is intended to be temporary; i.e., intention at the time of investing
is to dispose the relevant investment in the near future or the subsidiary
operates under severe long-term restrictions impairing transfer of funds to
the parent. Near future generally means not more than twelve months
fb.com//SUPERWHIZZ4U CACWACS.WORDPRESS.COM
from the date of acquisition of relevant investments (ASI-8). Control is to be
regarded as temporary when an enterprise holds shares as stock-in-trade
and has acquired and held with an intention to dispose them in the near
future (ASI-25).
CFS normally includes consolidated balance sheet, consolidated P & L,
notes and other statements necessary for preparing a true and fair view.
Cash flow only in case parent presents cash flow statement.
Consolidation to be done on a line by line basis by adding like items of
assets, liabilities, income and expenses which involves:
Elimination of cost to the parent of the investment in the subsidiary and the
parents portion of equity of the subsidiary at the date of investment. The
difference to be treated as goodwill/capital reserve, as the case may be.
Minority interest in the net income to be adjusted against income of the group.
Minority interest in net assets to be shown separately as a liability.
Intra-group balances and intra-group transactions and resulting unrealised profits
should be eliminated in full. Unrealised losses should also be eliminated unless
cost cannot be recovered.
The tax expense (current tax and deferred tax) of the parent and its subsidiaries
to be aggregated and it is not required to recompute the tax expense in context of
consolidated information (ASI-26).
The parents share in the post-acquisition reserves of a subsidiary is not required
to be disclosed separately in the consolidated balance sheet. (ASI-28).
Where two or more investments are made in a subsidiary, equity of the
subsidiary to be generally determined on a step by step basis.
Financial statements used in consolidation should be drawn up to the
same reporting date. If reporting dates are different, adjustments for the
effects of significant transactions/events between the two dates to be
made.
Consolidation should be prepared using same accounting policies. If the
accounting policies followed are different, the fact should be disclosed
together with proportion of such items.
In the year in which parent subsidiary relationship ceases to exist,
consolidation of P & L account to be made up to date of cessation.
Disclosure is to be of all subsidiaries giving name, country of incorporation
or residence, proportion of ownership and voting power held if different.
Also nature of relationship between parent and subsidiary if parent does
not own more than one half of voting power, effect of the acquisition and
disposal of subsidiaries on the financial position, names of the subsidiaries
whose reporting dates are different than that of the parent.
fb.com//SUPERWHIZZ4U CACWACS.WORDPRESS.COM
When the consolidated statements are presented for the first time, figures
for the previous year need not be given.
Notes forming part of the separate financial statements of the parent
enterprise and its subsidiaries which are material to represent a true and
fair view are required to be included in the notes to the consolidated
financial statements
(ASI-15).
Accounting Standard 22: Accounting for Taxes on Income
Effective date when mandatory (a) For listed companies and their
subsidiaries 1-4-2001 (b) For other companies - 1-4-2002 (c) All other
enterprises - 1-4-2003.
The differences between taxable income and accounting income to be
classified into permanent differences and timing differences.
Permanent differences are those differences between taxable income and
accounting income, which originate in one period and do not get reverse
subsequently.
Timing differences are those differences between taxable income and
accounting income for a period that originate in one period and are
capable of reversal in one or more subsequent periods.
Deferred tax should be recognised for all the timing differences, subject to
the consideration of prudence in respect of deferred tax assets (DTA).
When enterprise has carry forward tax losses, DTA to be recognised only if there is
virtual certainty supported by convincing evidence of future taxable income.
Unrecognised DTA to be reassessed at each balance sheet date. Virtual certainty
refers to the fact that there is practically no doubt regarding the determination of
availability of the future taxable income. Also, convincing evidence is required to
support the judgment of virtual certainty (ASI-9).
In respect of loss under the head Capital Gains, DTA shall be recognised
only to the extent that there is a reasonable certainty of sufficient future
taxable capital gain (ASI - 4). DTA to be recognised on the amount, which is
allowed as per the provisions of the Act; i.e., loss after considering the cost
indexation as per the Income Tax Act.
Treatment of deferred tax in case of Amalgamation
(ASI-11)
in case of amalgamation in nature of purchase, where identifiable assets /
liabilities are accounted at the fair value and the carrying amount for tax
purposes continue to be the same as that for the transferor enter price, the
difference between the values shall be treated as a permanent difference
and hence it will not give rise to any deferred tax. The consequent
difference in depreciation charge of the subsequent years shall also be
treated as a permanent difference.
The transferee company can recognise a DTA in respect of carry forward
losses of the transferor enterprise, if conditions relating to prudence as per
fb.com//SUPERWHIZZ4U CACWACS.WORDPRESS.COM
AS 22 are satisfied, though transferor enterprise would not have
recognised such deferred tax assets on account of prudence. Accounting
treatment will depend upon nature of amalgamation, which shall be as
follows :
o In case of amalgamation is in the nature of purchase and assets and
liabilities are accounted at the fair value, DTA should be recognised at the
time of amalgamation (subject to prudence).
o In case of amalgamation is in the nature of purchase and assets and
liabilities are accounted at their existing carrying value, DTA shall not be
recognised at the time of amalgamation. However, if DTA gets recognised
in the first year of amalgamation, the effect shall be through adjustment to
goodwill/ capital reserve.
o In case of amalgamation is in the nature of merger, the deferred tax assets
shall not be recognised at the time of amalgamation. However, if DTA gets
recognised in the first year of amalgamation, the effect shall be given
through revenue reserves.
o In all the above if the DTA cannot be recognised by the first annual balance
sheet following amalgamation, the corresponding effect of this recognition
to be given in the statement of profit and loss.
Tax expenses for the period, comprises of current tax and deferred tax.
Current tax [includes payment u/s 115JB of the Act
(ASI-6)] should be measured at the amount expected to be paid to
(recovered from) the taxation authorities, using the applicable tax rates.
Deferred tax assets and liabilities should be measured using the tax rates
and tax laws that have been enacted or substantively enacted by the
balance sheet date and should not be discounted to their present value.
Deferred Tax to be measured using the regular tax rates for companies that
pay tax u/s 115JB of the Act (ASI-6).
DTA should be disclosed separately after the head Investments and
deferred tax liability (DTL) should be disclosed separately after the head
Unsecured Loans
(ASI-7) in the balance sheet of the enterprise. Assets and liabilities to be
netted off only when the enterprise has a legally enforceable right to set
off.
The break-up of deferred tax assets and deferred tax liabilities into major
components of the respective balances should be disclosed in the notes to
accounts.
The nature of the evidence supporting the recognition of deferred tax
assets should be disclosed, if an enterprise has unabsorbed depreciation or
carry forward of losses under tax laws.
The deferred tax assets and liabilities in respect of timing differences
which originate during the tax holiday period and reverse during the tax
holiday period, should not be recognised to the extent deduction from the
total income of an enterprise is allowed during the tax holiday period.
However, if timing differences reverse after the tax holiday period, DTA and
fb.com//SUPERWHIZZ4U CACWACS.WORDPRESS.COM
DTL should be recognised in the year in which the timing differences
originate. Timing differences, which originate first, should be considered
for reversal first (ASI-3) and (ASI-5).
On the first occasion of applicability of this AS the enterprise should
recognise, the deferred tax balance that has accumulated prior to the
adoption of this Statement as deferred tax asset / liability with a
corresponding credit / charge to the revenue reserves.
Accounting Standard 23: Accounting for Investments in Associates in
Consolidated Financial Statements
Consolidation is applicable to all associates including foreign associates.
The statement deals with accounting of associates in the preparation and
presentation of CFS.
Associates is an enterprise in which the investor has significant influence
and which is neither a subsidiary nor a joint venture of the investor.
Significant influence (ordinarily having 20% or more of the voting power) is
termed as power to participate in the financial/operating policy decisions
but does not have control over such policies. The potential equity shares
held by the investee should not be taken into account for determining the
voting power of the investor. (ASI-18).
Investment in associates is accounted in CFS as per equity method. The
equity method is not applicable where the investment is acquired for
temporary period (AS 18), i.e. intention at the time of investing is to
dispose the relevant investment in the near future or where associates
operate under severe long-term restrictions. In these circumstances, the
investment should be recognised as per AS 13. The use of equity method to
be discontinued from the date when investor ceases to have significant
influence in an associate.
Provision for proposed dividend made by the associate in its financial
statements, should not be considered for the computation of the investors
share of the results of operations of the associate (ASI-16).
Goodwill / Capital Reserve on the acquisition of an associate should be
separately disclosed under carrying amount of investments.
Under the equity method, unrealised profit/losses resulting from the
transaction between investor and associates should be eliminated to the
extent of investors interest in the associates. However unrealised losses
should not be eliminated if cost of the assets cannot be recovered.
If associate has outstanding preference shares held outside the group,
preference dividends whether declared or not to be adjusted in arriving at
the investors share of profit or loss.
If investors share of losses of an associate equals or exceeds the carrying
amount of the investment, the investor will discontinue its share of loss
and will show its investment at nil value.
Where an associate presents consolidated financial statement, the results
and net assets of the associates CFS should be taken into account.
fb.com//SUPERWHIZZ4U CACWACS.WORDPRESS.COM
The carrying amount of investment in associates, on an individual basis,
should be reduced to recognize permanent decline in the value of
investment.
Listing and description of associates including proportion of ownership
interest and proportion of voting power should be disclosed in CFS.
The investors share of profits or losses and any extra- ordinary or prior
period items should be disclosed separately in CFS Profit and Loss A/c.
If reporting dates or accounting policies of associates are different from
that of financial statement of investor then the difference should be
reported in the CFS.
On the first occasion when investment in an associate is accounted for in
CFS, the carrying amount of investment in the associate should be
adjusted by using equity method, from the date of acquisition, with the
corresponding adjustment to the retained earnings in CFS.
Accounting Standard 24: Discontinuing Operations
The standard requires an enterprise to segregate information about
discontinuing operations from continuing one and establishes principles
for reporting information about discontinuing operations.
A Discontinuing operation is a part of an enterprise (a) which is being
disposed of or abandoned pursuant to a single co-ordinated plan; (b) it
represents separate line of business or geographical area of operations;
and (c) can be distinguished operationally and for financial reporting. All
these three conditions need to be satisfied simultaneously.
Initial Disclosure Event is the earliest occurrence of one of the following :
a. Entering into binding sale agreement for substantially all of the assets
attributable to the Discontinuing Operation.
b. Enterprises Governing body has approved a detailed, formal plan for the
discontinuance and made an announcement of the plan.
The statement does not establish any recognition and measurement
principles. It requires enterprise to follow principles established in other
Accounting Standard for the purpose of changes in assets, liabilities,
revenue, expenses etc.
An enterprise should give these information in its financial statements
beginning with the financial period in which the Initial Disclosure Event
occurs: (a) Description of discontinuing operation, (b) Segment in which it
is reported as per AS 17, (c) Date and nature of Initial Disclosure Event, (d)
Time by which the discontinuation is expected to be completed, (e) The
carrying amounts of the assets to be disposed of, (f) Revenue, expenses,
pre-tax profit / loss, income-tax in relation to the ordinary activities of
identified discounting operations.
On disposal of Assets or settlement of liabilities, disclosure is required for
gain/loss recognised on disposal/settlement and income tax expenses
thereto.
fb.com//SUPERWHIZZ4U CACWACS.WORDPRESS.COM
On entering into binding contract for sale of assets, disclosure is required
for Net Selling price after deducting expected disposal cost, the expected
timing of cash flow and the carrying amount of assets on the balance sheet
date.
For period subsequent to initial disclosure event period, description of any
significant changes in amount or timing of cash flow is required to be
disclosed.
The disclosures to continue up to the period in which the discontinuance is
completed; i.e., discontinuance plan is substantially completed or
abandoned.
In case discontinuance plan is abandoned, the disclosure is required of this
fact, reason therefore and its effect on the financial statements.
All disclosures should be separately presented for each discontinuing
operation.
Disclosure of pre-tax profit/loss from ordinary activities of the
discontinuing operation, income tax expenses related thereto, pre-tax
gain/loss recognised on the disposal / settlement to be made on the face
of profit and loss account.
Comparative information for prior periods to be re-stated to segregate
discontinuing operations.
In the Interim financial report, disclosure is required for any significant
activities or event and any significant changes in the amount or timing of
cash flows relating to disposal / settlement.
Accounting Standard 25: Interim Financial Reporting
Interim financial reports are financial statements (complete or condensed)
for on interim period that is shorter than a full financial year.
Interim financial report should include at a minimum a condensed balance
sheet, condensed profit and loss statement, cash flow and selected
explanatory notes.
They should include at least each of the heading and sub headings that
were included in the most recent annual financial statements.
Earnings per share if disclosed is to be calculated and presented as per AS
20.
Notes to include at least
o a statement on uniform accounting policies or any change therein.
o explanatory comments about the seasonality of interim operations.
o any unusual items (as per AS 5)
o changes in estimates of amounts reported in prior interim
periods/year, if material.
o issuances, buy-backs repayments and restructuring of debt, equity
and potential equity shares.
o dividends.
o segment reporting if required.
o any changes in composition of the enterprise.
o material changes in contingent liabilities.
fb.com//SUPERWHIZZ4U CACWACS.WORDPRESS.COM
Interim reports to include
o Balance sheet as of the end of current interim period and a
comparative balance sheet as of the end of the preceding financial
year.
o Statements of Profit & Loss for current interim period and
cumulative for current financial year to date and comparative
statements of the previous year (current and year to date)
o Cash flow statement cumulatively for the current financial year to
date with a comparative statement of previous year (year to date)
Interim measurements may rely on estimates.
For final interim period separate report not necessary as annual
statements are presented.
Uniform accounting policies to be applied in interim and annual financial
statements.
Seasonal/occasional revenues and uneven costs to be anticipated or
deferred only if appropriate to do so at the end of the financial year.
Estimates to be measured in such a way that resulting information is
reliable and all material information disclosed.
In case of change of accounting policies, other than one for which
transition is specified by an accounting standard, figures of prior interim
periods of current financial year to be restated.
Note: The presentation and disclosure requirements contained in AS 25 are not
required to be applied in respect of 'Interim financial results' example, the one
presented under Clause 41 of the Listing Agreement, since they do not meet the
definition of 'interim financial report'. However, the recognition and measurement
principles as per AS 25 should be applied.
(ASI-27)
Accounting Standard 26: Intangible Assets
Not applicable to intangibles covered by other AS, financial assets, mineral
rights/expenditure on exploration, etc. arising in insurance enterprises
from contracts with policy holders and also to expenditure in respect of
termination benefits.
An intangible asset is an identifiable non-monetary asset, without physical
substance, held for use in the production or supply of goods or services, for
rental to others, or for administrative purposes. An asset is a resource:
o controlled by an enterprise as a result of past events; and
o from which future economic benefits are expected to flow to the
enterprise.
Useful life is period of time over which an asset is expected to be used or
the number of production units expected to be obtained from the asset.
Impairment loss is the amount by which the carrying amount exceeds its
recoverable amount.
An intangible asset to be recognised only if future economic benefits will
flow and the cost of the asset can be measured reliably.
fb.com//SUPERWHIZZ4U CACWACS.WORDPRESS.COM
Probability of future economic benefits to be assessed using reasonable
and supportable assumptions.
An intangible asset should be measured initially at cost.
Internally generated goodwill, brands, mastheads, publishing titles etc.
should not be recognised as an asset.
No intangible asset arising from research to be recognised and expenditure
on research should be recognised as an expense, when incurred.
An intangible asset arising from development to be recognised, if an
enterprise can demonstrate its feasibility to complete, intention and ability
to use or sell, generation of future economic benefits, and availability of
resources for completion and ability to measure the expenditure.
Expenditure on an intangible item that cannot be treated as an asset,
should be recognised as an expense and treated as goodwill (capital
reserve), in case of an amalgamation (AS 14).
Treatment of expenditure (other than expenditure on VRS) incurred on
intangible items, which do not meet the criteria of an 'intangible asset':
o If incurred after the date of AS 26 becoming mandatory to be
expensed out when incurred;
The balances of expenditure incurred before the date of AS
26 becoming mandatory and appearing in the balance sheet,
should continue to be expensed out over a number of years
as originally contemplated;
If such balances have been adjusted against the opening
balances of revenue reserves as on 1-4-2003, it should be
rectified and treated on the above lines.
Expenditure, on an intangible item recognised as an expense should not
form part of cost of an intangible asset at a later date.
Subsequent expenditure to be added to cost only if is probable that the
expenditure will generate future benefits in excess of the original
estimates.
An intangible asset should be carried at its cost less any accumulated
amortisation and any accumulated impairment loses.
An intangible asset should be amortised over its useful life on a systematic
basis, to reflect the pattern in which the economic benefits are consumed
or if the pattern cannot be determined reliably, on the straightline method.
There is a rebuttable presumption for useful life of an intangible asset
not exceeding ten years from the date it is available for use. In case of
intangible assets in form of legal rights, the useful life is not to exceed the
period of the legal rights, unless renewable, which is virtually certain.
Residual value to be taken as zero unless a commitment to purchase the
asset or an active market exists.
The amortisation period and method to be reviewed at each financial year
end and any change to be accounted for as per
AS 5.
Any impairment losses to be recognised.
fb.com//SUPERWHIZZ4U CACWACS.WORDPRESS.COM
The recoverable amount of each intangible asset to be estimated at each
year end in case of an intangible asset which is not yet available for use
and one which is amortised over a period exceeding ten years.
An intangible asset to be derecognised on disposal or when no future
economic benefits are expected from its use and gain or loss recognised.
Disclosure for each class of intangibles, their useful lives, amortisation
rate, amount and method, carrying amount (gross and net), any additions,
retirements, impairment losses recognised or reversed and any other
change.
In case of useful life of an intangible asset exceeding ten years, proper
disclosure of the reasons for the same should be given.
Research and Development expenditure recognised as expense to be
disclosed.
On standard being applicable, adjustment to any intangible asset as
required to be made with a corresponding adjustment to the opening
revenue reserves.
Accounting Standard 27: Financial Reporting of Interests in Joint Ventures
A joint venture is a contractual arrangement whereby two or more parties
undertake an economic activity, which is subject to joint control.
In cases, wherein an enterprise by a contractual arrangement establishes
joint control over an entity which is a subsidiary (as per AS 21) the entity is
to be consolidated under AS 21 and is not to be treated as a joint venture
as per this Statement. The other venturer(s) may treat the same as a joint
venture. (Limited Revision to AS 27 w.e.f 1-4-2004)
Joint control is the contractually agreed sharing of control over an
economic activity.
For evaluating joint control, one need to consider whether the contractual
arrangement provides protective rights or participating rights to the
enterprise. The existence of participating rights would be evidence of joint
control. With effect from 1-4-2004 this explanations is removed by Limited
Revision to the Standard.
Control is the power to govern the financial and operating policies of an
economic activity so as to obtain benefits from it.
A venturer is a party to a joint venture and has joint control over that joint
venture.
An investor in a joint venture is a party to a joint venture and does not have
joint control over that joint venture.
Proportionate consolidation is a method of accounting and reporting
whereby a venturers share of each of the assets, liabilities, income and
expenses of a jointly controlled entity is reported as separate line items in
the venturers financial statements. The venturer's share in the post
acquisition reserves of the jointly controlled entity should be shown
separately under the relevant reserves in the consolidated financial
statements (ASI 28).
fb.com//SUPERWHIZZ4U CACWACS.WORDPRESS.COM
Venturer to recognise in individual and consolidated financial statements
its share of assets, liabilities, incomes and expenses in the jointly
controlled operations and also in jointly controlled assets.
In venturers separate financial statements any interest in a jointly
controlled entity to be accounted as an investment and AS 13 to be
followed.
In a venturers consolidated financial statements interest in jointly
controlled entity to be reported using proportionate consolidation except
o when interest is acquired and held with a view of disposal in near
future to be considered as not more than 12 months from
acquisition of relevant investments unless a longer period can be
justified on the basis of facts and circumstances (ASI 8)
o when severe long-term restrictions that impair the ability to transfer
funds to the venturer exists.
In such cases interest to be accounted as investments as per AS 13.
The venturers share in the post acquisition reserves of the jointly controlled entity
should be shown separately under the relevant reserves in the consolidated
financial statements (ASI-28).
A venturer to discontinue use of proportionate consolidation from the date
o it ceases to have joint control (may retain interest)
o use of proportionate consolidation is no longer appropriate.
In such cases AS 21 to be followed if venturer becomes parent and in other
cases AS 13 and/or AS 23 to be followed.
Cost in such cases is the venturers share in net assets on date of
discontinuance of proportionate consolidation as adjusted by any
goodwill/capital reserve recognised at the time of acquisition.
In case of sale of assets by a venturer to the joint venture the venturer
should recognise only that portion of gain or loss as attributable to the
interests of the other venturers. Full loss to be booked in case of evidence
of reduction in the net realisable value of current assets or on impairment
loss.
In case of purchase of assets by a venturer from a joint venture, the
venturer should recognise its share of profit only on a resale of the asset to
an independent party. Loss to be booked in case of reduction in net
realisable value of current asset or impairment loss.
In case of transactions between venturer and joint venture the above
principles to be followed only in consolidated financial statements.
Investor to follow AS 13, AS 21 and AS 23 as appropriate, for investments
in joint ventures.
Operators/Managers of joint ventures to account for fees as per AS 9.
A venturer to disclose separately, in respect of the joint venture,
contingent liabilities and capital commitments.
fb.com//SUPERWHIZZ4U CACWACS.WORDPRESS.COM
A venturer to disclose list of joint ventures and interests in significant joint
ventures.
A venturer to disclose aggregate amounts of each of the assets, liabilities,
income and expenses related to its interests in the jointly controlled
entities.
Accounting Standard 28 : Impairment of Assets
Applied in accounting for the impairment of all assets, other than:
o inventories (AS 2);
o assets arising from construction contracts (AS 7);
o financial assets, including investments (AS 13); and
o deferred tax assets (AS 22).
Recoverable amount is the higher of an assets net selling price and its
value in use.
Value in use is the present value of estimated future cash flows expected
to arise from the continuing use of an asset and from its disposal at the
end of its useful life.
An impairment loss is the amount by which the carrying amount of an asset
exceeds its recoverable amount.
Useful life is either:
o the period of time over which an asset is expected to be used ; or
o the number of production or similar units expected to be obtained
from the asset.
A cash generating unit is the smallest identifiable group of assets that
generates cash inflows largely independent of the cash inflows from other
assets.
Corporate assets are assets other than goodwill that contribute to the
future cash flows of both the cash generating unit under review and other
cash generating units.
An active market is a market where:
o the items traded are homogeneous;
o willing buyers and sellers can normally be found at any time; and
o prices are available to the public.
o To assess at each balance sheet date any indication, external or
internal as given in AS, that an asset may be impaired and estimate
the recoverable amount of the asset.
In measuring value in use:
o cash flow projections should be based on assumptions that
represent managements best estimate of the set of economic
conditions that will exist over the remaining useful life of the asset.
Greater weight should be given to external evidence;
o cash flow projections should be based on the most recent financial
budgets/forecasts (maximum 5 years, unless longer period justified)
that have been approved by management.
o cash flow projections beyond the period covered by the most recent
budgets/forecasts should be estimated by extrapolating the
fb.com//SUPERWHIZZ4U CACWACS.WORDPRESS.COM
projections based on the budgets/forecasts using a steady or
declining growth rate for subsequent years, unless an increasing
rate can be justified. This growth rate should not exceed the long-
term average growth rate for the products, industries, or country or
countries in which the enterprise operates, or for the market in
which the asset is used, unless a higher rate can be justified.
Estimates of future cash flows should include:
o projections of cash inflows from the continuing use of the asset;
o projections of cash outflows that are necessarily incurred to
generate the cash inflows from continuing use of the asset
(including cash outflows to prepare the asset for use) and that can
be directly attributed, or allocated on a reasonable and consistent
basis, to the asset; and
o net cash flows, if any, to be received (or paid) for the disposal of the
asset at the end of its useful life.
Future cash flows should be estimated for the asset in its current
condition. They should not include estimated future cash inflows or
outflows that are expected to arise from:
o a future restructuring to which an enterprise is not yet committed;
or
o future capital expenditure that will improve or enhance the asset in
excess of its originally assessed standard of performance.
Estimates of future cash flows should not include:
o cash inflows or outflows from financing activities; or
o income tax receipts or payments.
The estimate of net cash flows to be received (or paid) for the disposal of
an asset at the end of its useful life should be the amount that is expected
to be obtained from the disposal of the asset in an arms length transaction
between knowledgeable, willing parties, after deducting the estimated
costs of disposal.
The discount rate should be a pre tax rate that reflect current market
assessments of the time value of money and the risks specific to the asset
and should not reflect risks for which future cash flow estimates have been
adjusted.
Impairment loss is the reduction in carrying amount of the assets to its
recoverable amount.
An impairment loss should be recognised as an expense in the profit and
loss account immediately. Impairment loss of a revalued asset should be
treated as a revaluation decrease as per AS 10.
If the estimated impairment loss is greater than the carrying amount of the
asset, recognise a liability if, and only if, required by another AS.
The depreciation/amortisation charge for the asset should be adjusted in
future periods to allocate the assets revised carrying amount, less its
residual value on a systematic basis over its remaining useful life.
In case of any indication of impairment, the recoverable amount should be
estimated for the individual asset. If it is not possible, determine the
fb.com//SUPERWHIZZ4U CACWACS.WORDPRESS.COM
recoverable amount of the cash-generating unit to which the asset
belongs.
If an active market exists for the output produced by an asset or a group of
assets, the same should be identified as a separate cash-generating unit,
even if some or all of the output is used internally. In such case
managements
best estimate for future market price of output should be used:
o in determining the value in use of this cash-generating unit, when
estimating the future cash inflows that relate to the internal use of
the output; and
o in determining the value in use of other cash-generating units of the
reporting enterprise, when estimating the future cash outflows that
relate to the internal use of the output.
Cash-generating units should be identified consistently from period to
period for the same asset or types of assets, unless a change is justified.
The carrying amount of a cash-generating unit should be determined
consistently with the way the recoverable amount of the cash-generating
unit is determined
In testing a cash-generating unit for impairment, identify whether goodwill
that relates to this unit is recognised in the financial statements. If this is
the case, an enterprise should:
o perform a bottom-up test.
o if, in the bottom-up test, the carrying amount of goodwill could not
be allocated on a reasonable and consistent basis to the cash-
generating unit under review, the enterprise should also perform a
top-down test.
In testing a cash-generating unit for impairment, identify all the corporate
assets that relate to the cash-generating unit under review. For each
identified corporate asset, apply bottom-up test or bottom-up and top-
down test both as required.
Impairment loss should be recognised for a cash-generating unit if, and
only if, its recoverable amount is less than its carrying amount. The
impairment loss should be allocated to reduce the carrying amount of the
assets of the unit in the following order:
o first, to goodwill allocated to the cash-generating unit (if any); and
o then, to the other assets of the unit on a pro rata basis based on the
carrying amount of each asset in the unit.
These reductions in carrying amounts should be treated as impairment
losses on individual assets and recognised either in P & L account or as
revaluation decrease as applicable.
In allocating an impairment loss, the carrying amount of an asset should
not be reduced below the highest of:
o its net selling price (if determinable);
o its value in use (if determinable); and
o zero.
fb.com//SUPERWHIZZ4U CACWACS.WORDPRESS.COM
The amount of the impairment loss that would otherwise have been
allocated to the asset should be allocated to the other assets of the unit on
a pro rata basis.
A liability should be recognised for any remaining amount of an impairment
loss for a cash-generating unit if, required by another AS.
At each balance sheet date, if there are indications internal or external,
that an impairment loss recognised for an asset in prior accounting
periods, no longer exists/has decreased, then the recoverable amount of
that asset to be estimated. For the same consider the following as
minimum indications:
An impairment loss recognised for an asset in prior accounting periods
should be reversed if there is a change in the estimates of cash inflows,
cash outflows or discount rates used to determine the assets recoverable
amount since the last impairment loss was recognised. The carrying
amount of the asset should be increased to its recoverable amount.
The increased carrying amount of an asset due to a reversal of an
impairment loss should not exceed the carrying amount that would have
been determined (net of amortisation or depreciation) had no impairment
loss been recognised for the asset in prior accounting periods.
A reversal of an impairment loss for an asset should be recognised as
income immediately in profit and loss account. In case of revalued assets,
the same should be treated as a revaluation increase as per AS 10.
After a reversal of an impairment loss, the depreciation (amortisation)
charge for the asset should be adjusted in future periods to allocate the
assets revised carrying amount, less its residual value (if any), on a
systematic basis over its remaining useful life.
A reversal of an impairment loss for a cash-generating unit should be
allocated to increase the carrying amount of the assets of the unit in the
following order:
o first, assets other than goodwill on a pro rata basis based on the
carrying amount of each asset in the unit; and
o then, to goodwill allocated to the cash-generating unit, if the
requirements of reversal of impairment loss of goodwill are met.
These increases in carrying amounts should be treated as reversals of
impairment losses for individual assets and recognised accordingly.
In allocating a reversal of an impairment loss for a cash-generating unit,
the carrying amount of an asset should not be increased above the lower
of:
o its recoverable amount (if determinable); and
o the carrying amount that would have been determined (net of
amortisation or depreciation) had no impairment loss been
recognised for the asset in prior accounting periods.
The amount of the reversal of the impairment loss that would otherwise
have been allocated to the asset should be allocated to the other assets of
the unit on a pro-rata basis.
An impairment loss recognised for goodwill should not be reversed in a
subsequent period unless:
fb.com//SUPERWHIZZ4U CACWACS.WORDPRESS.COM
o the impairment loss was caused by a specific external event of an
exceptional nature that is not expected to recur; and
o subsequent external events have occurred that reverse the effect of
that event.
For each class of assets, the financial statements should disclose:
o the amount of impairment losses recognised in the statement of
profit and loss during the period and the line item(s) of the
statement of profit and loss in which those impairment losses arae
included;
o the amount of reversals of impairment losses recognised in the
statement of profit and loss during the period and the line item(s) of
the statement of
profit and loss in which those impairment losses are reversed;
o the amount of impairment losses recognised
directly against revaluation surplus during the period; and
o the amount of reversals of impairment losses recognised directly in
revaluation surplus during the period.
An enterprise that applies AS 17, should disclose the following for each
reportable segment based on an enterprises primary format (as defined in
AS 17):
o the amount of impairment losses recognised in the statement of
profit and loss and directly against revaluation surplus during the
period; and
o the amount of reversals of impairment losses recognised in the
statement of profit and loss and directly in revaluation surplus
during the period.
o If an impairment loss for an individual asset or a cash-generating
unit is recognised or reversed during the period and is material to
the financial statements of the reporting enterprise as a whole, an
enterprise should disclose;
o the events and circumstances that led to the recognition or reversal
of the impairment loss;
the amount of the impairment loss recognised or reversed;
o for an individual asset:
the nature of the asset; and
the reportable segment to which the asset belongs, based on the
enterprises primary format (as per AS 17);
o for a cash-generating unit:
a description of the cash-generating unit;
the amount of the impairment loss recognised or reversed by class
of assets and by reportable segment based on the enterprises
primary format (as defined in AS 17); and
if the aggregation of assets for identifying the cash-generating unit
has changed since the previous estimate of the cash-generating
fb.com//SUPERWHIZZ4U CACWACS.WORDPRESS.COM
units recoverable amount (if any), the enterprise should describe
the current and former way of aggregating assets and the reasons
for changing the way the cash-generating unit is identified;
whether the recoverable amount of the asset
(cash-generating unit) is its net selling price or its value in use;
o if recoverable amount is net selling price, the basis used to
determine net selling price; and
o if recoverable amount is value in use, the discount rate used in the
current estimate and previous estimate (if any) of value in use.
If impairment losses recognised (reversed) during the period are material in
aggregate to the financial statements of the reporting enterprise as a
whole, an enterprise should disclose a brief description of the following:
o the main classes of assets affected by impairment losses (reversals
of impairment losses) for which no information is disclosed; and
o the main events and circumstances that led to the recognition
(reversal) of these impairment losses for which no information is
disclosed.
As a transitional provision any impairment loss determined before this
standard becomes mandatory should be adjusted against the opening
balance of revenue reserve. Impairment losses on revalued assets to be
adjusted against balance in revaluation reserve and excess, if any against
the opening balance of revenue reserve.
Accounting Standard 29 : Provisions, Contingent Liabilities and Contingent Assets
This statement should be applied in accounting for provisions and
contingent liabilities and in dealing with contingent assets, other than
those resulting from financial instruments that are carried at fair value,
those resulting from executory contracts, those arising in insurance
enterprises from contracts with policy holders and those covered by
another Accounting Standard.
Provision is a liability, which can be measured only by using a substantial
degree of estimation.
Liability is a present obligation arising from past events, the settlement of
which is expected to result in an outflow of resources embodying economic
benefits.
Contingent Liability is -
o a possible obligation that arises from past events and the existence
of which will be confirmed only by the occurrence or non-occurrence
of one or more uncertain future events not wholly within the control
of the enterprise; or
o a present obligation, but is not recognised because it is not
probable that outflow of resources embodying economic benefits
will be required (or is remote) for its settlement or a reliable
estimate of the amount of the obligation cannot be made.
fb.com//SUPERWHIZZ4U CACWACS.WORDPRESS.COM
Contingent asset is a possible asset that arises from past events, the
existence of which will be confirmed only by the occurrences or non-
occurrence of one or more uncertain future events not wholly within the
control of the enterprise.
A provision should be recognised when
o an enterprises has a present obligation as a result of a past event;
o it is probable (more likely than not) that an outflow of resources will
be required to settle the obligation; and
o a reliable estimate can be made of the amount of the obligation.
A contingent liability is not recognised in financial statements but is
disclosed.
A contingent asset is not recognised in financial statements.
The amount of provision should be measure before tax at the best estimate
of the expenditure required to settle the present obligation and should not
be discounted to its present value.
The risks and uncertainties that inevitably surround many events and
circumstances should be taken into account in arriving at the best
estimate of provision to avoid its under or over statement.
Expected future events, which are likely to affect the amount required to
settle an obligation, may be important in measuring provisions.
Gains on the expected disposal of assets should not be taken into account
in measuring a provision, even if the expected disposal is closely linked
with the item requiring provision.
Whenever all or part of the expenditure relevant to a provision is expected
to be reimbursed by another party, the reimbursement should be
recognised only on virtual certainty of its receipt. The reimbursement
should be treated as a separate asset and should not exceed the amount of
the provision. In the statement of profit and loss, the expense relating to a
provision may be presented net of the amount recognised for a
reimbursement.
Provisions should be reviewed at each balance sheet date and adjusted to
reflect the current best estimate. The provision should be reversed, if it is
no longer probable to result in a liability.
A provision should be used only for expenditures for which the provision
was originally recognised and not against a provision recognised for
another purpose, so as not to conceal the impact of two different events.
Provision should not be recognised for future operating losses, since it is
not a liability nor meet the crieteria for provisions.
A restructuring provision should include only the direct expenditures,
necessarily entailed by the restructuring and not associated with the
ongoing activities of the enterprise.
Disclosure
o For each class of provision - the carrying amount at the beginning
and end of the period; additional provisions made, amounts used
and unused amounts reversed during the period.
o Also for each class of provision description of the nature of the
obligation, the expected timing of any resulting outflows of
fb.com//SUPERWHIZZ4U CACWACS.WORDPRESS.COM
economic benefits, the uncertainties about those outflows and the
amount of any expected reimbursement (also stating the amount of
any asset recognised therefor)
o For each class of contingent liability a brief description of its
nature and where practicable, an estimate of its financial effect, the
uncertainties relating to any outflow and the possibility of any
reimbursement. If the information is not disclosed, being not
practicable, the fact thereof is to be disclosed.
o In extremely rare cases, disclosure of any information can be
expected to prejudice seriously the position of the enterprise in a
dispute with other parties; in such cases the information need not
be disclosed but,
the fact and reason for such nondisclosure alongwith the general
nature of dispute should be disclosed.

fb.com//SUPERWHIZZ4U CACWACS.WORDPRESS.COM
sipoy satish kumar
8.38
UNIT 6: PROBLEMS BASED ON ACCOUNTING STANDARDS AND
GUIDANCE NOTES
Question 1
Events Occurring after the Balance Sheet Date and their disclosure requirements.
(5 marks) (IntermediateNov. 1994, May 97 and May 1998)
Answer
Events occurring after the balance sheet date are those significant events, both favourable and
unfavourable, that occur between the balance sheet date and the date on which the financial statements
are approved by the Board of Directors in the case of a company and in the case of any other entity by the
corresponding approving authority.
Assets and liabilities should be adjusted for events occurring after the balance sheet date that provide
additional evidence to assist the estimation of amounts relating to conditions existing at the balance sheet
date or that indicate that the fundamental accounting assumption of going concern (i.e., the continuance of
existence or substratum of the enterprise) is not appropriate. However, assets and liabilities should not be
adjusted for but disclosure should be made in the report of the approving authority of events occurring after
the balance sheet date that represent material changes and commitments affecting the financial position of
the enterprise.
(ii) Disclosure regarding events occurring after the balance sheet date :
(a) The nature of the event;
(b) An estimate of the financial effect, or a statement that such an estimate cannot be made.
Question 2
Prior-Period items. (2 marks) (IntermediateNov. 1994, May 1996 and May 1998)
Answer
When income or expenses arise in the current period as a result of errors or omissions in the preparation of
the financial statements of one or more prior periods, the said incomes or expenses have to be classified as
prior period items. The errors may occur as a result of mathematical mistakes, mistakes in applying
accounting policies, misinterpretation of facts or oversight.
Question 3
Preincorporation expenses. (5 marks) (IntermediateMay 1996)
Answer
Preincorporation expenses denote expenses incurred by the promoters for the purposes of the company
before its incorporation.

Broadly, these include expenses in connection with:
(a) preliminary analysis of the conceived idea,
(b) detailed investigation in terms of technical feasibility and commercial viability to establish the
soundness of the proposition,
(c) preparation of project report or feasibility report and its verification through independent
appraisal authority (before giving final approval to the proposition) and
(d) organisation of funds, property and managerial ability and assembling of other business
elements.
These expenses should be properly capitalised and shown in the balance sheet under the heading
Miscellaneous Expenditure. There is no legal requirement to writeoff these expenses to profit and loss


8.39
account within any specified period of time nor is there any rigid accounting convention in regard to this
matter. However, good corporate practice recognises the need to write off these expenses to profit and loss
account whtin a period of 3 to 5 years.
Question 4
Provisions contained in the Accounting Standard in respect of Revaluation of fixed assets.
(10 marks) (IntermediateNov. 1996)
Answer
(i) Revaluation of fixed Assts
According to Accounting Standard 10 on Accounting for Fixed Assets
(a) When fixed assets are revalued in financial statements, the basis of selection should be an entire
class of assets or the selection should be done on a systematic basis. The basis of selection
should be disclosed.
(b) The revaluation of any class of assets should not result in the net book value of that class being
greater than the recoverable amount of that class of assets.
(c) The accumulated depreciation should not be credited to profit and loss account.
(d) The net increase in book value should be credited to a revaluation reserve account.
(e) On disposal of a previously revalued item of fixed asset, the difference between net disposal
proceeds and the net book value should be charged or credited to the profit and loss account
except that to the extent to which such a loss is related to an increase and which has not been
subsequently reversed or utilised may be charged directly to that account.
Questiion 5
The difference between actual expense or income and the estimated expense or income as accounted for
in earlier years accounts, does not necessarily constitue the itemto be a prior period itemcomment. (2
marks) (IntermediateMay 1998)

Answer
The statement given in the question is correct and is in accordance with the Accounting Standard (AS) 5
(Revised) Net Profit or Loss for the Period. Prior Period Items and Changes in Accounting Policies.
The use of reasonable estimates is an essential part of the preparation of financial statements and does not
undermine their reliability. An estimate may have to be revised if changes occur regarding the
circumstances on which the estimate was based, or as a result of new information or subsequent
developments. The revision of the estimate, by its nature, does not bring the adjustments within the
definition of an extraordinary item or a prior period item.
Question 6
When can revenue be recognised in the case of transaction of sale of goods?
(2 marks) (IntermediateMay 1998)
Answer
As per AS 9 Revenue Recognition, revenue from sales transactions should be recognised when the
following requirements as to performance are satisfied, provided that at the time of performance it is not
unreasonable to expect ultimate collection :
(i) The seller of goods has transferred to the buyer the property in the goods for a price or all
significant risks and rewards of ownership have been transferred to the buyer and the seller
retains no effective control of the goods transferred to a degree usually associated with
ownership; and
8.40
(ii) No significant uncertainty exists regarding the amount of the consideration that will be derived
from the sale of goods.
Question 7
Valuation of fixed assets in special cases. (3 marks) (IntermediateNov. 1998)
Answer
Para 15 of Accounting Standard 10 on Accounting for Fixed Assets states the following provisions
regarding valuation of fixed assets in special cases :
1. In the case of fixed assets acquired on hire purchase terms, although legal ownership does not
vest in the enterprise, such assets are recorded at their cash value, which if not readily available,
is calculated by assuming an appropriate rate of interest. They are shown in the balance sheet
with an appropriate arration to indicate that the enterprise does not have full ownership thereof.
2. Where an enterprise owns fixed assets jointly with others (otherwise than as a partner in a firm),
the extent of its share in such assets, and the proportion in the original cost, accumulated
depreciaiton and written down value are stated in the balance sheet. Alternatively, the pro rata
cost of such jointly owned assets is grouped together with similar fully owned assets. Details of
such jointly owned assets are indicated separately in the fixed assets register.
3. Where several assets are purchased for a consolidated price, the consideration is apportioned to
the various assets on a fair basis as determined by competent valuers.
Question 8
What are the main features of the Cash Flow Statement? Explain with special reference to AS 3?
(5 marks) (IntermediateNov. 1999)
Answer
According to AS 3 (Revised) on Cash Flow Statements, cash flow statement deals with the provision of
information about the historical changes in cash and cash equivalents of an enterprise during the given
period from operating, investing and financing activities. Cash flows from operating activities can be
reported using either
(a) the direct method, whereby major classes of gross cash receipts and gross cash payments are
disclosed; or
(b) the indirect method, whereby net profit or loss is adjusted for the effects of transactions of non
cash nature, any deferrals or accruals of past or future operating cash receipts or payments, and
items of income or expense associated with investing or financing cash flows.
As per para 42 of AS 3 (Revised), an enterprise should disclose the components of cash and cash
equivalents and should present a reconciliation of the amounts in its cash flow statement with the
equivalent items reported in the balance sheet.
A cash flow statement when used in conjunction with the other financial statements, provides
information that enables users to evaluate the changes in net assets of an enterprise, its financial structure
(including its liquidity and solvency), and its ability to affect the amount and timing of cash flows in order to
adapt to changing circumstances and opportunities. This statement also enhances the comparability of the
reporting of operating performance by different enterprises because it eliminates the effects of using
different accounting treatments for the same transactions and events.
AS 3 (revised) is recommendatory at present but for companies listed on stock exchanges, its
compliance is mandatory due to the listing agreement which provides for the listed companies to furnish
cash flow statement in their Annual Reports.
Question 9
Extraordinary Items to be disclosed as per the Accounting Standard.
(3 marks) (IntermediateNov. 1994)


8.41
Answer
Extraordinary items are gains or losses which arise from events or transactions that are distinct from the
ordinary activities of the business and which are both material and expected not to recur in future
frequently. These would also include material adjustments necessitated by circumstances, which though
related to previous periods are determined in the current period. Some examples of extraordinary items
may be the sale of a signficant part of the business, the sale of an investment not acquired with the
intention of resale etc. The nature and amount of each extraordinary item are separately disclosed so that
users of financial statements can evaluate the relative significance of such items and their effect on the
current operating results. It may be noted that income or expenses arising from the ordinary activities of the
enterprise, though abnormal in amount or infrequent in occurrence, do not qualify as extraordinary.
Question 10
(i) A major fire has damaged the assets in a factory of a limited company on 2nd April-two days after the
year end closure of account. The loss is estimated at Rs. 20 crores out of which Rs. 12 crores will be
recoverable fromthe insurers. Explain briefly how the loss should be treated in the final accounts for
the previous year.
(ii) There is a sales tax demand of Rs. 2.50 crores against a company relating to prior years against
which the company has gone on appeal to the appellate authority in the department. The grounds of
appeal deal with points covering Rs. 2 crores of the demand. State how the matter will have to be
dealt with in the final accounts for the year.
(8 marks) (IntermediateMay 1995)
Answer
(i) The loss due to break out of fire is an example of event occurring after the balance sheet date that
does not relate to conditiont existing at the balance sheet date. It has not affected the financial
position as on the date of the balance sheet and therefore requires no specific adjustments in the
financial statements. However, paragraph 8.6 of AS-4 states that disclosure is generally made of
events in subsequent periods that represent unusual changes affecting the existence or substratum of
the enterprise at the balance sheet date. In the given case, the loss of assets in a factory is
considered to be an event affecting the substratum of the enterprise after the balance sheet date.
Hence, as recommended in paragraph 15 of AS-4, disclosure of the event should be made in the
report of the approving authority that represent material changes and commitments affecting the
financial position of the enterprise.
(ii) The undisputed part of sales tax liability of Rs. 0.50 crore should be considered as actual liability and
adequately provided for. The Institute of Chartered Accountants of India has issued Accounting
standard 29 on Provisions Contingent Liabilities and Contingent Assets (comes into effect in respect
of accounting periods commencing on or after 1.4.2004). According to the standard, an enterprise
should not recognise a contingent liability but should disclose it, as required by paragraph 68, unless
the possibility of an outflow of resources embodying economic benefits is remote. Accordingly the
company should disclose the disputed part of sales tax liability of Rs. 2 crore as contingent liability in
their financial statements of the year. However, the above disclosed contingent liability should be
reviewed continuosly and if it becomes probable that an outflow of future economic benefit will be
required , then recognise the contingent liability as a provision.

Question 11
J agannath Ltd. had made a rights issue of shares in 1996. In the offer document to its members, it had
projected a surplus of Rs. 40 crores during the accounting year to end on 31st March, 1998. The draft
results for the year, prepared on the hitherto followed accounting policies and presented for perusal of the
board of directors showed a deficit of Rs. 10 crores. The board in consultation with the managing director,
decided on the following :
8.42
(i) Value year-end inventory at works cost (Rs. 50 crores) instead of the hitherto method of valuation of
inventory at prime cost (Rs. 30 crores).
(ii) Provide depreciation for the year on straight line basis on account of substantial additions in gross
block during the year, instead of on the reducing balance method, which was hitherto adopted. As a
consequence, the charge for depreciation at Rs. 27 crores is lower than the amount of Rs. 45 crores
which would have been provided had the old method been followed, by Rs. 18 cores.
(iii) Not to provide for after sales expensesduring the warranty period. Till the last year, provision at 2%
of sales used to be made under the concept of matching of costs against revenue and actual
expenses used to be charged against the provision. The board now decided to account for expenses
as and when actually incurred. Sales during the year total to Rs. 600 crores.
(iv) Provide for permanent fall in the value of investments - which fall had taken place over the past five
years - the provision being Rs. 10 crores.
As chief accountant of the company, you are asked by the managing director to draft the notes on accounts
for inclusion in the annual report for 1997-1998 (6 Marks) (IntermediateMay 1998)
Answer
As per AS 1 Any change in the accounting policies which has a material effect in the current period or
which is reasonably expected to have a material effect in later periods should be disclosed. In the case of a
change in accounting policies which has a material effect in the current period, the amount by which any
item in the financial statements is affected by such change should also be disclosed to the extent
ascertainable. Where such amount is not ascertainable, wholly or in part, the fact should be indicated.
Accordingly, the notes on accounts should properly disclose the change and its effect.
Notes on Accounts :
(i) During the year inventory has been valued at factory cost, against the practice of valuing it at prime
cost as was the practice till last year. This has been done to take cognisance of the more capital
intensive method of production on account of heavy capital expenditure during the year. As a result of
this change, the year-end inventory has been valued at Rs. 50 crores and the profit for the year is
increased by Rs. 20 crores.
(ii) In view of the heavy capital intensive method of production introduced during the year, the company
has decided to change the method of providing depreciation from reducing balance method to straight
line method. As a result of this change, depreciation has been provided at Rs. 27 crores which is
lower than the charge which would have been made had the old method and the old rates been
applied, by Rs. 18 crores. To that extent, the profit for the year is increased.
(iii) So far, the company has been providing 2% of sales for meeting after sales expenses during the
warranty period. With the improved method of production, the probability of defects occurring in the
products has reduced considerably. Hence, the company has decided not to make provision for such
expenses but to account for the same as and when expenses are incurred. Due to this change, the
profit for the year is increased by Rs. 12 crores than would have been the case if the old policy were
to continue.
(iv) The company has decided to provide Rs. 10 crores for the permanent fall in the value of investments
which has taken place over the period of past five years. the provision so made has reduced the profit
disclosed in the accounts by Rs. 10 crores.
Question 12
Media Advertisers obtained advertisement rights for One Day World Cup Cricket Tournament to be held in
May/J une, 1999 for Rs. 250 lakhs.
By 31st March, 1999 they have paid Rs. 150 lakhs to secure these advertisement rights. The balance Rs.
100 lakhs was paid in April, 1999.


8.43
By 31st March, 1999 they procured advertisement for 70% of the available time for Rs. 350 lakhs. The
advertisers paid 60% of the amount by that date. The balance 40% was received in April, 1999.
Advertisements for the balance 30%time were procured in April, 1999 for Rs. 150 lakhs. The advertisers
paid the full amount while booking the advertisement.
25%of the advertisement time is expected to be available in May, 1999 and the balance 75% in J une,
1999.
You are asked to :
(i) Pass journal entries in relation to the above.
(ii) Show in columnar formas to how the items will appear in the monthly financial statements for March,
April, May and J une 1999.
Give reasons for your treatment. (12 marks) (IntermediateMay 1999)
8.44
Answer
In the books of Media Advertisers
Journal Entries
Dr. Cr.
Rs. in lakhs Rs. in lakhs
1999
March Advance for advertisement rights (purchase) A/c Dr. 150.00
To Bank A/c 150.00
(Being advance paid for obtaining advertisement
rights)
Bank A/c Dr. 210.00
To Advance for advertisement time (sale) A/c 210.00
(Being advance received from advertisers
amounting to 60% of Rs. 350 lakhs for booking
70% advertisement time)
April Advance for advertisement rights (purchase) A/c Dr. 100.00
To Bank A/c 100.00
(Being balance advance i.e., Rs. 250 lakhs less
Rs. 150 lakhs paid)
Bank A/c Dr. 140.00
To Advance for advertisement time (sale) A/c 140.00
(Being balance advance i.e., Rs. 350 lakhs less
Rs. 210 lakhs received from advertisers)
Bank A/c Dr. 150.00
To Advance for advertisement time (sale) A/c 150.00
(Being advance received from advertisers
in respect of booking of balance 30% time)
May Advertisement rights (purchase) A/c Dr. 62.50
To Advance for advertisement rights (purchase) A/c 62.50
(Being cost of advertisement rights used in May
i.e., 25% of Rs. 250 lakhs, adjusted against advance
paid)
Advance for advertisement time (sale) A/c Dr. 125.00
To Advertisement time (sale) A/c 125.00
(Being sale price of advertisement time in May i.e.,
25% of Rs. 500 lakhs adjusted, against advance
received from advertisers)
Profit and Loss A/c Dr. 62.50
To Advertisement rights (purchase) A/c 62.50


8.45
(Being cost of advertisement rights debited to Profit
and Loss Account in May)
Advertisement time (sale) A/c Dr. 125.00
To Profit and Loss A/c 125.00
(Being revenue recognised in Profit and Loss
Account in May)
June Advertisement rights (purchase) A/c Dr. 187.50
To Advance for advertisement rights (purchase) 187.50
A/c
(Being cost of advertisement rights used in June, i.e.,
75% of Rs. 250 lakhs, adjusted against
advance paid)
Advance for advertisement time (sale) A/c Dr. 375.00
To Advertisement time (sale) A/c 375.00
(Being sale price of advertisement time availed in
June i.e., 75% of Rs. 500 lakhs, adjusted against
advance received from advertisers)
June Profit and Loss A/c Dr. 187.50
To Advertisement rights (purchase) A/c 187.50
(Being cost of advertisement rights used in June,
debited to Profit and Loss Account in June)
Advertisement time (sale) A/c Dr. 375.00
To Profit and Loss Account 375.00
(Being revenue recognised in June)

(ii) Monthly financial statements
(1) Revenue statement (Rs. in lakhs)
March April May J une
Rs. Rs. Rs. Rs.
Sale of advertisement time 125.00 375.00
Less: Purchase of advertisement rights 62.50 187.50
Netprofit 62.50 187.50

(2) Balance sheet as at 31.3.99 30.4.99 31.5.99 30.6.99
Sources of funds:
Net profit 62.50 250.00
Application of funds:
Current assets, loans and advances:
Advance for advertisement rights 150.00 250.00 187.50
8.46
Bank Balance 60.00 250.00 250.00 250.00
210.00 500.00 437.50 250.00
Less: Current liabilities
Advance for advertisement time
(received from advertisers) 210.00 500.00 375.00
Net current assets 62.50 250.00
As per para 7.1 of AS 9 on Revenue Recognition, under proportionate completion method, revenue
from service transactions is recognised proportionately by reference to the performance of each act where
performance consists of the execution of more than one act. Therefore, income from advertisement is
recognised in May, 1999 (25%) and June, 1999 (75%) in the proportion of availability of the advertisement
time.
Question 13
(a) Describe the factors for determination of Reportable Segmentsas per AS-17.
(b) Briefly describe the disclosure requirements for related party transactions as per Accounting Standard
18.
(c) State the different types of Leases contemplated in Accounting Standard 19 and discuss briefly. (12
marks) (IntermediateMay 2002)
Answer
(a) Paragraphs 27 to 29 of AS 17 on Segment Reporting deals with reportable segments.
Paragraph 27 requires that a business segment or geographical segment should be identified as a
reportable segment if :
(i) its revenue from sales to external customers and from transactions with other segments is 10
percent or more of the total revenue, external and internal, of all segments; or
(ii) its segment result, whether profit or loss, is 10 percent or more of-
(a) the combined result of all segments in profit, or
(b) the combined result of all segments in loss, whichever is greater in absolute amount; or
(iii) its segment assets are 10 percent or more of the total assets of all segments.
A business segment or a geographical segment which is not a reportable segment as per paragraph
27, may be designated as a reportable segment despite its size at the discretion of the management
of the enterprise. If that segment is not designated as a reportable segment, it should be included as
an unallocated reconciling item.
If total external revenue attributable to reportable segments constitutes less than 75% of the total
enterprise revenue, additional segments should be identified as reportable segments, even if they do
not meet the 10 percent thresholds specified in paragraph 27 of the standard, until at least 75 percent
of the total enterprise revenue is included in reportable segments.
(b) Paragraph 23 of AS 18 on Related Party Disclosures requires that if there have been transactions
between related parties, during the existence of the a related party relationship, the reporting
enterprise should disclose the following :
(i) the name of the transacting related party;
(ii) a description of the relationship between the parties;
(iii) a description of the nature of transactions;
(iv) volume of the transactions either as an amount or as an appropriate proportion;


8.47
(v) any other elements of the related party transactions necessary for an understanding of the
financial statements;
(vi) the amounts or appropriate proportions of outstanding items pertaining to related parties at the
balance sheet date and provisions for doubtful debts due from such parties at that date;
(vii) amounts written off or written back in the period in respect of debts due from or to related
parties.
Point (v) requires disclosure of any other elements of the related party transactions necessary for an
understanding of the financial statements. An example of such a disclosure would be an indication
that the transfer of a major asset had taken place at an amount materially different from that
obtainable on normal commercial terms.
(c) Accounting Standard 19 has divided the lease into two types viz. (i) Finance Lease and (ii) Operating
Lease.
Finance Lease : A lease is classified as a finance lease if it transfers substantially all the risks and
rewards incident to ownership. title may or may not eventually be transferred. At the inception of a
finance lease, the lessee should recognise the lease as an asset and a liability. Such recognition
should be at an amount equal to the fair value of the leased asset at the inception of the lease.
However, if the fair value of the leased asset exceeds the present value of the minimum lease
payments from the standpoint of the lessee, the amount recorded as an asset and liability should be
the present value of the minimum lease payments from the standpoint of the lessee.
Operating Lease : A lease is classified as an operating lease if it does not transfer substantially all
the risks and rewards incident to ownership. Lease payments under an operating lease should be
recognised as an expense in the statement of profit and loss on a straight line basis over the lease
term unless another systematic basis is more representative of the time pattern of the users benefit.
Question 14
(a) When Capitalisation of borrowing cost should cease as per Accounting Standard 16?
(b) Define a "Business Segment" and a "Geographical Segment" as per Accounting Standard 17.
(c) Briefly describe, how do you calculate "Diluted Earnings per Share" as per Accounting Standard 20.
(d) Briefly describe the disclosure requirements for "Deferred Tax Assets" and "Deferred Tax Liabilities"
as per Accounting Standard 22.
(e) Write short note on Sale and Lease Back Transactions as per Accounting Standard 19.
( 20 marks) (PE-II Nov. 2002)
Answer
(a) Capitalisation of borrowing costs should cease when substantially all the activities necessary to
prepare the qualifying asset for its intended use or sale are complete.
An asset is normally ready for its intended use or sale when its physical construction or production is
complete even though routine administrative work might still continue. If minor modifications such as
the decoration of a property to the users specification, are all that are outstanding, this indicates that
substantially all the activities are complete.
When the construction of a qualifying asset is completed in parts and a completed part is capable of
being used while construction continues for the other parts, capitalisation of borrowing costs in relation
to a part should cease when substantially all the activities necessary to prepare that part for its
intended use or sale are complete.
(b) A Business Segment: A business segment is a distinguishable component of an enterprise that is
engaged in providing an individual product or service or a group of related products or services and
that is subject to risks and returns that are different from those of other business segments. Factors
that should be considered in determining whether products or services are related include:
8.48
(a) the nature of the products or services;
(b) the nature of the production processes;
(c) the type or class of customers for the products or services;
(d) the methods used to distribute the products or provide the services and
(e) if applicable, the nature of the regulatory environment, for example, banking, insurance or public
utilities.
A geographical segment: A geographical segment is a distinguishable component of an enterprise
that is engaged in providing product or services within a particular economic environment and that is
subject to risks and returns that are different from those of components operating in other economic
environments. Factors that should be considered in identifying geographical segments include:
(a) similarity of economic and political conditions;
(b) relationships between operations in different geographical areas;
(c) proximity of operations;
(d) special risks associated with operations in a particular area;
(e) exchange control regulations; and
(f) the underlying currency risks.
(c) For the purpose of calculating diluted earnings per share, the net profit or loss for the period
attributable to equity shareholders and the weighted average number of shares outstanding during the
period should be adjusted for the effects of all dilutive potential equity shares.
The amount of net profit or loss for the period attributable to equity shareholders should be adjusted,
after taking into account any attributable change in tax expense for the period.
The number of equity shares should be the aggregate of the weighted average number of equity
shares (as per paragraphs 15 and 22 of AS 20) and the weighted average number of equity shares
which would be issued on the conversion of all the dilutive potential equity shares into equity shares.
Dilutive potential equity shares should be deemed to have been converted into equity shares at the
beginning of the period or, if issued later, the date of the issue of the potential equity shares.
An enterprise should assume the exercise of dilutive options and other dilutive potential equity shares
of the enterprise. The assumed proceeds from these issues should be considered to have been
received from the issue of shares at fair value. The difference between the number of shares issuable
and the number of shares that would have been issued at fair value should be treated as an issue of
equity shares for no consideration.


(d) (i) An enterprise should offset deferred tax assets and deferred tax liabilities if:
(a) the enterprise has a legally enforceable right to set off assets against liabilities representing
current tax, and
(b) the deferred tax assets and the deferred tax liabilities relate to taxes on income levied by
the same governing taxation laws.
(ii) Deferred tax assets and liabilities should be distinguished from assets and liabilities representing
current tax for the period. Deferred tax assets and liabilities should be disclosed under a
separate heading in the balance sheet of the enterprise, separately from current assets and
current liabilities.
(iii) The break-up of deferred tax assets and deferred tax liabilities into major components of the
respective balances should be disclosed in the notes to accounts.


8.49
(iv) The nature of the evidence supporting the recognition of deferred tax assets should be disclosed,
if an enterprise has unabsorbed depreciation or carry forward of losses under tax laws.
(e) Sale and leaseback transactions: As per AS 19 on Leases, a sale and leaseback transaction
involves the sale of an asset by the vendor and the leasing of the asset back to the vendor. The lease
payments and the sale price are usually interdependent, as they are negotiated as a package. The
accounting treatment of a sale and lease back transaction depends upon the type of lease involved.
If a sale and leaseback transaction results in a finance lease, any excess or deficiency of sale
proceeds over the carrying amount should be deferred and amortised over the lease term in proportion
to the depreciation of the leased asset.
If sale and leaseback transaction results in a operating lease, and it is clear that the transaction is
established at fair value, any profit or loss should be recognised immediately. If the sale price is
below fair value any profit or loss should be recognised immediately except that, if the loss is
compensated by future lease payments at below market price, it should be deferred and amortised in
proportion to the lease payments over the period for which the asset is expected to be used. If the
sale price is above fair value, the excess over fair value should be deferred and amortised over the
period for which the asset is expected to be used.
Question 15
(a) X Co. Ltd. charged depreciation on its asset on SLM basis. For the year ended 31.3.2003 it changed
to WDV basis. The impact of the change when computed fromthe date of the asset coming to use
amounts to Rs. 20 lakhs being additional charge.
Decide how it must be disclosed in Profit and loss account. Also, discuss, when such changes in
method of depreciation can be adopted by an enterprise as per AS6.
(b) Decide when research and development cost of a project can be deferred to future periods as per AS
26.
(c) You are an accountant preparing accounts of A Ltd. as on 31.3.2003. After year end the following
events have taken place in April, 2003:
(i) A fire broke out in the premises damaging, uninsured stock worth Rs. 10 lakhs (Salvage value
Rs. 2 lakhs).
(ii) A suit against the companys advertisement was filed by a party claiming damage of Rs. 20
lakhs.
(iii) Dividend proposed @ 20% on share capital of Rs. 100 lakhs.
Describe, how above will be dealt with in the account of the company for the year ended on
31.3.2003.
(d) How the government grants related to specific fixed assets should be presented in the Balance Sheet
as per AS12?
(e) Briefly describe the disclosure requirements for amalgamation including additional disclosure, if any,
for different methods of amalgamation as per AS14.
(f) Mention the prescribed accounting treatment in respect of gratuity benefits payable to employees as
per AS15. (24 marks) (PE-II May 2003)
Answer
(a) The company should disclose the change in method of depreciation adopted for the accounting year.
The impact on depreciation charge due to change in method must be quantified and reported by the
enterprise.
Following aspects may be noted in this regard as per AS 6 on Depreciation Accounting.
(a) The depreciation method selected should be applied consistently from period to period.
8.50
(b) A change from one method of providing depreciation to another should be made only if the
adoption of the new method is required by statute or for compliance with an accounting standard
if it is considered that the change would result in a more appropriate preparation or presentation
of the financial statements of the enterprise.
(c) When such a change in the method of depreciation is made, depreciation should be recalculated
in accordance with the new method from the date of the asset coming into use. The deficiency or
surplus arising from retrospective recomputation of depreciation in accordance with the new
method should be adjusted in the accounts in the year in which the method of depreciation is
changed.
(d) In case the change in the method results in deficiency in depreciation in respect of past years, the
deficiency should be charged in the statement of profit and loss.
(e) In case the change in the method results in surplus, the surplus should be credited to the
statement of profit and loss. Such a change should be treated as a change in accounting policy
and its effect should be quantified and disclosed.
(b) As per para 41 of AS 26 Intangible Assets, no intangible asset arising from research should be
recognized. The expenditure incurred on development phase can be deferred to the subsequent
years if the company can demonstrate all of the following conditions (as specified in para 44 of AS 26
Intangible Assets):
(a) the technical feasibility of completing the intangible asset so that it will be available for use or
sale;
(b) its intention to complete the intangible asset and use or sell it;
(c) its ability to use or sell the intangible asset;
(d) how the intangible asset will generate probable future economic benefits. Among other things,
the enterprise should demonstrate the existence of a market for the output of the intangible asset
or the intangible asset itself or, if it is to be used internally, the usefulness of the intangible asset;
(e) the availability of adequate technical, financial and other resources to complete the development
and to use or sell the intangible asset; and
(f) its ability to measure the expenditure attributable to the intangible asset during its development
reliably.
(c) Events occurring after the Balance Sheet date that represent material changes and commitments
affecting the financial position of the enterprise must be disclosed according to para 15 of AS 4 on
Contingencies and Events occurring after the Balance Sheet date. Hence, fire accident and loss
thereof must be disclosed.
Suit filed against the company being a contingent liability must be disclosed with the nature of
contingency, an estimate of the financial effect and uncertainties which may affect the future outcome
must be disclosed as per para 16 of AS 4.
There are events which, although take place after the balance sheet date, are sometimes reflected in
the financial statements because of statutory requirements or because of their special nature. Such
items include the amount of dividend proposed or declared by the enterprise after the balance sheet
date in respect of the period covered by the financial statements. Thus, dividends which are proposed
or declared by the enterprise after the balance sheet date but before approval of the financial
statements, should be adjusted as per para 14 of AS 4.
(d) Paragraphs 8 and 14 of AS 12 on Accounting for Government Grants deal with presentation of
government grants related to specific fixed assets.
Government grants related to specific fixed assets should be presented in the balance sheet by
showing the grant as a deduction from the gross value of the assets concerned in arriving at their book


8.51
value. Where the grant related to a specific fixed asset equals the whole, or virtually the whole, of the
cost of the asset, the asset should be shown in the balance sheet at a nominal value. Alternatively,
government grants related to depreciable fixed assets may be treated as deferred income which
should be recognised in the profit and loss statement on a systematic and rational basis over the
useful life of the asset, i.e., such grants should be allocated to income over the periods and in
proportion in which depreciation on those assets is charged. Grants related to non-depreciable assets
should be credited to capital reserve under this method. However, if a grant related to a non-
depreciable asset requires the fulfillment of certain obligations, the grant should be credited to income
over the same period over which the cost of meeting such obligations is charged to income. The
deferred income balance should be separately disclosed in the financial statements.
(e) The disclosure requirements for amalgamations have been prescribed in paragraphs 43 to 46 of AS 14
on Accounting for Amalgamation.
For all amalgamations, the following disclosures should be made in the first financial statements
following the amalgamation:
(a) names and general nature of business of the amalgamating companies;
(b) the effective date of amalgamation for accounting purpose;
(c) the method of accounting used to reflect the amalgamation; and
(d) particulars of the scheme sanctioned under a statute.
For amalgamations accounted under the pooling of interests method, the following additional
disclosures should be made in the first financial statements following the amalgamation:
(a) description and number of shares issued, together with the percentage of each companys equity
shares exchanged to effect the amalgamation; and
(b) the amount of any difference between the consideration and the value of net identifiable assets
acquired, and the treatment thereof.
For amalgamations, accounted under the purchase method, the following additional disclosures should
be made in the first financial statements following the amalgamation;
(a) consideration for the amalgamation and a description of the consideration paid or contingently
payable; and
(b) the amount of any difference between the consideration and the value of net identifiable assets
acquired, and the treatment thereof including the period of amortisation of any goodwill arising on
amalgamation.
(f) Accounting treatment in respect of gratuity benefits payable to employees has been prescribed under
paragraph 28 of AS 15 on Accounting for Retirement Benefits in the Financial Statements of
Employers.
Accounting treatment in respect of gratuity benefit and other defined benefit schemes will depend on
the type of arrangement, which the employer has chosen to make.
(i) If the employer has chosen to make payment for retirement benefits out of his own funds, an
appropriate charge to the statement of profit and loss for the year should be made through a
provision for the accruing liability. The accruing liability should be calculated according to
actuarial valuation. However, those enterprises which employ only a few persons may calculate
the accrued liability by reference to any other rational method e.g., a method based on the
assumption that such benefits are payable to all employees at the end of the accounting year.
(ii) In case the liability for retirement benefits is funded through creation of a trust, the cost incurred
for the year should be determined actuarially. Such actuarial valuation should normally be
conducted at least once in every three years. However, where actuarial valuation are not
conducted annually, the actuarys report should specify the contributions to be made by the
employer on annual basis during the inter-valuation period. This annual contribution (which is in
8.52
addition to the contribution that may be required to finance unfunded past service cost) reflects
proper accrual of retirement benefit cost for each of the years during the inter-valuation period
and should be charged to the statement of profit or loss each year. Where the contribution paid
during a year is lower than the amount required to be contributed during the year to meet the
accrued liability as certified by the actuary, the shortfall should be charged to the statement of
profit or loss for the year. Where the contribution paid during a year is in excess of the amount
required to be contributed during the year to meet the accrued liability as certified by the actuary,
the excess should be treated as a pre-payment.
(ii) In case the liability for retirement benefits is funded through a scheme administered by an
insurer, an actuarial certificate or a confirmation from the insurer should be obtained that the
contribution payable to the insurer is the appropriate accrual of the liability for the year. Where
the contribution paid during a year is lower than the amount required to be contributed during the
year to meet the accrued liability as certified by the actuary or confirmed by the insurer, as the
case may be, the shortfall should be charged to the statement of profit or loss for the year.
Where the contribution paid during a year is in excess of the amount required to be contributed
during the year to meet the accrued liability as certified by the actuary or confirmed by the
insurer, as the case may be, the excess should be treated as a pre-payment.
Question 16
(a) How is software acquired for internal use accounted for under AS-26?
(b) What are the principles for recognition of deferred taxes under AS-22?
(c) Define related party transaction under AS-18.
(d) A Limited company charged depreciation on its assets on the basis of W.D.V. method fromthe date of
assets coming to use till date amounts to Rs. 32.23 lakhs. Now the company decides to switch over to
Straight Line method of providing for depreciation. The amount of depreciation computed on the basis
of S.L.M. fromthe date of assets coming to use till the date of change of method amounts to Rs. 20
lakhs.
Discuss as per AS-6, when such changes in method of can be adopted by the company and what
would be the accounting treatment and disclosure requirement.
(e) X Limited has recognized Rs. 10 lakhs on accrual basis income fromdividend on units of mutual funds
of the face value of Rs. 50 lakhs held by it as at the end of the financial year 31st March, 2003. The
dividends on mutual funds were declared at the rate of 20% on 15th J une, 2003. The dividend was
proposed on 10th April, 2003 by the declaring company. Whether the treatment is as per the relevant
Accounting Standard? You are asked to answer with reference to provisions of Accounting Standard.
(20 marks) (PE-II Nov. 2003)
Answer
(a) Paragraphs 10 and 11 of Appendix A to the Accounting Standard 26 on Intangible Assets, lays down
the following procedure for accounting of software acquired for internal use:-
The cost of a software acquired for internal use should be recognised as an asset if it meets the
recognition criteria prescribed in paragraphs 20 and 21 of this statement.
The cost of a software purchased for internal use comprises its purchase price, including any
import duties and other taxes (other than those subsequently recoverable by the enterprise from
the taxing authorities) and any directly attributable expenditure on making the software ready for
its use.
Any trade discounts and rebates are deducted in arriving at the cost. In the determination of cost,
matters stated in paragraphs 24 to 34 of the Statement which deal with the method of accounting for
Separate Acquisitions, Acquisitions as a part of Amalgamations, Acquisitions by way of Government
Grant, and Exchanges of Assets, need to be considered, as appropriate.


8.53
Recognition criteria as per paragraphs 20 and 21 of the standard are stated below:-
An intangible asset should be recognised if, and only if:
(a) it is probable that the future economic benefits that are attributable to the asset will flow to
the enterprise; and
(b) the cost of the asset can be measured reliably.
An enterprise should assess the probability of future economic benefits using reasonable and
supportable assumptions that represent best estimate of the set of economic conditions that will
exist over the useful life of the asset.
(b) Taxable income is calculated in accordance with tax laws. In some circumstances the requirements of
these laws to compute taxable income differ from the accounting policies applied to determine
accounting income. This results in a difference between the taxable and the accounting income.
Such differences are classified into Permanent and Timing differences. The tax effect of the timing
differences is known as Deferred Tax and is included as tax expense in the statement of profit and
loss and as deferred tax assets or as deferred tax liabilities, in the balance sheet.
Prudence would dictate that deferred tax liabilities are provided for without exception, even in
situations where an enterprise is incurring losses. Deferred tax assets should be recognized and
carried forward only to the extent that there is reasonable certainty that sufficient future taxable
income will be available against which such deferred tax asset can be realized. Reasonable certainty
can be demonstrated by providing robust and realistic estimates of profits for the future. A company
with a track record of losses with no immediate visibility of a turnaround should not recognise a
deferred tax asset as a matter of prudence. In the case of an unabsorbed depreciation and carry
forward losses under the tax laws, the recognition principles are more stricter, i.e. deferred tax asset
should be recognized only to the extent that there is virtual certainty supported by convincing evidence
that sufficient future taxable income will be available against which such deferred tax asset can be
realized. The existence of unabsorbed depreciation or carry forward of losses under tax laws is strong
evidence that future taxable income may not be available.
In that situation there has to be convincing evidence that sufficient future taxable income will be
available against which such deferred tax asset can be realized. This is a matter of judgement and the
conclusion would depend on facts and circumstances of each case.
(c) Accounting Standard 18 on Related Party Disclosures defines a related party transaction as transfer of
resources or obligations between related parties, regardless of whether or not a price is charged.
Related parties have been defined by the standard in the following words. Parties are considered to
be related if at any time during the reporting period one party has the ability to control the other party
or exercise significant influence over the other party in making financial and/or operating decisions.
Further, paragraph 24 of the Standard gives certain examples of related party transactions in respect
of which disclosures may be made by a reporting enterprise. Those examples are listed below:-
(a) purchases or sales of goods (finished or unfinished);
(b) purchases or sales of fixed assets;
(c) rendering or receiving of services;
(d) agency arrangements;
(e) leasing or hire purchase arrangements;
(f) transfer of research and development;
(g) license agreements;
(h) finance (including loans and equity contributions in cash or in kind);
(i) guarantees and collaterals; and
8.54
(j) management contracts including for deputation of employees.
(d) Paragraph 21 of Accounting Standard 6 on Depreciation Accounting says, "The depreciation method
selected should be applied consistently from period to period. A change from one method of providing
depreciation to another should be made only if the adoption of the new method is required by statute
or for compliance with an accounting standard or if it is considered that the change would result in a
more appropriate preparation or presentation of the financial statements of the enterprise."
The paragraph also mentions the procedure to be followed when such a change in the method of
depreciation is made by an enterprise. As per the said paragraph, depreciation should be recalculated
in accordance with the new method from the date of the asset coming to use. The difference in the
amount, being deficiency or surplus from retrospective recomputation should be adjusted in the profit
and loss account in the year such change is effected. Since such a change amounts to a change in
the accounting policy, it should be properly quantified and disclosed. In the question given, the
surplus arising out of retrospective recomputation of depreciation as per the straight line method is Rs.
12.23 lakhs (Rs. 32.23 lakhs Rs. 20 lakhs). This should be written back to Profit and Loss Account
and should be disclosed accordingly.
(e) Paragraph 8.4 and 13 of Accounting Standard 9 on Revenue Recognition states that dividends from
investments in shares are not recognised in the statement of profit and loss until a right to receive
payment is established.
In the given case, the dividend is proposed on 10th April, 2003, while it is declared on 15th June,
2003. Hence, the right to receive payment is established on 15th June, 2003. As per the above
mentioned paragraphs, income from dividend on units of mutual funds should be recognised by X Ltd.
in the financial year ended 31st March, 2004.
The recognition of Rs. 10 lakhs on accrual basis in the financial year 2002-2003 is not as per AS 9
'Revenue Recognition'.
(i) Acting as a banker in respect of funds of local bodies, Zilla Parishads, Panchayat Institutions etc.
who keep their funds with the treasuries.
(ii) Custody of opium and other valuables because of the strong room facility provided at the
treasury.
(iii) Custody of cash balances of the State Government and conducting cash business of Government
at non-banking treasuries.
Question 17
(a) X Ltd. received a grant of Rs. 2 crores fromthe Central Government for the purpose of a special
Machinery during 1998-99. The cost of Machinery was Rs. 20 crores and had a useful life of 9 years.
During 2002-03, the grant has become refundable due to non-fulfillment of certain conditions attached
to it. Assuming the entire grant was deducted fromthe cost of Machinery in the year of acquisition.
State with reasons, the accounting treatment to be followed in the year 2002-03.
(b) The company deals in three products, A, B and C, which are neither similar nor interchangeable. At
the time of closing of its account for the year 2002-03. The Historical Cost and Net Realizable Value
of the items of closing stock are determined as follows:
Items Historical Cost
(Rs. in lakhs)
Net Realisable
Value (Rs. in lakhs)
A 40 28
B 32 32
C 16 24
What will be the value of Closing Stock?


8.55
(c) During the current year 20022003, X Limited made the following expenditure relating to its plant
building:
Rs. in lakhs
Routine Repairs 4
Repairing 1
Partial replacement of roof tiles 0.5
Substantial improvements to the electrical wiring
systemwhich will increase efficiency

10
What amount should be capitalized?
(d) A plant was depreciated under two different methods as under:
Year SLM
(Rs. in lakhs)
W.D.V.
(Rs. in lakhs)
1 7.80 21.38
2 7.80 15.80
3 7.80 11.68
4 7.80 8.64
31.20 57.50
5 7.80 6.38
What should be the amount of resultant surplus/deficiency, if the company decides to switch over from
W.D.V. method to SLM method for first four years? Also state, how will you treat the same in
Accounts.
(e) Briefly explain the methods of accounting for amalgamation as per Accounting Standard-14.
(20 marks) (PE-II May 2004)
Answer
(a) As per para 11.3 of AS 12 on Accounting for Government Grants, the amount refundable in respect of
a government grant related to a specific fixed asset is recorded by increasing the book value of the
asset. Depreciation on the revised book value is provided prospectively over the residual useful life of
the asset. In the given case, book value of machinery will be increased by Rs. 2 crores in the year
2002-2003. The computations for the depreciation on machinery can be given as:
Cost of machinery Rs. 20 crores
Less: Grant received Rs. 2 crores
Cost of machinery Rs. 18 crores
Useful life of machinery 9 years
Depreciation per year as per straight line method Rs. 18 crores/9
(assuming residual value to be zero) = Rs. 2 crores
Total depreciation for 4 years (1998-99 to 2001-2002) Rs. 8 crores
Book value (in year 2002-2003) Rs. 10 crores
Add: Grant refunded Rs. 2 crores
Revised book value Rs. 12 crores
Remaining useful life 5 years
Revised annual depreciation Rs. 12 crores/5
8.56
= 2.4 crores
Thus, book value of machinery will be Rs. 12 crores in the year 2002-2003 and the depreciation
amounting Rs. 2.4 crores will be charged on machinery. Annual depreciation of Rs. 2.4 crores will be
charged in the next four years.
(b) As per para 5 of AS 2 on Valuation of Inventories, inventories should be valued at the lower of cost
and net realizable value. Inventories should be written down to net realizable value on an item-by-
item basis in the given case.
Items Historical Cost
(Rs. in lakhs)
Net Realisable Value
(Rs. in lakhs)
Valuation of closing
stock (Rs. in lakhs)
A 40 28 28
B 32 32 32
C 16 24 16
88 84 76
Hence, closing stock will be valued at Rs. 76 lakhs.
(c) As per para 12.1 of AS 10 on Accounting for Fixed Assets, expenditure that increases the future
benefits from the existing asset beyond its previously assessed standard of performance is included in
the gross book value, e.g., an increase in capacity. Hence, in the given case, Repairs amounting Rs.
5 lakhs and Partial replacement of roof tiles should be charged to profit and loss statement. Rs. 10
lakhs incurred for substantial improvement to the electrical writing system which will increase
efficiency should be capitalized.
(d) As per para 21 of AS 6 on Depreciation Accounting, when a change in the method of depreciation is
made, depreciation should be recalculated in accordance with the new method from the date of the
asset coming into use. The deficiency or surplus arising from retrospective recomputation of
depreciation in accordance with the new method should be adjusted in the accounts in the year in
which the method of depreciation is changed. In the given case, there is a surplus of Rs. 26.30 lakhs
on account of change in method of depreciation, which will be credited to Profit and Loss Account.
Such a change should be treated as a change in accounting policy and its effect should be quantified
and disclosed.
(e) As per AS 14 on Accounting for Amalgamations, there are two main methods of accounting for
amalgamations:
(i) The Pooling of Interest Method
Under this method, the assets, liabilities and reserves of the transferor company are recorded by the
transferee company at their existing carrying amounts (after making the necessary adjustments).
If at the time of amalgamation, the transferor and the transferee companies have conflicting
accounting policies, a uniform set of accounting policies is adopted following the amalgamation. The
effects on the financial statements of any changes in accounting policies are reported in accordance
with AS 5 on Net Profit or Loss for the Period, Prior Period Items and Changes in Accounting
Policies.
(ii) The Purchase Method
Under the purchase method, the transferee company accounts for the amalgamation either by
incorporating the assets and liabilities at their existing carrying amounts or by allocating the
consideration to individual identifiable assets and liabilities of the transferor company on the basis of
their fair values at the date of amalgamation. The identifiable assets and liabilities may include assets
and liabilities not recorded in the financial statements of the transferor company.
Where assets and liabilities are restated on the basis of their fair values, the determination of fair
values may be influenced by the intentions of the transferee company.


8.57
Question 18
(a) On 20.4.2003 J LC Ltd. obtained a loan fromthe Bank for Rs. 50 lakhs to be utilised as under:
Rs.
Construction of a shed 20 lakhs
Purchase of machinery 15 lakhs
Working capital 10 lakhs
Advance for purchase of truck 5 lakhs
In March, 2004 construction of shed was completed and machinery installed. Delivery of truck was not
received. Total interest charged by the bank for the year ending 31.3.2004 was Rs. 9 lakhs. Show the
treatment of interest under AS 16.
(b) A limited company created a provision for bad and doubtful debts at 2.5%on debtors in preparing the
financial statements for the year 2003-2004.
Subsequently on a review of the credit period allowed and financial capacity of the customers, the
company decided to increase the provision to 8%on debtors as on 31.3.2004. The accounts were not
approved by the Board of Directors till the date of decision. While applying the relevant accounting
standard can this revision be considered as an extraordinary itemor prior period item?
(c) Explain the treatment of cost arising from alteration in retirement benefit cost as per
AS 15. (12 marks) (PE-II Nov. 2004)
Answer
(a) As per AS 16, borrowing costs that are directly attributable to the acquisition, construction or
production of a qualifying asset should be capitalized. A qualifying asset is an asset that necessarily
takes a substantial period of time (usually 12 months or more) to get ready for its intended use or sale.
If an asset is ready for its intended use or sale at the time of its acquisition then it is not treated as a
qualifying asst for the purposes of AS 16.
Treatment of interest as per AS 16
Particulars Nature Interest to be capitalized Interest to be charged to profit
and loss account
(1) Construction
of a shed
Qualifying
asset
|
.
|

\
|

lakhs 50 Rs.
lakhs 20 Rs.
lakhs 9 Rs.

= Rs. 3.60 lakhs

(2) Purchase of
machinery
Not a qualifying
asset
-


|
.
|

\
|

lakhs 50 Rs.
lakhs 15 Rs.
lakhs 9 Rs. =
Rs. 2.70 lakhs.
(3) Working
capital
Not qualifying
asset

|
.
|

\
|

lakhs 50 Rs.
lakhs 10 Rs.
lakhs 9 Rs. =
Rs. 1.80 lakhs
(4) Advance for
purchase of
truck
Not a qualifying
asset

|
.
|

\
|

lakhs 50 Rs.
lakhs 5 Rs.
lakhs 9 Rs. =
Rs. 0.90 lakhs
Total
Rs.3.60 lakhs
Rs.5.40 lakhs

-
On the basis that machinery is ready for its intended use at the time of its acquisition/purchase.
8.58
(b) The preparation of financial statements involve making estimates which are based on the
circumstances existing at the time when the financial statements are prepared. It may be necessary to
revise an estimate in a subsequent period if there is a change in the circumstances on which the
estimate was based. Revision of an estimate, by its nature, does not bring the adjustment within the
definitions of a prior period item or an extraordinary item [para 21 of AS 5 (Revised) on Net Profit or
Loss for the Period, Prior Period Items and Changes in Accounting Policies].
In the given case, a limited company created 2.5% provision for doubtful debts for the year 2003-2004.
Subsequently in 2004 they revised the estimates based on the changed circumstances and wants to
create 8% provision. As per AS-5 (Revised), this change in estimate is neither a prior period item nor
an extraordinary item.
However, as per para 27 of AS 5 (Revised), a change in accounting estimate which has material effect
in the current period, should be disclosed and quantified. Any change in the accounting estimate
which is expected to have a material effect in later periods should also be disclosed.
(c) Alteration in the retirement benefit cost may arise from introduction of a retirement benefit scheme for
existing employees or because of making of improvements to an existing scheme. As per AS 15 any
alternation in retirement benefit cost arising from changes in the actuarial method used or assumptions
adopted should be charged or credited to the statement of profit or loss as they arise in accordance
with AS 5 Net Profit or Loss for the Period, Prior Period Items and Changes in Accounting Policies.
Additionally, a change in the actuarial method should be treated as a change in accounting policy and
disclosed in accordance with AS 5. The cost of additional benefits provided to retired employees due
to amendments in the retirement benefit scheme should also be treated in the same manner (i.e.
charged to profit and loss statement of the year).
Question 19
(a) A major fire has damaged assets in a factory of X Co. Ltd. on 8.4.2004, 8 days after the year end
closing of accounts. The loss is estimated to be Rs. 16 crores (after estimating the recoverable
amount of Rs. 24 crores fromthe Insurance Company).
If the company had no insurance cover, the loss due to fire would be Rs. 40 crores.
Explain, how the loss should be treated in the Final accounts of the year ended 31.3.2004.
(b) A Company had deferred research and development cost of Rs. 150 lakhs. Sales expected in the
subsequent years are as under:
Years Sales (Rs. in lakhs)
I 400
II 300
III 200
IV 100
You are asked to suggest how should Research and Development cost be charged to Profit and Loss
account.
If at the end of the III year, it is felt that no further benefit will accrue in the IV year, how the
unamortised expenditure would be dealt with in the accounts of the Company?
(c) In April, 2004 a Limited Company issued 1,20,000 equity shares of Rs. 100 each. Rs. 50 per share
was called up on that date which was paid by all shareholders. The remaining Rs. 50 was called up
on 1.9.2004. All shareholders paid the sumin September, 2004, except one shareholder having
24,000 shares. The net profit for the year ended 31.3.2005 is Rs. 2,64,000 after dividend on
preference shares and dividend distribution tax of Rs. 64,000.
Compute basic EPS for the year ended 31.3.2005 as per Accounting Standard 20.


8.59
(d) (i) Mr. Raj a relative of key Management personnel received remuneration of Rs. 2,50,000 for his
services in the company for the period from1.4.2004 to 30.6.2004. On 1.7.2004 he left the
service.
Should the relative be identified as at the closing date i.e. on 31.3.2005 for the purposes of AS
18?
(ii) X Ltd. sold goods to its associate Company for the 1st quarter ending 30.6.2004. After that, the
related party relationship ceased to exist. However, goods were supplied as was supplied to any
other ordinary customer. Decide whether transactions of the entire year has to be disclosed as
related party transaction.
(e) On 1.4.2001 ABC Ltd. received Government grant of Rs. 300 lakhs for acquisition of a machinery
costing Rs. 1,500 lakhs. The grant was credited to the cost of the asset. The life of the machinery is 5
years. The machinery is depreciated at 20% on WDV basis. The Company had to refund the grant in
May 2004 due to non-fulfillment of certain conditions.
How you would deal with the refund of grant in the books of ABC Ltd.?
(4 marks each) (PE-II May 2005
Answer
(a) The present event does not relate to conditions existing at the balance sheet date. Hence, no specific
adjustment is required in the financial statements for the year ending on 31.3.2004. But if the event
occurring after balance sheet date gives an indication that the enterprise may cease to be a going
concern, then the assets and liabilities are required to be adjusted for the financial year ended 31st
March, 2004. AS 4 (Revised) requires disclosure in respect of events occurring after the balance
sheet date representing unusual changes affecting the existence or substratum of the enterprise after
the date of the Balance Sheet. In the present event, the loss of assets in a factory can be considered
to be an event affecting the substratum of the enterprise. Hence, an appropriate disclosure should be
made in the report of the approving authority.
(b) (i) Based on sales, research and development cost to be allocated as follows:
Year Research and Development cost allocation
(Rs. in lakhs)
I
60 150
1,000
400
=
II
45 150
1,000
300
=
III
30 150
1,000
200
=
IV
15 150
1,000
100
=
(ii) If at the end of the III year, the circumstances do not justify that further benefit will accrue in IV
year, then the company has to charge the unamortised amount i.e. remaining Rs. 45 lakhs [150
(60 + 45)] as an expense immediately.
Note: As per para 41 of AS 26 on Intangible Assets, expenditure on research (or on the research
phase of an internal project) should be recognized as an expense when it is incurred. It has been
assumed in the above solution that the entire cost of Rs. 150 lakhs is development cost. Therefore,
the expenditure has been deferred to the subsequent years on the basis of presumption that the
company can demonstrate all the conditions specified in para 44 of AS 26. An intangible asset should
be derecognised when no future economic benefits are expected from its use according to para 87 of
8.60
the standard. Hence the remaining unamortised amount of Rs. 45,00,000 has been written off as an
expense at the end of third year.
(c) Basic earnings per share (EPS) =
year the during g outstandin shares equity of number average Weighted
rs shareholde equity to le attributab profit Net

= 3 Rs.
note) working in calculated (as shares 88,000
2,64,000 Rs.
=
Working Note:
Calculation of weighted average number of equity shares
Number of shares Nominal value of shares Amount paid
1st April, 2004 1,20,000 100 50
1st September, 2004 96,000 100 100
24,000 100 50
As per para 19 of AS 20 on Earnings per share, Partly paid equity shares are treated as a fraction of
equity share to the extent that they were entitled to participate in dividends relative to a fully paid
equity share during the reporting period. Assuming that the partly paid shares are entitled to
participate in the dividends to the extent of amount paid, weighted average number of shares will be
calculated as:
Shares

12
5

2
1
1,20,000 = 25,000

12
7
96,000 = 56,000

12
7

2
1
24,000 = 7,000
88,000 shares
(d) (i) According to para 10 of AS 18 on Related Party Disclosures, parties are considered to be related
if at any time during the reporting period one party has the ability to control the other party or
exercise significant influence over the other party in making financial and/or operating decisions.
Hence, Mr. Raj, a relative of key management personnel should be identified as relative as at the
closing date i.e. on 31.3.2005.
(ii) As per para 23 of AS 18, transactions of X Ltd. with its associate company for the first quarter
ending 30.06.2004 only are required to be disclosed as related party transactions. The
transactions for the period in which related party relationship did not exist need not be reported.
(e) According to para 21 of AS 12 on Accounting for Government Grants, the amount refundable in
respect of a grant related to a specific fixed asset should be recorded by increasing the book value of
the asset or by reducing the capital reserve or deferred income balance, as appropriate, by the
amount refundable. In the first alternative, i.e., where the book value is increased, depreciation on the
revised book value should be provided prospectively over the residual useful life of the asset. The
accounting treatment in both the alternatives can be given as follows:
Alternative 1:
Rs. (in lakhs)
1st April, 2001 Acquisition cost of machinery (Rs. 1,500 300) 1,200.00
31st March, 2002 Less: Depreciation @ 20% 240.00
Book value 960.00


8.61
31st March, 2003 Less: Depreciation @ 20% 192.00
Book value 768.00
31st March, 2004 Less: Depreciation @ 20% 153.60
1st April, 2004 Book value 614.40
May, 2004 Add: Refund of grant 300.00
Revised book value 914.40
Depreciation @ 20% on the revised book value amounting Rs. 914.40 lakhs is to be provided
prospectively over the residual useful life of the asset i.e. years ended 31st March, 2005 and 31st
March, 2006.
Alternative 2:
ABC Ltd. can also debit the refund amount of Rs. 300 lakhs in capital reserve of the company.
Question 20
(a) ABC Ltd. could not recover Rs. 10 lakhs froma debtor. The company is aware that the debtor is in
great financial difficulty. The accounts of the company were finalized for the year ended 31.3.2005 by
making a provision @ 20% of the amount due fromthe said debtor.
The debtor became bankrupt in April, 2005 and nothing is recoverable fromhim.
Do you advise the company to provide for the entire loss of Rs. 10 lakhs in the books of account for
the year ended 31st March, 2005?
(b) X Co. Ltd. signed an agreement with its employees union for revision of wages in J une, 2004. The
wage revision is with retrospective effect from1.4.2000. The arrear wages upto 31.3.2004 amounts to
Rs. 80 lakhs. Arrear wages for the period from1.4.2004 to 30.06.2004 (being the date of agreement)
amounts to Rs. 7 lakhs.
Decide whether a separate disclosure of arrear wages is required.
(c) An intangible asset appears in Balance Sheet of A Co. Ltd. at Rs. 16 lakhs as on 31.3.2004. The
asset was acquired for Rs. 40 lakhs in April, 1991. The Company has been amortising the asset value
on straight line basis. The policy is to amortise for 20 years.
Do you advise the Company to amortise the entire asset value in the books of the company as on
31.3.2004?
(d) RamCo. (P) Ltd. furnishes you the following information for the year ended 31.3.2005:
Depreciation for the year ended 31.3.2005
(under straight line method)
Rs. 100 lakhs
Depreciation for the year ended 31.3.2005
(under written down value method)
Rs. 200 lakhs
Excess of depreciation for the earlier years calculated under
written down value method over straight line method
Rs. 500 lakhs
The Company wants to change its method of claiming depreciation fromstraight line method to written
down value method.
Decide, how the depreciation should be disclosed in the Financial Statement for the year ended
31.3.2005.
(e) How refund of revenue grant received fromthe Government is disclosed in the Financial Statements?
(4 Marks each) (PE-II Nov. 2005)
Answer
8.62
(a) As per AS 4 Contingencies and Events occurring after the Balance Sheet Date, adjustments to assets
and liabilities are required for events occurring after the balance sheet date that provide additional
information materially affecting the determination of the amounts relating to conditions existing at the
Balance Sheet date.
In the given case, bankruptcy of the debtor in April, 2005 and consequent non-recovery of debt is an
event occurring after the balance sheet date which materially affects the determination of profits for
the year ended 31.3.2005. Therefore, the company should be advised to provide for the entire amount
of Rs. 10 lakhs according to para 8 of AS 4.
(b) It is given that revision of wages took place in June, 2004 with retrospective effect from 1.4.2000. The
arrear wages payable for the period from 1.4.2000 to 30.6.2004 cannot be taken as an error or
omission in the preparation of financial statements and hence this expenditure cannot be taken as a
prior period item.
Additional wages liability of Rs. 87 lakhs (from 1.4.2000 to 30.6.2004) should be included in current
years wages.
It may be mentioned that additional wages is an expense arising from the ordinary activities of the
company. Although abnormal in amount, such an expense does not qualify as an extraordinary item.
However, as per Para 12 of AS 5 (Revised), Net Profit or loss for the Period, Prior Period Items and
Changes in the Accounting Policies, when items of income and expense within profit or loss from
ordinary activities are of such size, nature or incidence that their disclosure is relevant to explain the
performance of the enterprise for the period, the nature and amount of such items should be disclosed
separately.
However, wages payable for the current year (from 1.4.2004 to 30.6.2004) amounting Rs. 7 lakhs is
not a prior period item, hence need not be disclosed separately. This may be shown as current year
wages.
(c) AS 26 Intangible Assets, came into effect for accounting periods commencing on or after 1.4.2003
and is mandatory in nature. Para 67 of the standard provides that if there is persuasive evidence that
the life of the intangible asset is 20 years, then no adjustment is required at 1.4.2003. However, para
63 of the standard states that if it cannot be demonstrated that the life of the intangible asset is greater
than 10 years, then AS 26 would require the asset to be amortised over not more than 10 years.
Since, in the given case, the amortisation period determined by applying para 63 has already expired
as on 1.4.2003, the carrying amount of Rs. 16 lakhs would be required to be eliminated with a
corresponding adjustment to the opening balance of revenue reserves as on 1.4.2003.
(d) As per para 21 of AS 26 Intangible Assets, when a change in the method of depreciation is made,
depreciation should be calculated in accordance with the new method from the date of the asset
coming into use. The deficiency or surplus arising from retrospective recomputation should be
adjusted in the accounts in the year in which the method of depreciation is changed. The deficiency
should be charged to profit and loss account. Similarly, any surplus should be credited in the
statement of profit and loss. Such change is a change in the accounting policy, and its effect should
be quantified and disclosed.
In the given case, the deficiency of Rs. 500 lakhs would be charged to the profit and loss account of
31.3.2005. In the notes to account, the fact of change in method of depreciation should be elaborated
along with the effect of Rs. 500 lakhs. The current depreciation charge of 200 lakhs determined in
accordance with the written down value method should be debited to the profit and loss account.
(e) The amount refundable in respect of a grant related to revenue should be applied first against any
unamortised deferred credit remaining in respect of the grant. To the extent that the amount


8.63
refundable exceeds any such deferred credit, or where no deferred credit exists, the amount should be
charged to profit and loss statement. The amount refundable in respect of a grant related to a specific
fixed asset should be recorded by increasing the book value of the asset or by reducing the capital
reserve or the deferred income balance, as appropriate, by the amount refundable. In the first
alternative, i.e., where the book value of the asset is increased, depreciation on the revised book value
should be provided prospectively over the residual useful life of the asset.
Question 21
(a) X Co. Limited purchased goods at the cost of Rs.40 lakhs in October, 2005. Till March, 2006, 75%of the
stocks were sold. The company wants to disclose closing stock at Rs.10 lakhs. The expected sale value is
Rs.11 lakhs and a commission at 10%on sale is payable to the agent. Advise, what is the correct closing
stock to be disclosed as at 31.3.2006.
(b) Explain the Accounting of Revaluation of Assets with reference to AS 10.
(c) Arjun Ltd. sold farmequipments through its dealers. One of the conditions at the time of sale is, payment of
consideration in 14 days and in the event of delay interest is chargeable @ 15%per annum. The Company
has not realized interest fromthe dealers in the past. However, for the year ended 31.3.2006, it wants to
recognise interest due on the balances due fromdealers. The amount is ascertained at Rs.9 lakhs. Decide
whether the income by way of interest fromdealers is eligible for recognition as per AS 9.
(d) AB Ltd. launched a project for producing product X in October, 2004. The Company incurred Rs.20 lakhs
towards Research and Development expenses upto 31
st
March, 2006. Due to prevailing market conditions,
the Management came to conclusion that the product cannot be manufactured and sold in the market for
the next 10 years. The Management hence wants to defer the expenditure write off to future years.
Advise the Company as per the applicable Accounting Standard.
(4 Marks each) (PE-II May 2006)
Answer
(a) As per Para 5 of AS 2 Valuation of Inventories, the inventories are to be valued at lower of cost and net
realizable value.
In this case, the cost of inventory is Rs.10 lakhs. The net realizable value is 11,00,000 90% =
Rs.9,90,000. So, the stock should be valued at Rs.9,90,000.
(b) As per Para 30 of AS 10 Accounting for Fixed Assets, an increase in net book value arising on revaluation
of fixed assets should be credited to owners interests under the head of revaluation reserve, except that,
to the extent that such increase is related to and not greater than a decrease arising on revaluation
previously recorded as a charge to the profit and loss statement, it may be credited to the profit and loss
statement. A decrease in net book value arising on revaluation of fixed assets is charged directly to profit
and loss statement except that to the extent such a decrease is related to an increase which was previously
recorded as a credit to revaluation reserve and which has not been subsequently reversed or utilized , it
may be charged directly to that account.
(c) As per AS 9 Revenue Recognition, where the ability to assess the ultimate collection with reasonable
certainty is lacking at the time of raising any claim, the revenue recognition is postponed to the extent of
uncertainty inverted. In such cases, the revenue is recognized only when it is reasonably certain that the
ultimate collection will be made.
In this case, the company never realized interest for the delayed payments make by the dealers. Hence, it
has to recognize the interest only if the ultimate collection is certain. The interest income hence is not to be
recognized.
(d) As per Para 41 of AS 26 Intangible Assets, expenditure on research should be recognized as an expense
when it is incurred. An intangible asset arising from development (or from the development phase of an
internal project) should be recognized if, and only if, an enterprise can demonstrate all of the conditions
specified in para 44 of the standard. An intangible asset (arising from development) should be
derecognised when no future economic benefits are expected from its use according to para 87 of the
8.64
standard. Therefore, the manager cannot defer the expenditure write off to future years.
Hence, the expenses amounting Rs. 20 lakhs incurred on the research and development project has to be
written off in the current year ending 31
st
March, 2006.


8.65
Question 22
(a) What are the costs that are to be included in Research and Development costs as per
AS 8.
(b) The Company reviewed an actuarial valuation for the first time for its Pension Scheme, which revalued
a surplus of Rs.12 lacs. It wants to spread the same over the next 2 years by reducing the annual
contribution to Rs.4 lacs instead of Rs.10 lacs. The average remaining life of the employees, if
estimated to be 6 years, you are required to advise the Company considering the accounting
standards 5 and 15.
(c) X Ltd. entered into an agreement to sell its immovable property included in the Balance Sheet at Rs.10
lacs to another company for Rs.15 lacs. The agreement to sell was concluded on 28
th
February, 2006
and the sale deed was registered on 1
st
May, 2006. Comment with reference to AS 4.
(d) Define related party transaction under AS 18. (4 Marks each) (PE-II- Nov. 2006)
Answer
(a) According to paras 41 and 43 of AS 26
-
, No intangible asset arising from research (or from the
research phase of an internal project) should be recognized in the research phase. Expenditure on
research (or on the research phase of an internal project) should be recognized as an expense when it
is incurred.
Examples of research costs are:
+ Costs of activities aimed at obtaining new knowledge;
+ Costs of the search for, evaluation and final selection of, applications of research findings or
other knowledge;
+ Costs of the search for alternatives for materials, devices, products, processes, systems or
services; and
+ Costs of the activities involved in formulation, design, evaluation and final selection of possible
alternatives for new or improved materials, devices, products, processes systems or services.
According to paras 45 and 46 of AS 26, In the development phase of a project, an enterprise can, in
some instances, identify an intangible asset and demonstrate that future economic benefits from the
asset are probable. This is because the development phase of a project is further advanced than the
research phase.

Examples of development activities/costs are:
+ Costs of the design, construction and testing of pre-production or pre-use prototypes and models;
+ Costs of the design of tools, jigs, moulds and dies involving new technology;
+ Costs of the design, construction ad operation of a pilot plant that is not of a scale economically
feasible for commercial production; and
+ Costs of the design, construction and testing of a chosen alternative for new or improved
materials, devices, products, processes, systems or services.
(b) According to para 92 of AS 15 (Revised 2005) on Employee Benefits, any actuarial gains and losses
should be recognized immediately in the statement of profit and loss account as income or expense.

-
AS 8 stands withdrawn w.e.f. 1st April, 2003 i.e. the date from which AS 26 Intangible Assets becomes
mandatory. Therefore the above answer has been given as per AS 26.

8.66
In the given case, the amount of surplus from pension scheme of Rs. 12 lacs is an actuarial gain,
which should be recognized as income in the profit and loss account of the current year and not to be
adjusted from the amount of annual contribution.
The surplus arising due to review of actuarial valuation of pension scheme by a company should be treated
as a change in accounting policy and disclosed in accordance with
AS 5(Revised).
(c) According to para 13 of AS 4 Contingences and Events occurring after the Balance Sheet Date,
assets and liabilities should be adjusted for events occurring after the balance sheet date that provide
additional evidence to assist the estimation of amounts relating to conditions existing at the balance
sheet date.
In this case the sale of immovable property was carried out before the closure of the books of
Accounts. This is clearly an event occurring after the balance sheet date. Agreement to sell was
effected before the balance sheet date and the registration was done after the balance sheet date. So
the adjustment for the sale of immovable property is necessary in the books of account for the year
ended 31
st
March, 2006.
(d) According to AS 18, Parties are considered to be related if at any time during the reporting period one
party has the ability to control the other party or exercise significant influence over the other party in
making financial and/or operating decisions.
A related party transaction involves a transfer of resources or obligations between related parties,
regardless of whether or not a price is charged.
Following are the examples of the related party transactions in respect of which disclosures may be
made by a reporting enterprise:
+ Purchases or sales of goods (finished or unfinished);
+ Purchases or sales of fixed assets;
+ Rendering or receiving of services;
+ Agency arrangements;
+ Leasing or hire purchase arrangements;
+ Transfer of research and development;
+ Licence agreements;
+ Finance (including loans and equity contributions in cash or in kind);
+ Guarantees and collateral etc.
+ Management contracts including for deputation of employees.
Question 23
(a) What are the disclosure requirements of AS-7 (Revised)?
(b) How would you treat the Government grant received relating to a depreciable asset under the
following cases as per AS-12?
Case i: Gross value of asset Rs.2 crores and Grant received Rs.20 lakhs only.
Case ii: Gross value of asset Rs.2 crores and Grant received Rs.2 crores.
(c) Explain the concept of actuarial valuation.
(d) What are the information that are to be disclosed in the financial statements as per
AS-10? (4x4=16 Marks) (PE II- May, 2007)


8.67
Answer
(a) According to paragraphs 38, 39 and 41 of AS 7, an enterprise should disclose:
(a) the amount of contract revenue recognized as revenue in the period;
(b) the methods used to determine the contract revenue recognized in the period; and
(c) the methods used to determine the stage of completion of contracts in progress.
In case of contract still in progress the following disclosures are required at the reporting date:
(a) the aggregate amount of costs incurred and recognised profits (less recognised losses) upto the
reporting date;
(b) the amount of advances received; and
(c) the amount of retentions.
An enterprise should also present:
(a) the gross amount due from customers for contract work as an asset; and
(b) the gross amount due to customers for contract work as a liability.
(b) In accordance with AS 12, government grants related to specific fixed assets should be presented in
the balance sheet by showing the grant as a deduction from the gross value of the assets concerned
in arriving at their book value. Where the grant related to a specific fixed asset equals the whole, or
virtually the whole, of the cost of the asset, the asset should be shown in the balance sheet at a
nominal value.
Alternatively, government grants related to depreciable fixed assets may be treated as deferred
income which should be recognized in the profit and loss statement on a systematic and rational basis
over the useful life of the asset, i.e., such grants should be allocated to income over the periods and in
the proportions in which depreciation on those assets is charged.
Case i
Grant received amounting Rs.20 lakhs is required to be deducted from Rs.2 crores. The balance of
Rs.1.80 crores to be shown as an assest in the Balance Sheet and depreciation should also be
charged on Rs.1.80 crores.
Case ii
As the grant is received for the entire cost of the asset, the asset shall be recorded at a nominal value
of Rs.100 in the Balance sheet so that the existence of the amount is reflected. No depreciation is to
be charged in this case.
Note: Alternatively, in both the cases government grant may be treated as deferred income which
should be recognized in the profit and loss statement on a systematic and rational basis over the
useful life of the asset.
(c) Actuarial valuation is the process used by an actuary
-
to estimate the present value of benefits to be
paid under a retirement scheme and the present values of the scheme assets and, sometimes, of
future contributions. In the case of defined benefit scheme the cost of retirement benefits, to be
charged to Profit and Loss Account on year to year basis, is determined on actuarial basis. According
to paragraph 65 of AS 15 (revised 2005), an enterprise should use the Projected Unit Credit
--
method

-
Actuary is an expert person who can calculate the liability where the factors affecting the calculation of liability
are uncertain and cannot be determined in ordinary course.
--
Projected Unit Credit method (sometimes known as the accrued benefit method pro-rated on service or as the benefit/years
of service method) considers each period of service as giving rise to an additional unit of benefit entitlement and measures
each unit separately to build the final obligation.
8.68
to determine the present value of its defined benefit obligations and the related current service cost
and, wherever applicable, past service cost.


8.69
(d) As per AS 10, the following information should be disclosed in the financial statements :
(i) gross and net book values of fixed assets at the beginning and end of an accounting period
showing additions, disposals, acquisitions and other movements ;
(ii) expenditure incurred on account of fixed assets in the course of construction or acquisition ; and
(iii) revalued amount substituted for historical costs of fixed assets, the method adopted to compute
the revalued amounts, the nature of indices used, the year of any appraisal made, and whether
an external valuer was involved, in case where fixed assets are stated at revalued amounts.
Question 24
(a) Explain the treatment of Refund of Government Grants as per AS-12.
(b) The Company X Ltd., has to pay for delay in cotton clearing charges. The company up to 31.3.2006
has included such charges in the valuation of closing stock. This being in the nature of interest, X Ltd.
decided to exclude such charges fromclosing stock for the year 2006-07. This would result in
decrease in profit by Rs.5 lakhs. Comment.
(c) The Board of Directors of X Ltd. decided on 31.3.2007 to increase sale price of certain items of goods
sold retrospectively from1
st
J anuary, 2007. As a result of this decision the company has to receive
Rs.5 lakhs fromits customers in respect of sales made from1.1.2007 to 31.3.2007. But the
Companys Accountant was reluctant to make-up his mind. You are asked to offer your suggestion.
(d) Briefly explain disclosure requirements for Investments as per AS-13.
(4x4 =16 Marks)(PE II-Nov. 2007)
Answer
(a) As per para 11 of AS 12 Accounting for Government Grants, government grant that becomes
refundable is treated as an extraordinary item.
The amount refundable in respect of a government grant related to revenue is first applied against any
unamortised deferred credit remaining in respect of the grant.
The amount refundable in respect of a government grant related to a specific fixed asset is recorded
by increasing the book value of the asset or by reducing the capital reserve or the deferred income
balance, as appropriate, by the amount refundable.
Where a grant which is in the nature of promoters contribution becomes refundable, in part or in full,
to the government on non-fulfillment of some specified conditions, the relevant amount recoverable by
the government is reduced from the capital reserve.
8.70
(b) As per para 12 of AS 2 (revised), interest and other borrowing costs are usually considered as not
relating to bringing the inventories to their present location and condition and are therefore, usually not
included in the cost of inventories. However, X Ltd. was in practice to charge the cost for delay in
cotton clearing in the closing stock. As X Ltd. decided to change this valuation procedure of closing
stock, this treatment will be considered as a change in accounting policy and such fact to be disclosed
as per AS 1. Therefore, any change in amount mentioned in financial statement, which will affect the
financial position of the company should be disclosed properly as per AS 1, AS 2 and AS 5.
Also a note should be given in the annual accounts that, had the company followed earlier system of
valuation of closing stock, the profit before tax would have been higher by Rs. 5 lakhs.
(c) As per para 10 of AS 9 Revenue Recognition, the additional revenue on account of increase in sales
price with retrospective effect, as decided by Board of Directors of X Ltd., of Rs.5 lakhs to be
recognised as income for financial year 2006-07, only if the company is able to assess the ultimate
collection with reasonable certainty. If at the time of raising of any claim it is unreasonable to expect
ultimate collection, revenue recognition should be postponed.
(d) The disclosure requirements as per para 35 of AS 13 are as follows:
(i) Accounting policies followed for valuation of investments.
(ii) Classification of investment into current and long term in addition to classification as per
Schedule VI of Companies Act in case of company.
(iii) The amount included in profit and loss statements for
(a) Interest, dividends and rentals for long term and current investments, disclosing therein
gross income and tax deducted at source thereon;
(b) Profits and losses on disposal of current investment and changes in carrying amount of
such investments;
(c) Profits and losses and disposal of long term investments and changes in carrying amount of
investments.
(iv) Aggregate amount of quoted and unquoted investments, giving the aggregate market value of
quoted investments;
(v) Any significant restrictions on investments like minimum holding period for sale/disposal,
utilisation of sale proceeds or non-remittance of sale proceeds of investment held outside India.
(vi) Other disclosures required by the relevant statute governing the enterprises.

Question 25
Answer any four of the following:
(i) (a) X Ltd. purchased debentures of Rs.10 lacs of Y Ltd., which are traded in stock exchange. How
will you show this itemas per AS 3 while preparing cash flow statement for the year ended on
31
st
March, 2008?
(b) Mr. Raj a relative of key management personnel received remuneration of Rs.2,50,000 for his
services in the company for the period from1.4.2007 to 30.6.2007. On 1.7.2007, he left the
service.
Should the relative be identified as a related party at the closing date i.e., on 31.3.2008 for the
purpose of AS 18?
(ii) A manufacturing company purchased shares of another company fromstock exchange on 1
st
May,
2007 at a cost of Rs.5,00,000. It also purchased Gold of Rs.2,00,000 and Silver of Rs.1,50,000 on 1
st



8.71
April, 2005. How will you treat these investments as per the applicable AS in the books of the
company for the year ended on 31
st
March, 2008, if the values of these investments are as follows:
Rs.
Shares 2,00,000
Gold 4,00,000
Silver 2,50,000
(iii) (a) Wye Ltd. received Rs.50 lacs fromthe Central Government as subsidy for setting up an industry
in backward area. How will you treat it in accounts?
(b) How Government grant relating to Specific Fixed Assets is treated in the books as per AS 12?
(iv) A Ltd. had 6,00,000 equity shares on April 1, 2007. The company earned a profit of Rs.15,00,000
during the year 2007-08. The average fair value per share during 2007-08 was Rs.25. The company
has given share option to its employees of 1,00,000 equity shares at option price of Rs.15. Calculate
basic EPS and diluted EPS.
(v) In a production process, normal waste is 5% of input. 5,000 MT of input were put in process resulting
in wastage of 300 MT. Cost per MT of input is Rs.1,000. The entire quantity of waste is on stock
at the year end. State with reference to Accounting Standard, how will you value the inventories
in this case?
(4 x 4=16 Marks)(PEII-May, 2008)
8.72
Answer
(i) (a) As per AS 3 on Cash flow Statement, cash and cash equivalents consists of cash in hand,
balance with banks and short-term, highly liquid investments
1
. If investment, of Rs.10 lacs, made
in debentures is for short-term period then it is an item of cash equivalents.
However, if investment of Rs.10 lacs made in debentures is for long-term period then as per AS
3, it should be shown as cash flow from investing activities.
(b) According to para 10 of AS 18 on Related Party Disclosures, parties are considered to be
related if at any time during the reporting period one party has the ability to control the other party
or exercise significant influence over the other party in making financial and/or operating
decisions.
Here, Mr. Raj, who received remuneration of Rs.2,50,000 from the company, is the relative of the
key management personnel of that company. And as per para 3 clause (d) of the Standard, key
management personnel and relatives of such personnel are said to be in related party
relationships. Hence, Mr. Raj, a relative of key management personnel ,should be identified as
related party at the closing date i.e. on 31.3.2008.
(ii) As per para 32 of AS 13 on Accounting for Investments, any investment of long term period is shown
at cost. Hence, the investment in Gold and Silver (purchased on 1
st
April 2005) shall continue to be
shown at cost i.e., Rs.2,00,000 and Rs.1,50,000 respectively as their value have increased.
Also as per AS 13, for investment in shares - if the investment is for short-term period then the loss of
Rs.3,00,000 is to be charged to profit & loss account for the year ended 31
st
March, 2008. If
investment is of long term period then it will continue to be shown at cost in the Balance Sheet of the
company. However, provision for diminution shall be made to recognize a decline, other than
temporary, in the value of the investments, such reduction being determined and made for each
investment individually.
(iii) (a) As per para 10 of AS 12 on Accounting for Government Grants, subsidy of Rs.50 lacs from the
Central government, for setting up an industry in backward area is a government grant in the
nature of promoters contribution. Such grants are treated as capital reserve which can be
neither distributed as dividend nor considered as deferred income.

1
As per para 6 of AS 3, an investment normally qualifies as a cash equivalent only when it has a short maturity
of, say three months or less from the date of acquisition.


8.73
(b) According to para 8 of AS 12 on Accounting for Government Grants, two methods of
presentation, in financial statements, of grants related to specific fixed assets are regarded as
acceptable alternatives.
+ Under one method, the grant is shown as a deduction from the gross value of the asset
concerned in arriving at its book value.
+ Under the other method, grant related to depreciable asset is treated as deferred income
which is recognized in the profit and loss statement on a systematic and rational basis over
the useful life of the assets. Grants related to non-depreciable assets are credited to capital
reserve under this method, as there is usually no charge to income in respect of such
assets. However, if a grant related to a non-depreciable asset requires the fulfillment of
certain obligations, the grant is credited to income over the same period over which the cost
of meeting such obligations is charged to income. The deferred income is suitably disclosed
in the balance sheet pending its apportionment to profit and loss account.
(iv) Computation of earnings per share
Earnings Shares Earnings per
share
Net profit for the year 2007-08 Rs.15,00,000
Weighted average number of shares
outstanding during year 2007-08
6,00,000
Basic earnings per share Rs. 2.50
Number of shares under option 1,00,000
Number of shares that would have
been issued at fair value:
(100,000 x 15.00)/25.00

*

(60,000)

Diluted earnings per share Rs. 15,00,000 6,40,000 Rs. 2.34
(approx.)
*The earnings have not been increased as the total number of shares has been increased
only by the number of shares (40,000) deemed for the purpose of the computation to have
been issued for no consideration.
(v) As per para 13 of AS 2 (Revised), abnormal amounts of wasted materials, labour and other
production costs are excluded from cost of inventories and such costs are recognized as
expenses in the period in which they are incurred.
In this case, normal waste is 250 MT and abnormal waste is 50 MT.
The cost of 250 MT will be included in determining the cost of inventories (finished goods) at the
year end. The cost of abnormal waste amounting to Rs.50,000
(50 MT Rs.1,000) will be charged to the profit and loss statement.

Question 26
Following is the cash flow abstract of Alpha Ltd. for the year ended 31
st
March, 2008:
Cash Flow Abstract
Inflows Rs. Outflows Rs.
Opening balance: Payment to creditors 90,000
Cash 10,000 Salaries and wages 25,000
Bank 70,000 Payment of overheads 15,000
Share capital shares issued 5,00,000 Fixed assets acquired 4,00,000
Collection fromDebtors 3,50,000 Debentures redeemed 50,000
8.74
Sale of fixed assets 70,000 Bank loan repaid 2,50,000
Taxation 55,000
Dividends 1,00,000
Closing balance:
Cash 5,000
bank 10,000
10,00,000 10,00,000
Prepare Cash Flow Statement for the year ended 31
st
March, 2008 in accordance with Accounting
standard 3.
(8 Marks) (PE II- Nov. 2008)
Answer
Cash FlowStatement
for the year ended 31.3.2008
Rs. Rs.
Cash flowfromoperating activities
Cash received from customers 3,50,000
Cash paid to suppliers (90,000)
Cash paid to employees (salaries and wages) (25,000)
Other cash payments (overheads) (15,000)
Cash generated from operations 2,20,000
Income tax paid (55,000)
Net cash from operating activities 1,65,000




Cash flowfrominvesting activities
Payment for purchase of fixed assets (4,00,000)
Proceeds from sale of fixed assets 70,000
Net cash used in investment activities (3,30,000)
Cash flowfromfinancing activities
Proceeds from issue of share capital 5,00,000
Bank loan repaid (2,50,000)
Debentures redeemed (50,000)
Dividends paid (1,00,000)
Net cash from financing activities 1,00,000
Net decrease in cash and cash equivalents (65,000)
Cash and cash equivalents at the beginning of the year 80,000
Cash and cash equivalents at the end of the year 15,000

Question 27


8.75
(a) B Ltd. undertook a construction contract for Rs. 50 crores in April, 2007. the cost of construction was
initially estimated at Rs. 35 crores. The contract is to be completed in 3 years. While executing the
contract, the company estimated the cost of completion of the contract at Rs. 53 crores.
Can the company provide for the expected loss in the book of account for the year ended 31
st
March,
2008?
(b) List any five related party transactions, which require disclosure as per AS 18.
(c) A Government grant of Rs. 25 lakhs received 3 years ago in respect of a machinery which costs Rs.
200 lakhs, became refundable in March, 2008.
(i) How the receipt of grant would have been recorded in the books of the recipient?
(ii) How the refund of grant would be reflected in the books, at the time of its refund?
(d) List the conditions to be fulfilled as per Accounting Standard 14 (AS 14) for an amalgamation to be in
the nature of merger, in the case of companies.
(e) Discuss the treatment of exchange loss relating to fixed assets as per AS 11 vis a vis the Schedule
VI disclosure under the Companies Act, 1956.
(4 x 5 =20 Marks) (PE II- Nov. 2008)
8.76
Answer
(a) As per para 35 of AS 7 Construction Contracts, when it is probable that total contract costs will
exceed total contract revenue, the expected loss should be recognised as an expense immediately.
Therefore, The foreseeable loss of Rs.3 crores (Rs. 53 crores less Rs. 50 crores) should be
recognised as an expense immediately in the year ended 31
st
march, 2008. The amount of loss is
determined irrespective of
(i) Whether or not work has commenced on the contract;
(ii) Stage of completion of contract activity; or
(iii) The amount of profits expected to arise on other contracts which are not treated as a single
construction contract in accordance with para 8 of AS 7.
(b) Five examples of related party transactions for which disclosure is required according to AS 18 are:
(i) Purchase and/or sales of goods (finished or unfinished)
(ii) Purchase or sale of fixed assets.
(iii) Rendering or receiving of services.
(iv) Agency arrangements.
(v) Leasing or hire purchase arrangements.
(c) The grant is shown as a deduction from the gross value of the asset. Depreciation on machinery
would be charged on the reduced value of Rs.175 lakhs. Alternatively, the grant may be treated
as deferred income which should be credited to profit and loss statement on a systematic and rational
basis over the useful life of the asset.
As per para 21 of AS 12, the amount refundable in respect of a grant related to a specific fixed asset
should be recorded by increasing the book value of the asset or by reducing the capital reserve or the
deferred income balance, as appropriate, by the amount refundable. In the first alternative, i.e., where
the book value of the asset is increased, depreciation on the revised book value should be provided
prospectively over the residual useful life of the asset.
(d) An amalgamation should be considered to be an amalgamation in the nature of merger if the following
conditions are satisfied:
(i) All the assets and liabilities of the transferor company become, after amalgamation, the assets
and liabilities of the transferee company.


8.77
(ii) Shareholders holding not less than 90% of the face value of the equity shares of the transferor
company (other than the equity shares already held therein, immediately before the amal-
gamation, by the transferee company or its subsidiaries or their nominees) become equity
shareholders of the transferee company by virtue of the amalgamation.
(iii) The consideration for the amalgamation receivable by those equity shareholders of the transferor
company who agree to become equity shareholders of the transferee company is discharged by
the transferee company wholly by the issue of equity shares in the transferee company, except
that cash may be paid in respect of any fractional shares.
(iv) The business of the transferor company is intended to be carried on, after the amalgamation, by
the transferee company.
(v) No adjustment is intended to be made to the book values of the assets and liabilities of the
transferor company when they are incorporated in the financial statements of the transferee
company except to ensure uniformity of accounting policies.
(e) Schedule VI to The Companies Act, 1956 provides that any increase or decrease in liability due to
change in the rate of exchange relating to any fixed asset should be added to or deducted from the
cost of the asset. The amount arrived at should be taken to be the cost of the fixed asset.
AS 11 (revised), however, does not require adjustment of exchange difference in the carrying amount
of fixed assets. The exchange difference is required to be recognised in the statement of profit or loss
since it is felt that this treatment is conceptually preferable to that required in Schedule VI and is in
consonance with the international position in this regard.
The provisions of AS 11 will prevail over Schedule VI of the Companies Act. National Advisory
Committee on Accounting Standards (NACAS) has notified AS 11 for preparation of financial
statements of companies. ICAI has come up with the announcement in this regard, stating that after
the notification of AS 11 by NACAS, AS 11 will overrule Schedule VI of the Companies Act.

13
FINANCIAL ANALYSIS
UNITS 1 &2 : FUND FLOWSTATEMENT AND CASH FLOWSTATEMENT
(A) Write short notes on:
Question 1
Cash Flow Statement. (5 marks) (IntermediateNov. 1997)
Answer
Cash flow statement is a statement of inflows and outflows of cash and cash equivalents. It starts with
the opening balance of cash and cash equivalents at the start of the accounting period. It then gives in a
summary form, the inflows and outflows relating to the following three classifications of activities :
(i) Operating activities : They are the principal revenue producing activities of the enterprise.
(ii) Investing activities : They deal with the acquisition and disposal of long-term assets and long term
investments.
(iii) Financing activities : They reflect changes in the size and composition of capital in the case of a
company this would preference capital and borrowings of the enterprise.
The cash flows arising from extraordinary items are disclosed separately under each of the above three
classifications.
Likewise where the amount of significant cash and cash equivalent balances held by an enterprise are not
available for use by the enterprise, the same should be disclosed separately together with a commentary by
the management.
Question 2
In the case of manufacturing company :
(i) List the items of inflows of cash receipts from operating activities;
(ii) List the items of outlflows of investing activities. (4 marks) [Intermediate May 1998]
13.2
Answer
(i) Inflows of cash receipts fromoperating activities :
(a) Cash receipts from the sales of goods;
(b) Royalties, fees, commission and other revenue;
(c) Refunds of income-tax.
(ii) Outflows of investing activities :
(a) Cash payments for acquisition of fixed assets;
(b) Cash payments for acquisition of shares, warrants or debts instruments of other enterprises and
interests in joint ventures (other than payments for instruments considered to cash equivalents
and those for dealing or trading purposes);
(c) Cash advances and loans to third parties.
Question 3
Classification of activities (with two examples) as suggested in AS 3, to be used for preparing a cash flow
statements. (5 marks) (IntermediateMay 2001)
Answer
AS 3 (Revised) on Cash Flow Statements requires that the cash flow statement should report cash flows by
operating, investing and financing activities.
(i) Operating activities are the principal revenue-producing activities of the enterprise and other
activities that are not investing or financing activities. Cash receipts from sale of goods and cash
payments to suppliers of goods are two examples of operating activities.
(ii) Investing activities are acquisition and disposal of long-term assets and other investments not
included in cash equivalents. Payment made to acquire machinery and cash received for sale of
furniture are examples of investing activities.
(iii) Financial activities are those activities that result in changes in the size and composition of the
owners capital (including preference share capital in the case of a company) and borrowings of the
enterprise. Cash proceeds from issue of shares and cash paid to redeem debentures are two
examples of financing activities.
Question 4
Explain the difference between direct and indirect methods of reporting cash flows from operating activities
with reference to Accounting Standard 3, (AS 3) revised.
(8 marks) (Final Nov. 2001)
Answer
As per para 18 of AS 3 (Revised) on Cash Flow Statements, an enterprise should report cash flows from
operating activities using either :

13.3
(a) the direct method, whereby major classes of gross cash receipts and gross cash payments are
disclosed; or
(b) the indirect method, whereby net profit or loss in adjusted for the effects of transactions of a non-cash
nature, any deferrals or accruals of past or future operating cash receipts or payments, and items of
income or expense associated with investing or financing cash flows.
The direct method provides information which may be useful in estimating future cash flows and which is
not available under the indirect method and is, therefore, considered more appropriate than the indirect
method. Under the direct method, information about major classes of gross cash receipts and gross cash
payments may be obtained either :
(a) from the accounting records of the enterprise; or
(b) by adjusting sales, cost of sales (interest and similar income and interest expense and similar charges
for a financial enterprise) and other items in the statment of profit and loss for :
(i) changes during the period in inventories and operating receivables and payables;
(ii) other non-cash items; and
(iii) other items for which the cash effects are investing or financing cash flows.
Under the indirect method, the net cash flow from operating activies is determined by adjusting net profit or loss
for the effects of :
(a) changes during the period in inventories and operating receivables and payables;
(b non-cash items such as depreciation, provisions, deferred taxes and unrealised foreign exchange
gains and losses; and
(c) all other items for which the cash effects are investing or financing cash flows.
Alternatively, the net cash flow from operating activities may be presented under the indirect method by showing
the operating revenues and expenses, excluding non-cash items disclosed in the statement of profit and loss and
the changes during the period in inventories and operating receivables and payables.
Question 5
What all are the differences between Cash Flow statement and Fund Flow statement?
(4 Marks) (PE-II May 2006)
Answer
Differences between cash flowstatement and fund flowstatement
(i) Cash flow statement deals with the change in cash position between two points of time. Fund flow
statement deals with the changes in working capital position.
(ii) Cash flow statement contains opening as well as closing balances of cash and cash equivalents. The
fund flow statement does not contain any such opening and closing balance.
(iii) Cash flow statement records only inflow and outflow of cash. Fund flow statement records sources
and application of funds.
(iv) Fund flow statement can be prepared from the cash flow statement under indirect method. However, a
cash flow statement cannot be prepared from fund flow statement.
(v) A statement of changes in working capital is usually prepared alongwith fund flow statement. No such
statement is prepared along with the cash flow statement.
(B) Practical Questions:
Question 1
Given below are the condensed Balance Sheets of Lambakadi Ltd. for two years and the statement of Profit and
Loss for one year :
(Figures Rs. in lakhs)
13.4
As at 31st March 1998 1997
Share Capital
In equity shares of Rs. 100 each 150 110
10% redeemable preference shares of Rs. 100 each 10 40
Capital redemption reserve 10
General reserve 15 10
Profit and loss account balance 30 20
8% debentures with convertible option 20 40
Other term loans 15 30
250 250
Fixed assets less depreciation 130 100
Long term investments 40 50
Working capital 80 100
250 250
Statement of Profit and Loss for the year ended 31st March, 1998
(Figures Rs. in lakhs)
Sales 600
Less : Cost of sales 400
200
Establishment charges 30
Selling and distribution expenses 60
Interest expenses 5
Loss on sale of equipment (Book value Rs. 40 lakhs) 15 110
90
Interest income 4
Dividend income 2
Foreign exchange gain 10
Damages received for loss of reputation 14 30
120
Depreciation 50
70
Taxes 30
40
Dividends 15
Net profit carried to Balance Sheet 25
Your are informed by the accountant that ledgers relating to debtors, creditors and stock for both the years were
seized by the income-tax authorities and it would take atleast two months to obtain copies of the same. However,
he is able to furnish the following data :
(Figures Rs. in lakhs)
1998 1997
Dividend receivable 2 4
Interest receivable 3 2
Cash on hand and with bank 7 10
Investments maturing within two months 3 2
15 18
Interest payable 4 5
Taxes payable 6 3
10 8
Current ratio 1.5 1.4
Acid test ratio 1.1 0.8

13.5
It is also gathered that debentureholders owning 50% of the debentures outstanding as on 31.3.97 exercised the
option for conversion into equity shares during the financial year and the same was put through.
You are required to prepare a direct method cash flow statement for the financial year, 1998 in accordance with
para 18(a) of Accounting Standard (AS) 3 revised. (20 marks) (Final May 1998)
Answer
Lambakadi Ltd.
Direct Method Cash FlowStatement
for the year ended 31st March, 1998
(Rs. in lakhs)
Cash flows fromoperating activities
Cash receipts from customers 621
Cash paid to suppliers and employees (496)
Cash generated from operations 125
Taxes paid (27)
Cash flows before extraordinary item 98
Damages received for loss of reputation 14
Net cash from operating activities 112
Cash flows frominvesting activities
Purchase of fixed assets (120)
Proceeds from sale of equipment 25
Proceeds from sale of long term investments 10
Interest received 3
Dividend received 4
Net cash used in investing activities (78)
Cash flows fromfinancing activities
Proceeds from issuance of share capital 20
Redemption of preference share capital (30)
Repayments of term loans (15)
Interest paid (6)
Dividend paid (15)
Net cash used in financing activities (46)
Net increase in cash and cash equivalents (12)
Cash and cash equivalents at beginning of period 12
(See Note 1 to the Cash Flow Statement)
Cash and cash equivalents at end of the period
(See Note 1 to the Cash Flow Statement) NIL
Notes to the Cash FlowStatement
(Rs. in lakhs)
1. Cash and Cash Equivalents 31.3.1998 31.3.1997
Cash on hand and with bank 7 10
Short-term investments 3 2
13.6
10 12
Effect of exchange rate changes (10)
Cash and cash equivalents Nil 12
2. Conversion of debentures into equity shares, a non-cash transaction, amounted to
Rs.20 lakhs.
Working Notes :
(Rs. in lakhs)
1. Calculation of debtors, creditors and stock 31.3.98 31.3.97
(a) Current Ratio 1.5:1 1.4:1
Working Capital to Current Liabilities Ratio 0.5:1 0.4:1
Working Capital (Rs.in lakhs) 80 100
Current Assets (Rs.in lakhs) 240
5 . 0
5 . 1 80

350
4 . 0
4 . 1 10


Current Liabilities (Rs.in lakhs) 240 80 = 160 350 100 = 250
(b) Current Ratio 1.5 1.4
Less : Acid Test Ratio 1.1 0.8
0.4 0.6
Stock : Current Liabilities 0.4:1 0.6:1
Stock (Rs.in lakhs) 160 0.4 = 64 250 0.6 = 150
(Rs. in lakhs)
(c) Break-up of Current Assets
Stock 64 150
Debtors (Balancing figures) 161 182
Other Current Assets 15 18
240 350
(d) Break-up of Current Liabilities
Creditors (Balancing figures) 150 242
Others 10 8
160 250
2. Cash receipts from customers
Sales 600
Add: Debtors at the beginning of the year 182
782
Less : Debtors at the end of the year 161
621
3. Cash paid to suppliers and employees
Cost of sales 400
Establishment charges 30
Selling and distribution expenses 60
490
Add: Creditors at the beginning of the year 242
Stock at the end of the year 64 306
796
Less : Creditors at the end of year 150
Stock at the beginning of the year 150 300
496

13.7


4. Taxes paid
Tax expense for the year 30
Add : Tax liability at the beginning of the year 3
33
Less : Tax liability at the end of year 6
27

5. Fixed assets acquisitions
W.D.V. at 31.3.1998 130
Add back : Depreciation for the year 50
Disposals 40
220
Less : W.D.V. at 31.3.1997 100
Purchase of fixed assets 120

6. Interest received
Interest income for the year 4
Add : Amount receivable at the beginning of the year 2
6
Less : Amount receivable at the end of the year 3
3

7. Dividend received
Dividend income for the year 2
Add : Amount receivable at the beginning of the year 4
6
Less : Amount receivable at the end of the year 2
4

8. Issue of shares
Equity share capital at the end of the year 150
Less : Equity share capital at the beginning of the year 110
40
Less : Conversion of debentures into equity shares
during the year (non-cash transaction) 20
Cash flow from issue of equity shares 20

9. Interest paid
Interest expense for the year 5
Add : Interest payable at the beginning of the year 5
10
Less : Interest payable at the end of the year 4
6
Notes :
1. It has been assumed that dividends for the year, Rs. 15 lakhs have been paid off.
13.8
2. It has been assumed that foreign exchange gain represents the effect of changes in exchange rates on
cash and cash equivalents held in a foreign currency.
Question 2
The following are the changes in the account balances taken from the Balance Sheets of PQ Ltd. as at the
beginning and end of the year. :
Changes in Rupees in
debt or [credit]
Equity share capital 30,000 shares of Rs. 10 each issued and fully paid 0
Capital reserve ]49,200]
8% debentures [50,000]
Debenture discount 1,000
Freehold property at cost/revaluation 43,000
Plant and machinery at cost 60,000
Depreciation on plant and machinery [14,400]
Debtors 50,000
Stock and work-in-progress 38,500
Creditors [11,800]
Net profit for the year [76,500]
Dividend paid in respect of earlier year 30,000
Provision for doubtful debts [3,300]
Trade investments at cost 47,000
Bank [64,300]
0
You are informed that.
(a) Capital reserve as at the end of the year represented realised profits on sale of one freehold property
together with surplus arising on the revaluation of balance of freehold properties.
(b) During the year plant costing Rs. 18,000 against which depreciation provision of Rs. 13,500 was lying,
was sold for Rs. 7,000.
(c) During the middle of the year Rs. 50,000 debentures were issued for cash at a discount of Rs. 1,000.
(d) The net profit for the year was after crediting the profit on sale of plant and charging debenture interest.
You are required to prepare a statement which will explain, why bank borrowing has increased by Rs. 64,300
during the year end. Ignore taxation. (15 marks)(Final Nov. 1998)
Answer
PQ Ltd.
Cash FlowStatement for the year ended...
Rs.
Cash flows from operating activities
Net profit 76,500
Adjustments for :
Depreciation 27,900
Profit on sale of plant (2,500)
Interest expense 2,000

13.9
Operating profit before working capital changes 1,03,900
Increase in debtors (less provision) (46,700)
Increase in stock and work-in-progress (38,500)
Increase in creditors 11,800
Net cash operating activities 30,500
Cash flows from investing activities
Purchase of plant and machinery (78,000)
Proceeds from sale of plant 7,000
Proceeds from sale of freehold property 6,200
Increase in trade investments (47,000)
Net cash used in investing activities (1,11,800)
Cash flows from financing activities
Proceeds from issuance od debentures at discount 49,000
Debenture interest paid (2,000)
Dividend paid in financing activities (30,000)
Net cash from financing activities 17,000
Excess of outflows over inflows 64,300
Thus the shortfall of Rs. 64,300 was made up through borrowing from bank.
Working Notes :
(1) Plant and Machinery Rs
Amount of increase (at cost) 60,000
Add : Disposal (at cost) 18,000
Acquisition during the year 78,000
Disposal of plant :
proceeds from sale 7,000
Net book value (18,000 13,500) 4,500
Profit on sale 2,500
(2) Freehold property
Capital Reserve 49,200
Less : Increase in freehold property (closing balance minus opening balance) 43,000
Proceeds from sale of freehold property 6,200

Memorandum Accounts
(a) Plant and Machinery Account
Rs. Rs.
To Balance b/d By Bank (Sale proceeds) 7,000
To Profit and Loss A/c 2,500 By Provision for Depreciation 13,500
(Profit on sale) By Balance c/d 60,000
To Bank (Balancing figure) 78,000
80,500 80,500

(b) Provision for Depreciation (Plant and Machinery) Account
To Plant and Machinery A/c 13,500 By Balance b/d
To Balance c/d 14,400 By Profit and Loss A/c 27,900
(Balancing figure)
13.10
27,900 27,900

(c) Freehold Property Account
To Balance b/d By Bank A/c 6,200
To Capital reserve 49,200 (Balancing figure)
By Balance c/d 43,000
49,200 49,200
In the absence of information about the opening balances, the entire amount of change has been considered
under the closing balances for the purpose of calculation of missing figures.
Notes :
(1) Investment income and dividend pertaining to the current year have not been considered in the absence
of any related information.
(2) Debenture interest has been calculated for 6 months @ 8% on Rs. 50,000.
Question 3
Examine the following schedule prepared by K Ltd.
K Ltd.
Schedule of funds provided by operations for the year ended 31st July, 1999
(Rs.000) (Rs.000)
Sales 32,760
Add : Decrease in bills receivable. 1,000
Less : Increase in accounts receivable (626)
Inflow from operating revenues 33,134
Cost of goods sold 18,588
Less : Decrease in inventories (212)
Add : Decrease in trades payable 81 18,457
Wages and Salaries 5,284
Less : Increase in wages payable (12) 5,272
Administrative Expenses 3,066
Add : Increase in prepaid expenses 11 3,077
Property taxes 428
Interest expenses 532
Add : Amortisation of premium on bonds payable 20 552
Outflow from operating expenses 27,786
From operations 5,348
Rent Income 207
Add : Increase in unearned rent 3 210
5,558
Income tax 1,330
Less : Increase in deferred tax 50 1,280

13.11
Funds from operations 4,278
Required :
(i) What is the definition of funds shown in the schedule?
(ii) What amount was reported as gross margin in the income statement?
(iii) How much cash was collected from the customers?
(iv) How much cash was paid for the purchases made?
(v) As a result of change in inventories, did the working capital increase or decrease and by what
amount?
(vi) How much rent was actually earned during the year?
(vii) What was the amount of tax expenses reported on the income statement?
Can you reconcile the profit after tax-with the funds provided by the operations?
(16 marks)(Final May 2000)
Answer
(i) Funds shown in the schedule refer to the cash and cash equivalents [as defined in AS 3 (Revised) on
Cash Flow Statements].
(ii) Gross margin in the income statement :
Rs. (000)
Sales 32,760
Cost of goods sold 18,588
14,172
(iii) Cash collected from the customers 33,134
(iv) Cash paid for purchases made 18,457
(v) Change in inventories would reduce the working capital by 212
(vi) Rental income earned during the year 207
(vii) Tax expenses reported in the income statement 1330

(Viii) Reconciliation Statement Rs.(000)
Profit after tax (See W.N.) 3,719
Decrease in bills receivable 1,000
Increase in accounts receivable (626)
Decrease in inventories 212
Decrease in trades payable (81)
Increase in wages payable 12
Increase in prepaid expenses (11)
Increase in unearned rent 3
Increase in deferred tax 50
Funds from operations as shown in the schedule 4,278
(i.e. cash and cash equivalents)

Working Note :
13.12
Calculation of Profit after Tax Rs. (000)
Sales 32,760
Less : Cost of goods sold 18,588
Gross margin 14,172
Add : Rental income 207
14,379
Less : Wages and salaries 5,284
Administrative expenses 3,066
Property taxes 428
Interest expenses 532
Amortisation of premium on bonds payable 20
9,330
Profit before tax 5,049
Less : Income tax 1,330
Profit after tax 3,719
Question 4
Ms. Joyti of Star Oils Limited has collected the following information for the preparation of cash flow statement for
the year 2000 :
(Rs. in Lakhs)
Net Profit 25,000
Dividend (including dividend tax) paid 8,535
Provision for Income tax 5,000
Income tax paid during the year 4,248
Loss on sale of assets (net) 40
Book value of the assets sold 185
Depreciation charged to Profit & Loss Account 20,000
Amortisation of Capital grant 6
Profit on sale of Investments 100
Carrying amount of Investment sold 27,765
Interest income on investments 2,506
Increase expenses 10,000
Interest paid during the year 10,520
Increase in Working Capital (excluding Cash & Bank Balance) 56,075
Purchase of fixed assets 14,560
Investment in joint venture 3,850
Expenditure on construction work in progress 34,740
Proceeds from calls in arrear 2
Receipt of grant for capital projects 12
Proceeds from long-term borrowings 25,980
Proceeds from short-term borrowings 20,575
Opening cash and Bank balance 5,003

13.13
Closing cash and Bank balance 6,988
Required :
Prepare the Cash Flow Statement for the year 2000 in accordance with AS 3, Cash Flow Statements issued by
the Institute of Chartered Accounants of India. (make necessary assumptions).(16 marks)(Final May 2001)
Answer
Star Oils Limited
Cash FlowStatement
for the year ended 31st December, 2000
(Rs. in lakhs)
Cash flows fromoperating activities
Net profit before taxation (25,000 + 5,000) 30,000
Adjustments for :
Depreciation 20,000
Loss on sale of assets (Net) 40
Amortisation of capital grant (6)
Profit on sale of investments (100)
Interest income on investments (2,506)
Interest expenses 10,000
Operating profit before working capital changes 57,428
Changes in working capital (Excluding cash and bank balance) (56,075)
Cash generated from operations 1,353
Income taxes paid (4,248)
Net cash used in operating activities (2,895)
Cash flows frominvesting activities
Sale of assets 145
Sale of investments (27,765 + 100) (27,865)
Interest income on investments 2,506
Purchase of fixed assets (14,560)
Investment in joint venture (3,850)
Expenditure on construction work-in progress (34,740)
Net cash used in investing activities (22,634)
Cash flows fromfinancing activities
Proceeds from calls in arrear 2
Receipts of grant for capital projects 12
Proceeds from long-term borrowings 25,980
Proceed from short-term borrowings 20,575
Interest paid (10,520)
Dividend (including dividend tax) paid (8,535)
27,514
Net increase in cash and cash equivalents 1,985
Cash and cash equivalents at the beginning of the period 5,003
Cash and cash equivalents at the end of the period 6,988
13.14
Working note :
Book value of the assets sold 185
Less : Loss on sale of assets 40
Proceeds on sale 145
Assumption :
Interest income on investments Rs. 2,506 has been received during the year.
Question 5
From the following Summary Cash Account of X Ltd. prepare Cash Flow Statement for the year ended 31st
March, 2001 in accordance with AS 3 (Revised) using the direct method. The company does not have any cash
equivalents.
Summary Cash Account for the year ended 31.3.2001
Rs. 000 Rs. 000
Balance on 1.4.2000 50 Payment to Suppliers 2,000
Issue of Equity Shares 300 Purchase of Fixed Assets 200
Receipts from Customers 2,800 Overhead expense 200
Sale of Fixed Assets 100 Wages and Salaries 100
Taxation 250
Dividend 50
Repayment of Bank Loan 300
Balance on 31.3.2001 150
3,250 3,250
(8 marks)(Final Nov. 2001)
Answer
X Ltd.
Cash FlowStatement for the year ended 31st March, 2001
(Using the direct method)
Rs. 000 Rs.000
Cash flows fromoperating activities
Cash receipts from customers 2,800
Cash payments to suppliers (2,000)
Cash paid to employees (100)
Cash payments for overheads (200)
Cash generated from operations 500
Income tax paid (250)
Net cash from operating activities 250
Cash flows frominvesting activities
Payments for purchase of fixed assets (200)
Proceeds from sale of fixed assets 100
Net cash used in investing activities (100)
Cash flows fromfinancing activities

13.15
Proceeds from issuance of equity shares 300
Bank loan repaid (300)
Dividend paid (50)
Net cash used in financing activities (50)
Net increase in cash 100
Cash at beginning of the period 50
Cash at end of the period 150
13.16
Question 6
From the following details relating to the Accounts of Grow More Ltd. prepare Cash Flow Statement:
Liabilities 31.03.2002 (Rs.) 31.03.2001 (Rs.)
Share Capital 10,00,000 8,00,000
Reserve 2,00,000 1,50,000
Profit and Loss Account 1,00,000 60,000
Debentures 2,00,000
Provision for taxation 1,00,000 70,000
Proposed dividend 2,00,000 1,00,000
Sundry Creditors 7,00,000 8,20,000
25,00,000 20,00,000
Assets
Plant and Machinery 7,00,000 5,00,000
Land and Building 6,00,000 4,00,000
Investments 1,00,000
Sundry Debtors 5,00,000 7,00,000
Stock 4,00,000 2,00,000
Cash on hand/Bank 2,00,000 2,00,000
25,00,000 20,00,000
(i) Depreciation @ 25% was charged on the opening value of Plant and Machinery.
(ii) During the year one old machine costing 50,000 (WDV 20,000) was sold for Rs. 35,000.
(iii) Rs. 50,000 was paid towards Income tax during the year.
(iv) Building under construction was not subject to any depreciation.
Prepare Cash flow Statement. (16 marks) (PE-IINov. 2002)
Answer
GrowMore Ltd
Cash FlowStatement
for the year ended 31
st
March, 2002
Cash Flow from Operating Activities
Net Profit 40,000
Proposed Dividend 2,00,000
Provision for taxation 80,000
Transfer to General Reserve 50,000
Depreciation 1,25,000
Profit on sale of Plant and Machinery (15,000)
Operating Profit before Working Capital changes 4,80,000
Increase in Stock (2,00,000)
Decrease in debtors 2,00,000
Decrease in creditors (1,20,000)

13.17
Cash generated from operations 3,60,000
Income tax paid (50,000)
Net Cash from operating activities 3,10,000
Cash Flow from Inventing Activities
Purchase of fixed assets (3,45,000)
Expenses on building (2,00,000)
Increase in investments (1,00,000)
Sale of old machine 35,000
Net Cash used ininvesting activities (6,10,000)
Cash Flow from financing activities:
Proceeds from issue of shares 2,00,000
Proceeds from issue of debentures 2,00,000
Dividend paid (1,00,000)
Net cash used in financing activities 3,00,000
Net increase in cash or cash equivalents NIL
Cash and Cash equivalents at the beginning of the year 2,00,000
Cash and Cash equivalents at the end of the year 2,00,000
Working Notes:
Provision for taxation account
Rs. Rs.
To Cash (Paid) 50,000 By Balance b/d 70,000
To Balance c/d 1,00,000 By Profit and Loss A/c 80,000
(Balancing figure)
1,50,000 1,50,000

13.18
Plant and Machinery account
Rs. Rs.
To Balance b/d 5,00,000 By Depreciation 1,25,000
To Cash (Balancing figure) 3,45,000 By Cash (sale of machine) 20,000
_______ By Balance c/d 7,00,000
8,45,000 8,45,000
Question 7
From the following Balance Sheet and information, prepare Cash Flow Statement of Ryan Ltd. for the
year ended 31st March, 2003:
Balance Sheet
31st March,
2003
31st March,
2002
Rs. Rs.
Liabilities
Equity Share Capital 6,00,000 5,00,000
10% Redeemable Preference
Capital



2,00,000
Capital Redemption Reserve 1,00,000
Capital Reserve 1,00,000
General Reserve 1,00,000 2,50,000
Profit and Loss Account 70,000 50,000
9% Debentures 2,00,000
Sundry Creditors 95,000 80,000
Bills Payable 20,000 30,000
Liabilities for Expenses 30,000 20,000
Provision for Taxation 95,000 60,000
Proposed Dividend 90,000 60,000
15,00,000 12,50,000

31st March,
2003
31st March,
2002
Rs. Rs.
Assets
Land and Building 1,50,000 2,00,000
Plant and Machinery 7,65,000 5,00,000
Investments 50,000 80,000
Inventory 95,000 90,000
Bills Receivable 65,000 70,000
Sundry Debtors 1,75,000 1,30,000
Cash and Bank 65,000 90,000
Preliminary Expenses 10,000 25,000
Voluntary Separation Payments 1,25,000 65,000
15,00,000 12,50,000

13.19
Additional Information:
(i) A piece of land has been sold out for Rs. 1,50,000 (Cost Rs. 1,20,000) and the balance land was
revalued. Capital Reserve consisted of profit on sale and profit on revaluation.
(ii) On 1st April, 2002 a plant was sold for Rs. 90,000 (Original Cost Rs. 70,000 and W.D.V. Rs.
50,000) and Debentures worth Rs. 1 lakh was issued at par as part consideration for plant of Rs. 4.5
lakhs acquired.
(iii) Part of the investments (Cost Rs. 50,000) was sold for Rs. 70,000.
(iv) Pre-acquisition dividend received Rs. 5,000 was adjusted against cost of investment.
(v) Directors have proposed 15% dividend for the current year.
(vi) Voluntary separation cost of Rs. 50,000 was adjusted against General Reserve.
(vii) Income-tax liability for the current year was estimated at Rs. 1,35,000.
(viii) Depreciation @ 15% has been written off from Plant account but no depreciation has been charged on
Land and Building. (20 marks) (PE-IIMay 2003)
Answer
Cash FlowStatement of Ryan Limited
For the year ended 31st March, 2003
Cash flowfromoperating activities Rs. Rs.
Net Profit before taxation 2,45,000
Adjustment for
Depreciation 1,35,000
Preliminary expenses 15,000
Profit on sale of plant (40,000)
Profit on sale of investments (20,000)
Interest on debentures 18,000
Operating profit before working capital changes 3,53,000
Increase in inventory (5,000)
Decrease in bills receivable 5,000
Increase in debtors (45,000)
Increase in creditors 15,000
Decrease in bills payable (10,000)
Increase in accrued liabilities 10,000
Cash generated fromoperations 3,23,000
Income taxes paid (1,00,000)
2,23,000
Voluntary separation payments (1,10,000)
Net cash from operating activities 1,13,000
Cash flowfrominvesting activities
Proceeds from sale of land 1,50,000
Proceeds from sale of plant 90,000
Proceeds from sale of investments 70,000
13.20
Purchase of plant (3,50,000)
Purchase of investments (25,000)
Pre-acquisition dividend received 5,000
Net cash used in investing activities (60,000)
Cash flowfromfinancing activities
Proceeds from issue of equity shares 1,00,000
Proceeds from issue of debentures 1,00,000
Redemption of preference shares (2,00,000)
Dividends paid (60,000)
Interest paid on debentures (18,000)
Net cash used in financing activities (78,000)
Net decrease in cash and cash equivalents (25,000)
Cash and cash equivalents at the beginning of the year 90,000
Cash and Cash equivalents at the end of the year 65,000
Working Notes:
1. Rs.
Net profit before taxation
Retained profit 70,000
Less: Balance as on 31.3.2002 (50,000)
20,000
Provision for taxation 1,35,000
Proposed dividend 90,000
2,45,000

2. Land and Building Account
Rs. Rs.
To Balance b/d 2,00,000 By Cash (Sale) 1,50,000
To Capital reserve (Profit on sale) 30,000 By Balance c/d 1,50,000
To Capital reserve
(Revaluation profit)

70,000

_______
3,00,000 3,00,000

3. Plant and Machinery Account
Rs. Rs.
To Balance b/d 5,00,000 By Cash (Sale) 90,000
To Profit and loss account 40,000 By Depreciation 1,35,000
To Debentures 1,00,000 By Balance c/d 7,65,000
To Bank 3,50,000
9,90,000 9,90,000

4. Investments Account
Rs. Rs.

13.21
To Balance b/d 80,000 By Cash (Sale) 70,000
To
To
Profit and loss account
Bank (Balancing figure)
20,000
25,000
By Dividend
(Pre-acquisition)

5,000
_______ By Balance c/d 50,000
1,25,000 1,25,000

5. Capital Reserve Account
Rs. Rs.
To Balance c/d 1,00,000 By Profit on sale of land 30,000

_______
By Profit on revaluation
of land

70,000
1,00,000 1,00,000

6. General Reserve Account
Rs. Rs.
To Voluntary separation cost 50,000 By Balance b/d 2,50,000
To
To
Capital redemption reserve
Balance c/d
1,00,000
1,00,000

_______
2,50,000 2,50,000

7. Proposed Dividend Account
Rs. Rs.
To Bank (Balancing figure) 60,000 By Balance b/d 60,000
To Balance c/d 90,000 By Profit and loss account 90,000
1,50,000 1,50,000

8. Provision for Taxation Account
Rs. Rs.
To Bank (Balancing figure) 1,00,000 By Balance b/d 60,000
To Balance c/d 95,000 By Profit and loss account 1,35,000
1,95,000 1,95,000

9. Voluntary Separation Payments Account
Rs. Rs.
To Balance b/d 65,000 By General reserve 50,000
To Bank (Balancing figure) 1,10,000 By Balance c/d 1,25,000
1,75,000 1,75,000
Note: Cash Flow statement has been prepared using indirect method.
Question 8
The Balance Sheet of New Light Ltd. for the years ended 31st March, 2001 and 2002 are as follows:
Liabilities 31st
March
2001
31st
March
2002
Assets 31st
March
2001
31st
March
2002
13.22
(Rs.) (Rs.) (Rs.) (Rs.)
Equity share capital 12,00,000 16,00,000 Fixed Assets 32,00,000 38,00,000
10% Preference
share capital

4,00,000

2,80,000
Less: Depreciation 9,20,000
22,80,000
11,60,000
26,40,000
Capital Reserve 40,000 Investment 4,00,000 3,20,000
General Reserve 6,80,000 8,00,000 Cash 10,000 10,000
Profit and Loss A/c 2,40,000 3,00,000 Other current assets 11,10,000 13,10,000
9% Debentures 4,00,000 2,80,000 Preliminary expenses 80,000 40,000
Current liabilities 4,80,000 5,20,000
Proposed dividend 1,20,000 1,44,000
Provision for Tax 3,60,000 3,40,000
Unpaid dividend 16,000 ________ ________
38,80,000 43,20,000 38,80,000 43,20,000

Additional information:
(i) The company sold one fixed asset for Rs. 1,00,000, the cost of which was Rs. 2,00,000 and the
depreciation provided on it was Rs. 80,000.
(ii) The company also decided to write off another fixed asset costing Rs. 56,000 on which depreciation
amounting to Rs. 40,000 has been provided.
(iii) Depreciation on fixed assets provided Rs. 3,60,000.
(iv) Company sold some investment at a profit of Rs. 40,000, which was credited to capital reserve.
(v) Debentures and preference share capital redeemed at 5% premium.
(vi) Company decided to value stock at cost, whereas previously the practice was to value stock at cost
less 10%. The stock according to books on 31.3.2001 was Rs. 2,16,000. The stock on 31.3.2002 was
correctly valued at Rs. 3,00,000.
Prepare Cash Flow Statement as per revised Accounting Standard 3 by indirect method.
(16 marks) (PE-IINov. 2003)
Answer
NewLight Ltd.
Cash FlowStatement for the year ended 31st March, 2002

A. Cash Flowfromoperating activities Rs. Rs.
Profit after appropriation
Increase in profit and loss A/c after inventory
adjustment [Rs.3,00,000 (Rs.2,40,000 + Rs.24,000)]

36,000

Transfer to general reserve 1,20,000
Proposed dividend 1,44,000
Provision for tax 3,40,000
Net profit before taxation and extraordinary item 6,40,000
Adjustments for:
Preliminary expenses written off 40,000
Depreciation 3,60,000
Loss on sale of fixed assets 20,000
Decrease in value of fixed assets 16,000

13.23
Premium on redemption of preference share capital 6,000
Premium on redemption of debentures 6,000
Operating profit before working capital changes 10,88,000
Increase in current liabilities
(Rs.5,20,000 Rs.4,80,000)

40,000

Increase in other current assets
[Rs.13,10,000 (Rs.11,10,000 + Rs.24,000)]

(1,76,000)

Cash generated from operations 9,52,000
Income taxes paid (3,60,000)
Net Cash from operating activities 5,92,000

B. Cash Flowfrominvesting activities
Purchase of fixed assets (8,56,000)
Proceeds from sale of fixed assets 1,00,000
Proceeds from sale of investments 1,20,000
Net Cash from investing activities (6,36,000)
C. Cash Flowfromfinancing activities
Proceeds from issuance of share capital 4,00,000
Redemption of preference share capital
(Rs.1,20,000 + Rs.6,000)
(1,26,000)
Redemption of debentures (Rs. 1,20,000 + Rs. 6,000) (1,26,000)
Dividend paid (1,04,000)
Net Cash from financing activities 44,000
Net increase/decrease in cash and cash equivalent
during the year

Nil
Cash and cash equivalent at the beginning of the year 10,000
Cash and cash equivalent at the end of the year 10,000
Working Notes:
1. Revaluation of stock will increase opening stock by Rs. 24,000.
24,000 Rs. 10
90
2,16,000


Therefore, opening balance of other current assets would be as follows:
Rs. 11,10,000 + Rs. 24,000 = Rs. 11,34,000
Due to under valuation of stock, the opening balance of profit and loss account be increased by Rs.
24,000.
The opening balance of profit and loss account after revaluation of stock will be
Rs. 2,40,000 + Rs. 24,000 = Rs. 2,64,000
2. Investment Account
Rs. Rs.
To
To
Balance b/d
Capital reserve A/c
(Profit on sale of
investment)
4,00,000


40,000
By


By
Bank A/c
(balancing figure being investment
sold)
Balance c/d
1,20,000


3,20,000
4,40,000 4,40,000
13.24
3. Fixed Assets Account
Rs. Rs. Rs.
To Balance b/d 32,00,000 By Bank A/c (sale of assets) 1,00,000
To Bank A/c
(balancing figure
being assets
purchased)
8,56,000 By

By

Accumulated
depreciation A/c
Profit and loss A/c(loss
on sale of assets)

80,000

20,000



2,00,000
By Accumulated
depreciation A/c

40,000

By Profit and loss A/c
(assets written off)

16,000

56,000
By Balance c/d 38,00,000
40,56,000 40,56,000
4. Accumulated Depreciation Account
Rs. Rs.
To Fixed assets A/c 80,000 By Balance b/d 9,20,000
To Fixed assets A/c 40,000 By Profit and loss A/c
To Balance c/d 11,60,000 (depreciation for the period) 3,60,000
12,80,000 12,80,000

5. Unpaid dividend is taken as non-current item and dividend paid is shown at Rs. 1,04,000 (Rs.1,20,000
Rs.16,000).
Note: Alternatively, unpaid dividend can be assumed as current liability and hence, dividend paid can be shown
at Rs. 1,20,000. Due to this assumption cash flow from operating activities would be affected. The cash flow
from operating activities will increase by Rs. 16,000 to Rs. 6,08,000 and cash flow from financing activities will
get reduced by Rs. 16,000 to Rs. 28,000.
Question 9
ABC Ltd. gives you the following informations. You are required to prepare Cash Flow Statement by using
indirect methods as per AS 3 for the year ended 31.03.2004:
Balance Sheet as on
Liabilities 31
st
March
2003
31
st
March
2004
Assets 31
st
March
2003
31
st
March
2004
Rs. Rs. Rs. Rs.
Capital 50,00,000 50,00,000 Plant & Machinery 27,30,000 40,70,000
Retained Earnings 26,50,000 36,90,000 Less: Depreciation 6,10,000 7,90,000
Debentures 9,00,000 21,20,000 32,80,000
Current Liabilities Current Assets
Creditors 8,80,000 8,20,000 Debtors 23,90,000 28,30,000
Bank Loan 1,50,000 3,00,000 Less: Provision 1,50,000 1,90,000
Liability for expenses 3,30,000 2,70,000 22,40,000 26,40,000
Dividend payable 1,50,000 3,00,000 Cash 15,20,000 18,20,000
Marketable
securities
11,80,000 15,00,000
Inventories 20,10,000 19,20,000
Prepaid Expenses 90,000 1,20,000

13.25
91,60,000 1,12,80,000 91,60,000 1,12,80,000
Additional Information:
(i) Net profit for the year ended 31st March, 2004, after charging depreciation Rs. 1,80,000 is Rs.
22,40,000.
(ii) Debtors of Rs. 2,30,000 were determined to be worthless and were written off against the provisions
for doubtful debts account during the year.
(ii) ABC Ltd. declared dividend of Rs. 12,00,000 for the year 2003-2004.
(16 marks) (PE-IIMay 2004)
Answer
Cash flowStatement of ABC Ltd. for the year ended 31.3.2004
Cash flows fromOperating activities Rs. Rs.
Net Profit 22,40,000
Add: Adjustment for Depreciation
(Rs.7,90,000 Rs.6,10,000)

1,80,000

Operating profit before working capital changes 24,20,000
Add: Decrease in Inventories
(Rs.20,10,000 Rs.19,20,000)

90,000

Increase in provision for doubtful debts
(Rs. 4,20,000 Rs.1,50,000)

2,70,000

27,80,000
Less: Increase in Current Assets:
Debtors (Rs. 30,60,000 Rs.23,90,000) 6,70,000
Prepaid expenses (Rs. 1,20,000 Rs.90,000) 30,000


Decrease in current liabilities:
Creditors (Rs. 8,80,000 Rs. 8,20,000) 60,000
Expenses outstanding
(Rs. 3,30,000 Rs.2,70,000)

60,000

8,20,000

Net cash from operating activities 19,60,000


Cash flows fromInvesting activities
Purchase of Plant & Equipment
(Rs. 40,70,000 Rs.27,30,000)

13,40,000

Net cash used in investing activities (13,40,000)
Cash flows fromFinancing Activities
Bank loan raised (Rs. 3,00,000 Rs. 1,50,000) 1,50,000
Issue of debentures 9,00,000
Payment of Dividend (Rs. 12,00,000 Rs. 1,50,000) (10,50,000)
Net cash used in financing activities NIL
Net increase in cash during the year 6,20,000
Add: Cash and cash equivalents as on 1.4.2003
13.26
(Rs. 15,20,000 +Rs.11,80,000) 27,00,000
Cash and cash equivalents as on 31.3.2004
(Rs. 18,20,000 + Rs.15,00,000)

33,20,000

Note: Bad debts amounting Rs. 2,30,000 were written off against provision for doubtful debts account during the
year. In the above solution, Bad debts have been added back in the balances of provision for doubtful debts and
debtors as on 31.3.2004. Alternatively, the adjustment of writing off bad debts may be ignored and the solution
can be given on the basis of figures of debtors and provision for doubtful debts as appearing in the balance sheet
on 31.3.2004.
Question 10
From the following balance sheets of Sneha Ltd. as on 31.3.2003 and 31.3.2004 prepare a statement of
sources and applications of fund and a schedule of changes in working capital for the year ending
31.3.2004:
Balance Sheets
Liabilities 31.3.2003 31.3.2004 Assets 31.3.2003 31.3.2004
Rs. Rs. Rs. Rs.
Equity share capital 13,00,000 16,90,000 Goodwill 65,000 42,500
Profit and loss account 4,90,100 8,77,500 Building 11,70,000 11,37,500
10% Debentures 16,25,000 13,00,000 Machinery 16,18,500 21,38,500
Creditors 9,00,000 10,00,000 Non-trade investments 5,07,000 3,93,250
Bills payable 42,500 1,70,000 Debtors 4,16,000 11,70,000
Provision for tax 2,60,000 9,75,000 Stock 5,07,000 7,99,500
Dividend payable 42,250 Cash 2,60,000 2,92,500
Prepaid expenses 42,250 52,000
Debenture discount 31,850 29,000
46,17,600 60,54,750 46,17,600 60,54,750

The following additional information is given:
(i) Building Machinery
Rs. Rs.
Accumulated depreciation 31.3.2003 4,87,500 15,92,500
Accumulated depreciation 31.3.2004 5,20,000 15,66,500
Depreciation for 2003-2004 32,500 1,36,500


(ii) Profit and loss account for 2003-2004 is as follows:
Rs.
Balance as on 31.3.2003 4,90,100
Add: Profit for 2003-2004 4,71,900
9,62,000
Less: Dividend 84,500
8,77,500
(iii) During 2003-2004 machinery costing Rs. 2,92,500 was sold for Rs. 97,500.
(iv) Investments which were sold for Rs. 1,17,000 had cost Rs. 97,500.
(v) Provision for Taxation and Dividend are to be taken as Non-current liabilities.

13.27
(20 marks) (PE-IINov. 2004)
Answer
(a) Sneha Ltd.
Fund FlowStatement
for the year ended 31st March, 2004
Amount (Rs.)
Sources of funds
Share capital
(Rs. 16,90,000 Rs. 13,00,000)
3,90,000
Sale of machinery 97,500
Sale of investments 1,17,000
Funds from operation (W.N. 1) 16,70,500
22,75,000
Applications of funds
Debentures redeemed
(Rs. 16,25,000 Rs. 13,00,000)
3,25,000
Machinery purchased (W.N. 4) 7,86,500
Tax paid

2,60,000
Dividend (Rs. 84,500 Rs. 42,250) 42,250
Increase in working capital 8,61,250
22,75,000
Schedule of Changes in Working Capital
for the year ended 31st March, 2004
Balance as on Changes in working capital
1.4.2003 31.3.2004 Increase Decrease
Rs. Rs. Rs. Rs.
Current Assets:
Debtors 4,16,000 11,70,000 7,54,000
Stock 5,07,000 7,99,500 2,92,500
Cash 2,60,000 2,92,500 32,500
Prepaid expenses 42,250 52,000 9,750
A 12,25,250 23,14,000
Current Liabilities:
Creditors 9,00,000 10,00,000 1,00,000
Bills payable 42,500 1,70,000 1,27,500
B 9,42,500 11,70,000 10,88,750 2,27,500
Working capital (A B) 2,82,750 11,44,000
Increase in working
capital

________

8,61,250
10,88,750 10,88,750

The provision for taxation has been treated as a non-current liability as per the requirement of the question. Last
years provision for taxation amounting Rs. 2,60,000 has been assumed to be paid in the current year ended 31st
March, 2004.
13.28
Working Notes:
1. Statement showing funds generated fromoperations
(Rs.)
Increase in profit and loss account during the year
(Rs. 8,77,500 Rs. 4,90,100)
3,87,400
Add: Non-cash expenditures
(1) Loss on sale of machinery (W.N. 4) 32,500
(2) Investments written off (W.N. 2) 16,250
(3) Provision for tax 9,75,000
(4) Depreciation
on building (Rs. 11,70,000 Rs. 11,37,500) 32,500

on machinery (W.N. 3) 1,36,500 1,69,000
(5) Goodwill written off (Rs. 65,000 Rs. 42,500) 22,500
(6) Debenture discount written off (Rs. 31,850 Rs. 29,000) 2,850
(7) Dividend 84,500 13,02,600
16,90,000
Less: Non-cash incomes
(1) Profit on sale of investments (Rs. 1,17,000 Rs. 97,500) 19,500
Funds from operations 16,70,500

2. Non Trade Investment Account
Dr. Cr.
Rs. Rs.
To Balance b/d 5,07,000 By Bank -Sale 1,17,000
To Profit on sale
(Rs. 1,17,000 Rs. 97,500)
19,500

By

Profit and loss account written off
(balancing figure)

16,250
_______ By Balance c/d 3,93,250
5,26,500 5,26,500


13.29
3. Provision for Depreciation on Machinery Account
Dr. Cr.
Rs. Rs.
To Machinery -sale (balancing
figure)
1,62,500 By
By
Balance b/d
Depreciation
15,92,500
1,36,500
To Balance c/d 15,66,500
17,29,000 17,29,000

4. Machinery Account
Dr. Cr.
Rs. Rs.
To Balance b/d 16,18,500 By Bank (sale) 97,500
Add: Provision for
depreciation

15,92,500

32,11,000
By
By
Depreciation
Loss on sale
1,62,500
32,500
To Bank -purchase
(balancing figure)

7,86,500
By Balance c/d
W.D.V.

21,38,500

________ Add: Provision for
depreciation

15,66,500

37,05,000
39,97,500 39,97,500
Question 11
The following figures have been extracted from the Books of X Limited for the year ended on 31.3.2004.
You are required to prepare a cash flow statement.
(i) Net profit before taking into account income tax and income from law suits but after taking into account
the following items was Rs. 20 lakhs:
(a) Depreciation on Fixed Assets Rs. 5 lakhs.
(b) Discount on issue of Debentures written off Rs. 30,000.
(c) Interest on Debentures paid Rs. 3,50,000.
(d) Book value of investments Rs. 3 lakhs (Sale of Investments for Rs. 3,20,000).
(e) Interest received on investments Rs. 60,000.
(f) Compensation received Rs. 90,000 by the company in a suit filed.
(ii) Income tax paid during the year Rs. 10,50,000.
(iii) 15,000, 10% preference shares of Rs. 100 each were redeemed on 31.3.2004 at a premium of 5%.
Further the company issued 50,000 equity shares of Rs. 10 each at a premium of 20% on 2.4.2003.
Dividend on preference shares were paid at the time of redemption.
13.30
(iv) Dividends paid for the year 2002-2003 Rs. 5 lakhs and interim dividend paid Rs. 3 lakhs for the year
2003-2004.
(v) Land was purchased on 2.4.2003 for Rs. 2,40,000 for which the company issued 20,000 equity shares
of Rs. 10 each at a premium of 20% to the land owner as consideration.
(vi) Current assets and current liabilities in the beginning and at the end of the years were as detailed
below:
As on 31.3.2003 As on 31.3.2004
Rs. Rs.
Stock 12,00,000 13,18,000
Sundry Debtors 2,08,000 2,13,100
Cash in hand 1,96,300 35,300
Bills receivable 50,000 40,000
Bills payable 45,000 40,000
Sundry Creditors 1,66,000 1,71,300
Outstanding expenses 75,000 81,800
(20 marks) (PE-II May 2005)
Answer
X Ltd.
Cash FlowStatement
for the year ended 31st March, 2004
Rs. Rs.
Cash flow from Operating Activities
Net profit before income tax and extraordinary items: 20,00,000
Adjustments for:
Depreciation on fixed assets 5,00,000
Discount on issue of debentures 30,000
Interest on debentures paid 3,50,000
Interest on investments received (60,000)
Profit on sale of investments (20,000) 8,00,000
Operating profit before working capital changes 28,00,000
Adjustments for:
Increase in stock (1,18,000)
Increase in sundry debtors (5,100)
Decrease in bills receivable 10,000
Decrease in bills payable (5,000)
Increase in sundry creditors 5,300
Increase in outstanding expenses 6,800
(1,06,000)
Cash generated from operations 26,94,000
Income tax paid (10,50,000)
16,44,000
Cash flow from extraordinary items:
Compensation received in a suit filed 90,000
Net cash flow from operating activities 17,34,000

13.31

Cash flowfromInvesting Activities
Sale proceeds of investments 3,20,000
Interest received on investments 60,000
Net cash flow from investing activities 3,80,000
Cash flowfromFinancing Activities
Proceeds by issue of equity shares at 20% premium 6,00,000
Redemption of preference shares at 5% premium (15,75,000)
Preference dividend paid (1,50,000)
Interest on debentures paid (3,50,000)
Dividend paid (5,00,000 + 3,00,000) (8,00,000)
Net cash used in financing activities (22,75,000)
Net decrease in cash and cash equivalents during the year (1,61,000)
Add: Cash and cash equivalents as on 31.3.2003 1,96,300
Cash and cash equivalents as on 31.3.2004 35,300
Note: Purchase of land in exchange of equity shares (issued at 20% premium) has not been considered
in the cash flow statement as it does not involve any cash transaction.
Question 12
Raj Ltd. gives you the following information for the year ended 31
st
March, 2006:
(i) Sales for the year Rs.48,00,000. The Company sold goods for cash only.
(ii) Cost of goods sold was 75% of sales.
(iii) Closing inventory was higher than opening inventory by Rs.50,000.
(iv) Trade creditors on 31.3.2006 exceed the outstanding on 31.3.2005 by Rs.1,00,000.
13.32
(v) Tax paid during the year amounts to Rs.1,50,000.
(vi) Amounts paid to Trade creditors during the year Rs.35,50,000.
(vii) Administrative and Selling expenses paid Rs.3,60,000.
(viii) One new machinery was acquired in December, 2005 for Rs.6,00,000.
(ix) Dividend paid during the year Rs.1,20,000.
(x) Cash in hand and at Bank on 31.3.2006 Rs.70,000.
(xi) Cash in hand and at Bank on 1.4.2005 Rs.50,000.
Prepare Cash Flow Statement for the year ended 31.3.2006 as per the prescribed Accounting standard.
(12 Marks) (PE-II May 2006)
Answer
Cash flowstatement of Raj Limited
for the year ended 31.3.2006
Direct Method

Cash flow fromoperating activities:
Rs. Rs.
Cash receipt from customers (sales) 48,00,000
Cash paid to suppliers and expenses
(Rs.35,50,000 + Rs.3,60,000)

39,10,000

Cash flow from operation 8,90,000
Less: Tax paid 1,50,000
Net cash from operating activities 7,40,000
Cash flow frominvesting activities:
Purchase of fixed assets (6,00,000)
Net cash used in investing activities (6,00,000)
Cash flow fromfinancing activities:
Dividend Paid (1,20,000)
Net cash from financing activities (1,20,000)
20,000
Add: Opening balance of Cash in Hand and at Bank 50,000
Cash in Hand and at Bank on 31.3.2006 70,000


13.33
Question 13
The following are the summarized Balance Sheets of X Ltd. as on March 31, 2005 and 2006:
Liabilities As on 31.3.2005
(Rs.)
As on 31.3.2006
(Rs,.)
Equity share capital 10,00,000 12,50,000
Capital Reserve --- 10,000
General Reserve 2,50,000 3,00,000
Profit and Loss A/c 1,50,000 1,80,000
Long-term loan from the Bank 5,00,000 4,00,000
Sundry Creditors 5,00,000 4,00,000
Provision for Taxation 50,000 60,000
Proposed Dividends 1,00,000 1,25,000
25,50,000 27,25,000

Assets Year
2005
(Rs.)
Year
2006
(Rs.)
Land and Building 5,00,000 4,80,000
Machinery 7,50,000 9,20,000
Investment 1,00,000 50,000
Stock 3,00,000 2,80,000
Sundry Debtors 4,00,000 4,20,000
Cash in Hand 2,00,000 1,65,000
Cash at Bank 3,00,000 4,10,000
25,50,000 27,25,000
Additional Information:
(i) Dividend of Rs.1,00,000 was paid during the year ended March 31, 2006.
(ii) Machinery during the year purchased for Rs.1,25,000.
(iii) Machinery of another company was purchased for a consideration of Rs.1,00,000 payable in equity
shares.
(iv) Income-tax provided during the year Rs.55,000.
13.34
(v) Company sold some investment at a profit of Rs.10,000, which was credited to Capital reserve.
(vi) There was no sale of machinery during the year.
(vii) Depreciation written off on Land and Building Rs.20,000.
From the above particulars, prepare a cash flow statement for the year ended March, 2006 as per AS 3
(Indirect method). (16 Marks) (PE-II - Nov. 2006)
Answer
Cash FlowStatement for the year ending on March 31, 2006
Rs. Rs.
I. Cash flows fromOperating Activities
Net profit made during the year (W.N.1) 2,60,000
Adjustment for depreciation on Machinery (W.N.2) 55,000
Adjustment for depreciation on Land & Building 20,000
Operating profit before change in Working Capital 3,35,000
Decrease in Stock 20,000
Increase in Sundry Debtors (20,000)
Decrease in Sundry Creditors (1,00,000)
Income-tax paid (45,000)
Net cash from operating activities 1,90,000
II. Cash flows fromInvesting Activities
Purchase on Machinery (1,25,000)
Sale of Investments 60,000 (65,000)
III. Cash flows fromFinancing Activities
Issue of equity shares (2,50,000-1,00,000) 1,50,000
Repayment of Long term loan (1,00,000)
Dividend paid (1,00,000) (50,000)
Net increase in cash and cash equivalent 75,000
Cash and cash equivalents at the beginning of the period 5,00,000
Cash and cash equivalents at the end of the period 5,75,000


13.35
Working Notes:
(i) Net Profit made during the year ended 31.3.2006
Increase in P & L (Cr.) Balance 30,000
Add: Transfer to general reserve 50,000
Add: Provision for taxation made during the year 55,000
Add: Provided for proposed dividend during the year 1,25,000
2,60,000
(ii) Machinery Account
Rs. Rs.
To Balance b/d 7,50,000 By Depreciation
(Bal. Fig.)
55,000
To Bank 1,25,000 By Balance c/d 9,20,000
To Equity share capital 1,00,000
9,75,000 9,75,000
(iii) Provision for Taxation Account
Rs. Rs.
To Cash (Bal. Fig.) 45,000 By Balance b/d 50,000
To Balance c/d 60,000 By P & L A/c 55,000
1,05,000 1,05,000
(iv) Proposed Dividend Account
Rs. Rs.
To Bank 1,00,000 By Balance b/d 1,00,000
To Balance c/d 1,25,000 By P & L A/c (Bal. Fig.) 1,25,000
2,25,000 2,25,000
(v) Investment Account
Rs. Rs.
To Balance b/d 1,00,000 By Bank A/c 60,000
To Capital Reserve A/c (Profit
on sale of investment)

10,000
(Balancing figure for
investment sold)

By Balance c/d 50,000
1,10,000 1,10,000

PCC/IPCC_Accounts_theory_______________________________________1
MEC/CEC,CA/CWA&B.Com By Mattupalli Associates for Master Minds
THE WAY OF ASKING THEORY QUESTIONS
IN EXAMINATION POINT OF VIEW FOR PCC /IPCC
FUNDAMENTALS OF ACCOUNTING:
1. What are Fundamental Accounting Assumptions? Write short notes on them.
Ans: The Fundamental Accounting Assumptions are
a. Going Concern: The enterprise is normally viewed as a going concern, that is as continuing
in operation for the foreseeable future. It is assumed that the enterprise has neither the
intention nor the necessity of liquidation or of curtailing materially the scale of operation.
b. Consistency: It is assumed that accounting policies are consistent from one period to another.
c. Accrual: Revenues and Costs are accrued, that is recognized as they are earned or incurred
and recorded in the Financial Statements of the periods to which they relate.
AVERAGE DUE DATE:
1. What is ADD, areas where ADD is calculated?
Ans: Definition: Average Due Date (ADD) is an equated date of payment on which a single
payment can be made in lieu of several payments due for payment on different dates, without
loss of interest to either party. Thus, ADD is the Arithmetic Average of various payments.
Areas where ADD method is followed:
a. For calculation of ADD when various payments are due on different dates and single payment
is to be made by debtor. (Including the settlement of various bills due on different dates).
b. For calculating Interest on drawings made by partners on different dates.
c. For settlement of Contra Accounts. E.g. X & Y sells goods to each other on different dates.
d. For calculation of ADD when amount is lent by the creditor in one installment and
repayment of the amount lent is to be made in various installments.
ACCOUNT CURRENT:
1. What is an Account Current? Explain Briefly.
Ans: Account Current: Account current is a statement in the debit and credit form i.e., in the
ledger form recording the transactions between the two parties in a chronological order or time
sequence order. It is the copy of the accounts appearing in the books of sender with an
additional column for interest. It is sent by one party to another usually by the agent to his
principal or by the banker to his client.
An account current bears the following characteristics:
a. It is an ordinary form of ledger account.
b. The transactions are arranged in a sequential manner.
c. There is an additional column of interest on each side of the account.
d. It is the copy of accounts of one party in the books of another party.
e. Any of the two parties can prepare this account.
f. The interest columns are purely on the memorandum basis and are not a part of double entry.
2. What is meant by Red Ink Interest?
Ans: Sometimes, the due date of the transactions fall beyond the date of settlement i.e., the
date on which the account is prepared, in such cases, the days are counted from the settlement
date to the date of transaction. These days are written with a negative sign in the days column
or with a positive sign on the opposite side where the transaction does not appears. The
PCC/IPCC_Accounts_theory_______________________________________2
Ph: 08632242355 www.gntmasterminds.com
products were written with Red Ink. So the interest on such products is called as Red Ink
Interest. This Red Ink Interest is treated as a negative interest.
3. What are various methods of accounting in an account current?
Ans: There are three ways of preparing an Account Current:
a. Interest Tables method
b. The Method of Products
c. The Method of product of balances.
a. Interest Tables method:
i. Format: According to this method, all the transactions are arranged in the form of an
account. There are two additional columns on both the sides of such an account.
One Column is meant to indicate the number of days counted from the due date of
each transaction to the date of rendering the account. If no specific date is mentioned
as the date on which payment is due, the date of the transactions is presumed to be
the due date.
The other column is meant for writing interest.
ii. Calculation of Interest: With the help of ready made tables (Simple Interest Tables),
interest due on different amounts at given rates for different periods of time is found out
and this is entered against each item separately. The interest columns of both the sides
are totaled up and the balance is drawn.
b. The Method of Products:
i. Format: The method of preparing the Account Current is the same as in the method of
interest tables, the method of calculating the interest alone remains the same but for
minor modifications.
ii. Product Column: In this method the Product Column replaces the Interest Column of
the Interest Method. The product is obtained by multiplying the amount of the
transaction by the number of days from the Due Date to the date of the Account. The
Product column is then balanced so as to ascertain the figure on which the interest (net)
is to be calculated.
iii. Calculation of Interest: The interest is then calculated for a single day on the balance
in the product column and is posted to the Account Current on the side opposite to the
side where the Product Balance stands.
iv. Net Interest: This method calculates the net interest directly, i.e., interest payable and
interest due are mutually set off and only the net interest due/receivable is reflected in
the Account Current. Interest on the individual transactions is not reflected as in the
Interest Method.
v. Procedure: Steps involved in this method is summarized as follows:
Find out the balance of the products as in point (iii).
Calculate Interest at the given rate on the balance of the products for a single day,
Enter the interest on the appropriate side in the amount column. This entry is made
on the side opposite to the side on which the balance of product appears.
c. The Method of product of balances:
i. Meaning: This method is also known as periodic balance method and is usually adopted
in the case of banks where the balance of the account is taken out after every
transaction.
ii. Format: The format of preparation of Account Current is as given below.
Date Particulars Dr. Cr. Nature of
Balance (Dr./Cr.)
Balance Amount Days Dr.
Product
Cr.
Product
PCC/IPCC_Accounts_theory_______________________________________3
MEC/CEC,CA/CWA&B.Com By Mattupalli Associates for Master Minds
iii. Purpose of Columns: The usage of the date and the particulars column are to denote the
date and the details of the transactions. The other columns are for the following purposes:
The debits/credits to the account are entered in the Dr./Cr. Columns respectively.
The Nature of the balance i.e. whether the account has a Dr./Cr. balance is reflected
the Nature of Balance Column.
The Amount of balance is reflected in the Balance Amount Column.
In the Days Column the no. of days from the date of the transaction to that of the
next transactions in recorded.
Dr. Product/ Cr. Product consists of the product of the Dr. /Cr. column and the
Days column.
iv. Calculation of Interest: The Interest is calculated as under:
The Columns are filled up to the date of the account as specified in point (c).
The total of the Dr. Product and the Cr. Product Columns are arrived at.
The interest for one day is calculated for the Dr./Cr. Products, at the appropriate
rates and netted off to arrive at the net interest.
v. Posting of Interest: The interest amount is posted as follows: If the Dr. Product is
greater than the Cr. Product the interest is posted to the Debit of the account, else it is
posted to the Credit of the Account.
STATUTORY REPORT:
1. Write short note on Contents of Statutory report?
Ans: Contents of Statutory Report [Sec. 165 (3)]
a. The total number of shares allotted, distinguishing shares allotted as fully paid or partly
paid, otherwise than in cash and in case of partly paid up share stating the extent to which
they are so paid up and in both cases the consideration for which they have been allotted.
b. The total amount of cash received in respect of shares allotted for cash.
c. An abstract of the Receipts and Payment A/c up to date within 7 days of the report, showing
under the different headings the receipts of the company from shares, debentures and other
sources, payments made, balance in hand and an account or estimate of the preliminary
expenses of the company, showing separately any commission or discount paid, or to be
paid on the issue or sale of shares or debentures.
d. The names, addresses and occupations of the director, auditor, manager and secretary and
the change if any.
e. The particulars of any contract which, or the modification or the proposed modification of
which, is to be submitted to the meeting for its approval together in the latter case with the
particulars of the modification or the proposed modification.
f. The extent, if any, to which each underwriting contract has not been carried out and the
reasons thereof.
g. The callinarrears from director and manager of the company.
h. The particulars of any commission or brokerage paid or payable in connection with the issue
or sale of shares and debentures to any director or manager.
DEPARTMENTAL ACCOUNTS:
1. Write short notes on Basis of allocation of common expenditure among different
departments?
Ans: Apportionment of Common Expenses:
No. Items of Expenses and Income Basis of Apportionment
a. Salesmens commission, discounts allowed Sales (turnover) of each
PCC/IPCC_Accounts_theory_______________________________________4
Ph: 08632242355 www.gntmasterminds.com
(including provision for such discounts),
bad debts, carriage outwards,
advertisement, packing expenses etc.
department.
b. Discounts received (including provision for
such discounts) etc.
Purchases of each
department
c. Rent, rates and taxes, repairs and
maintenance of building etc.
Floor space of each
department
d. Depreciation of assets, fire insurance
premium etc.
Asset values of each
department
e. Workmens compensation insurance,
employers contribution to Employees State
Insurance etc.
Wages of each department
f. Canteen expenses, medical benefits, safety
measures and such other labour welfare
expenses etc.
No. of workers of each
department.
Notes:
a. Expenses incurred for the direct benefit of a particular department should be allocated
to the department concerned. E.g.: Special Advertisement, Insurance of Stock,
Departmental Salaries.
b. If expenses incurred for the benefit of more than one department are not capable of accurate
measurement, should be distributed on arbitrary basis (i.e., either in turnover ratio or in the
cost of goods sold etc.) E.G., Salary paid to the General Manager, Expenses of Accounts
dept. etc.
c. Expenses which cannot be apportioned satisfactorily should be left intact and finally to be
debited to General P & L A/c.
FINANCIAL STATEMENT OF NOT FOR PROFIT ORGANISATION:
1. Explain the accounting treatment of donation received for specific purpose in the case
of charitable society?
Ans:
a. Donation may have been raised either for meeting some revenue or capital expenditure.
b. When expenditure intended for the revenue purpose are credited directly to the Income and
Expenditure Account but others, if the donors have declared their specific intention, are
credited to special fund account and in the absence thereof, to the Capital Fund Account.
c. When any investments are purchased out of a special fund or an asset is acquired there
from, these are disclosed separately.
d. Any income received from such investments or any donations collected for a special purpose
are credited to an account indicating the purpose and correspondingly the expenditure
incurred in carrying out the purpose of the fund is debited to this account. In such case
expenses are not charged to Income & Expenditure Account.
e. The term Fund is strictly applicable to the amounts collected for a special purpose when
these are invested, e.g. scholarship fund, recreation fund etc
f. Other wise, when the amounts collected are not invested in securities or assets
distinguishable from those belonging to the institution, the word Account is more
appropriate e.g. Building Account, Sports Goods Account etc.
UNDER WRITING:
1. What are the terms used in under writing?
Ans:
a. Marked Applications:
PCC/IPCC_Accounts_theory_______________________________________5
MEC/CEC,CA/CWA&B.Com By Mattupalli Associates for Master Minds
i. The application forms bearing the stamp of the underwriter are termed as Marked
Application forms.
ii. The benefit of marked applications is given to the concerned underwriters in whose
favour application forms have been marked.
b. Unmarked Applications:
i. The application forms which do not bear the stamp of the underwriter are termed as
Unmarked Application forms.
ii. The benefit of unmarked applications is given first to the company to the extent of issue
not underwritten by underwriters (in case any part of the issue is not underwritten).
iii. In case there is surplus, the benefit of surplus unmarked applications will be given to
the underwriters in the ratio of their gross liability.
c. Full Underwriting:
i. When the entire issue is underwritten such underwriting is termed as full underwriting.
For example, X Ltd. decided to make a public issue of 1,00,000 Equity Shares of Rs. 10
each which is entirely underwritten by A,B,C and D in the ratio of 2:2:1:1.
ii. In such a case the benefit of unmarked applications is given to the underwriter in the
ratio of their gross liability i.e., 2:2:1:1.
d. Partial Underwriting:
i. When only a part of issue is underwritten, such underwriting is termed as Partial
Underwriting. For example, X Ltd., decided to make a public issue of 1,00,000 Equity
Shares of Rs. 10 each out of which 90,000 shares are underwritten by A, B, C and D in
the ratio of 2:2:1:1. It means 10,000 shares are underwritten by the company itself.
ii. In such a case, the benefit of unmarked applications will first be given to the company.
iii. In case there is surplus, such surplus will be distributed among other underwriters in
the ratio of their gross liability.
e. Sole Underwriting:
i. When the issue is underwritten only by one underwriter, such underwriting is termed as
Sole Underwriting. For example, if an issue of 1,00,000 shares of Rs. 10 each of X Ltd.,
is underwritten by A, it is a case of sole underwriting.
ii. In such a case, the distinction between marked and unmarked applications is not of
such significance.
f. Joint Underwriting:
i. When the issue is underwritten by two or more underwriters, such underwriting is termed
as Joint Underwriting. For Example, if an issue of 1,00,000 shares of Rs. 10 each of X Ltd.,
is underwritten by A, B, C and D in the ratio 2:2:1:1, it is a case of joint underwriting.
ii. In such a case the benefit of unmarked applications is given to the underwriters in the
ratio of their gross liability.
iii. The benefit of marked applications is given to the concerned underwriters in whose
favour applications have been marked.
g. Firm Underwriting:
i. Meaning: Firm underwriting refers to a definite commitment by the underwriter to take
up a specified number of securities irrespective of the number of securities subscribed
by the public.
For example, the entire issue of X Ltd., is underwritten as follows:
A. 1,60,000 shares (firm underwriting 3,600 shares)
B. 1,60,000 shares (firm underwriting 2,000 shares)
PCC/IPCC_Accounts_theory_______________________________________6
Ph: 08632242355 www.gntmasterminds.com
C. 80,000 shares (firm underwriting 1,200 shares)
D. 80,000 shares (firm underwriting 10,000 shares)
In this case only 4,63,200 shares (i.e., 4,80,000 shares firm underwriting of 16,800
shares) will be offered to public and 16,800 shares will be taken up by the underwriters
even if the issue is oversubscribed.
ii. Treatment: The benefit of firm underwriting may be given either.
To an individual underwriter on the basis of his individual firm underwriting, or
To all the underwriters in the ratio of their gross liability
In other words, firm underwriting shares may be treated at par with either Marked
Applications or Unmarked Applications.
2. Write a short note on firm underwriting and partial underwriting along with firm
underwriting?
Ans: In firm underwriting the underwriter decides to subscribe upto a certain number of shares
/debentures irrespective of the nature of public response to issue of securities. He gets these
securities even if the issue is fully subscribed or oversubscribed. These securities are taken
over by the underwriter in addition to his liability for securities not subscribed by the public.
Under partial underwriting along with firm underwriting, unless otherwise agreed, individual
underwriters does not gets the advantage of firm underwriting in determination of number of
shares/debentures to be taken up by him.
3. Pass the accounting entries relating to firm underwriting in the books of Company
and Underwriter.
Ans:
Entries in the books of AB Co.Ltd (Company)
Particulars Dr.
(Rs.)
Cr.
Rs.)
a. As Account Dr.
To Equity Share Capital Account.
(Being allotment of underwritten equity shares in pursuance
of firm underwriting contract, vide Boards resolution).
XXX
XXX
b. Underwriting commission on issue of Shares Account Dr.
To As Account
(Being underwriting commission due to the underwriter
under the firm underwriting contract)
XXX
XXX
c. Bank Account Dr.
To As Account
(Being money received in full settlement of
account from under writer)
XXX
XXX
Entries in the books of A (Underwriter)
Particulars Dr.
(Rs.)
Cr.
(Rs.)
a. Underwriting Account Dr.
To AB Co. Ltd Account
(Being the liability to take up necessary number of
shares of the company in pursuance of firm under
writing contract recorded)
XXX
XXX
b. AB Co. Ltd Account Dr.
To Underwriting Account
(Being underwriting commission income credited
to underwriting A/c)
XXX
XXX
PCC/IPCC_Accounts_theory_______________________________________7
MEC/CEC,CA/CWA&B.Com By Mattupalli Associates for Master Minds
c. AB Co. Ltd Account Dr.
To Bank Account
(Being balance money paid to the co. in full settlement
of account)
XXX
XXX
SELF BALANCING LEDGERS (SBS):
1. What are the advantages of Selfbalancing Ledger system?
Ans: Advantages of Selfbalancing ledger:
a. A number of book keepers can work on different ledgers.
b. Arithmetic accuracy of each ledger can be proved independently.
c. Each ledger is of a suitable size.
d. A complete trial balance can be prepared without balancing subsidiary ledgers, thus
facilitating the quick assessment.
e. Since error can be localized, delay in detection is minimized, there by saving labour and
time of the book keepers.
LIQUIDATION OF COMPANIES:
1. Write short notes on Overriding Preferential Creditors?
Ans: Overriding Preferential payment (Section 529A): This section gives priority in payment
to workmens dues and debts due to secured creditors to the extent they could not be paid
because of the former ranking pari passu with the later.
Example: The following details have been extracted from the books of a company at the time of
the liquidation:
Secured creditors (with assets charged in their favour Rs.2,00,000)
Workmens dues
Preferential creditors (excluding workmens dues)
Unsecured creditors
Other assets
3,00,000
1,00,000
50,000
2,00,000
2,50,000
The assets available will be used as follows:
a. Assets charged in favour of secured creditors worth Rs.2,00,000 will be shared by Secured
Creditors and workers in the ratio of 3:1.
i. Share of secured creditors 2,00,000 x = 1,50,000
ii. Share of workers 2,00,000 x = 50,000
b. Overriding preferential payments amount to:
50,000
50,000
Secured creditors to the extent of their security
being used for workmens dues:
Balance of workmens dues (1,00,000 50,000)
1,00,000
c. Other Assets will be used as follows:
1,00,000
50,000
1,00,000
Overriding preferential payments
Preferential creditors
Unsecured creditors
2,50,000
2. What are the contents of Liquidators Statement of Account? How frequently does a
liquidator has to submit such statement?
Ans: It is a final statement A/c that is to be submitted by official Liquidator/Liquidator to the
court/members creditors as the case may be in the event the company is finally being wound up.
PCC/IPCC_Accounts_theory_______________________________________8
Ph: 08632242355 www.gntmasterminds.com
PROFORMA
Name of the company
Nature of Winding Up voluntary/ compulsory
Liquidator Final A/c
[In pursuance of sec. 497 & 509 of the companies Act]
Receipts Rs. Payments Rs.
XXX
XXX
XXX
XXX
XXX
XXX
XXX
XXX
XXX
XXX
XXX
To Realisation from sale of Assets
[not specifically pledged]
To Realisation from Assets
specifically pledged XXX
Less: amount paid to
Secured creditors XXX
To Receipts from contributory
(to the extent of uncalled capital)
XXX
By Legal Charges
By Liquidator remuneration
By Expenses of Liquidation
By Amt. paid to Debenture holders
By Preferential Creditors
By Unsecured Creditors
By Preference Share Holders
(at the rate of per share)
By Equity Share Holders
(at the rate of per share)
XXX
Liquidators statement of account of the winding up is prepared for the period starting from the
commencement of winding up to the close of winding up. If winding up process is not completed
within one year after its commencement, Liquidators Statement of account pursuant to Section
551 of the Companies Act, 1956 (Form No.153) is to be filed by a Liquidator within a period of
two months of the conclusion of one year and thereafter on interval of six months.
3. What is meant by B list of contributories? What is the liability of contributories
included in this list?
Ans: B list Contributories: The shareholders who transferred partly paid shares (otherwise
than by operation of law or by death) within one year, before the date of winding up may be
called upon to pay an amount (not exceeding the amount not called up when the shares were
transferred) to pay off such creditors as existed on the date of transfer of shares.
a. When the existing assets available with the liquidator are not sufficient to cover the
liabilities.
b. When the existing shareholders fail to pay the amount due on the shares to the liquidator.
AMALGAMATION OF COMPANIES:
1. What are the two main methods of accounting amalgamation of Companies?
Ans: Method of accounting of amalgamation:
a. Purchase method
b. Pooling of interest method
2. Distinguish between Pooling of interest method and purchase method of recording
transactions relating to amalgamation.
Ans:
Sl.
No
Basis Pooling of interest method Purchase method of recording
transaction
a. Applicability The pooling of interest method
is applied in case of an
amalgamation in the nature of
merger.
Purchase method is applied in
the case of an amalgamation in
the nature of purchase.
b. Recording In the pooling of interest
method all the reserves of the
transferor Co. are also recorded
by the transferee Co. in its
books of account
In the purchase method the
transferee Co. records in its
books of accounts only the
assets and liabilities taken over
the reserves, except the statutory
reserves of the transferor
company are not aggregated with
PCC/IPCC_Accounts_theory_______________________________________9
MEC/CEC,CA/CWA&B.Com By Mattupalli Associates for Master Minds
those of the transferee Co.
c. Adjustment
of the
differences
Under the pooling of interest
method, the difference between
the consideration paid and the
share capital of the transferor
company is adjusted in the
general reserve or other
reserves of the transferee
company.
Under the purchase method, the
difference between the
consideration and net assets
taken over is treated by the
transferee company as goodwill
or capital reserve.
d. Statutory
reserves
In this method, the statutory
reserves are recorded by the
transferee co. like all other
reserves without opening
Amalgamation and Adjustment
A/c.
In the purchase method, while
incorporating the statutory
reserves, the transferee Co. has
to open amalgamation
adjustment account debiting it
with the amt. of the statutory
reserves being incorporated.
3. What are the conditions that are to be satisfied for Amalgamation in the nature of
Merger in an Amalgamation?
Ans: According to AS14 on Accounting for Amalgamation, the following conditions must be
satisfied for an amalgamation in the nature of merger:
a. After amalgamation, all the assets and liabilities of the transferor company becomes the
assets and liabilities of the transferee company.
b. Shareholders holding not less than 90% of the face value of the equity shares of the
transferor company becomes the equity shareholders of the transferee company by virtue of
amalgamation.
c. The business of the transferor company is intended to be carried on after the amalgamation
by the transferee company.
d. Purchase consideration should be discharged only by issue of equity shares in the transferee
company except that cash may be paid in respect of any fractional shares.
e. No adjustments are required to be made in the book values of the assets and liabilities of
the transferor company, when they are incorporated in the financial statements of the
transferee company.
If any one of the condition is not satisfied in a process of amalgamation, it will not be
considered as amalgamation in the nature of merger.
4. What are the methods for calculating purchase consideration?
Ans: Different methods in computing the Purchase Consideration.
a. Lumpsum Method: Under this method purchase consideration will be paid in lump sum as
per the valuation of purchasing companies valuationer.
E.g., If it is stated that A Ltd. takes over the business of B Ltd. for Rs.15,00,000 here the
sum of the Rs.15,00,000 is the Purchase Consideration.
b. Net Assets Method: Under this method P.C. shall be computed as follows:
Particulars Rs.
XXX
XXX
Agreed value of assets taken over
Less: Agreed value of Liabilities taken over
Purchase Consideration XXX
Note:
i. The term agreed value means the amount at which the transferor company has
agreed to sell and the transferee company has agreed to take over a particular assets
or a liability Otherwise book value will be the agreed value.
ii. Fictitious assets (i.e., preliminary expenses, underwriting commission, discount on
issue of shares, discount on issue of debentures and debit balance in P & L A/c) are
not taken over.
PCC/IPCC_Accounts_theory_______________________________________10
Ph: 08632242355 www.gntmasterminds.com
c. Payment Method: Under this method P.C. should be calculated by aggregating total
payments made by the purchasing company.
E.g.: A Ltd. had taken over B Ltd. and for that it agreed to pay Rs.5,00,000 in cash 4,00,000
Equity Shares of Rs.10 each fully paid at an agreed value of Rs.15 per share then the P.C.
will be ascertained as follows:
Particulars Rs.
5,00,000
60,00,000
Cash
4,00,000 E. Shares of Rs.10 each fully paid,
at Rs.15 per share
Purchase Consideration 65,00,000
Note: A modified method of indicating consideration is to say how much a shareholder get
per share on the transfer of the companys business to transferee company.
5. Distinguish between Amalgamation in the nature of purchase and Amalgamation in
the nature of merger?
Ans:
Basis of
Distinction
Amalgamation in Nature
of Merger
Amalgamation in Nature of
Purchase
a. T/f of
Assets &
Liabilities
There is transfer of all
assets & liabilities.
There need not be transfer for
all assets & liabilities.
b. Equity
Shareholder
s holding
90%
Equity shareholders
holding 90% equity shares
in transferor company
become shareholders of
transferee company.
Equity shareholders, need not
become shareholders of
transferee company.
c. Purchase
Considerati
on
Purchase consideration is
discharged wholly by issue
of equity shares (except
cash for fractional shares)
Purchase consideration need
not be discharged wholly by
issue of equity shares.
d. Same
Business
The same business of the
transferor company is
intended to be carried on by
the transferee company.
The business of the transferor
company need not be intended
to be carried on by the
transferee company.
e. Recording
of Assets &
Liabilities
The assets & liabilities
taken over are recorded at
their existing carrying
amounts except where
adjustment is required to
ensure uniformity of
accounting policies.
The assets & liabilities taken
over are recorded at their
existing carrying amounts or
the basis of their fair values.
f. Recording
of Reserves
of
Transferor
Co.
All reserves are recorded at
their existing carrying
amounts and in the same
form.
(a) Only statutory reserves
are recorded at their
existing carrying amounts
as follows:
Amalgamation adjustment
A/c Dr.
To Statutory Reserve A/c
(b) Other reserves are not
recorded at all.
g. Recording
of Balance
of Profit &
Loss A/c of
Transferor
The balance of P&L A/c
should be aggregated with
the corresponding balance
of the transferee co. or
transferred to the General
The balance of P&L A/c losses
its identify and is not
recorded at all.
PCC/IPCC_Accounts_theory_______________________________________11
MEC/CEC,CA/CWA&B.Com By Mattupalli Associates for Master Minds
Co. Reserve.
h. Difference
between the
Purchase
Considerati
on and
Share
Capital/Net
Assets of
transfer co.
The excess of the purchase
consideration over the
share capital of transferor
company is debited to
Reserves and the excess of
share capital over purchase
consideration is credited
to reserves.
The excess of purchase
consideration over the net
assets is treated as Goodwill
and the excess of net assets
over purchase consideration is
treated as Capital Reserve.
HIRE PURCHASE:
1. Pass Accounting Entries for Repossessed goods under HP system of Debtors method.?
Ans: The following entries will be made in respect of repossessed goods.
1. For goods repossessed on default of Purchaser.
Goods repossessed A/c Dr.
To HP Debtor A/c (installment due)
To HP Stock A/c (installment not due)
2. For difference between installment the unpaid and value of
repossessed goods or loading amount only.
In Case of Loss:
HP Adjustment A/c Dr.
To Goods Repossessed A/c
In Case of Profit:
The above entry will be reversed
3. For sale of goods repossessed
Bank A/c Dr.
To Goods repossessed A/c
4. For profit on sale of goods repossessed.
Goods repossessed A/c Dr.
To H.P. Adjustment A/c
(in case of loss entry will be reversed)
ISSUE & REDEMPTION OF DEBENTURES:
1. State the guidelines of SEBI regarding issue of convertible debentures for disclosure
and investor protection?
Ans: SEBI has prescribed the following guidelines for the issue of convertible debentures for
disclosure and investor protections:
a. Issue of fully Convertible Debentures, having a conversion period of more than 36 months
will not be permissible, unless the conversion is made optional with put and call option.
b. Premium amount on conversion, time of conversion, in stages, if any shall be determined in
advance and must be stated in the prospectus. Interest rates for the above debentures will
be fixed by the issue.
c. Any public or right issue of debt instruments shall have to be compulsorily rated by the
approved credit rating agencies, irrespective of their maturity or conversion period.
d. Any conversion in part or whole of the debenture will be optional at the hands of the
debentureholders, if the alteration takes place at or after 18 months, from the date of
allotment, but before 36 months.
e. Issue of debentures with maturity of 18 months or less are exempted from the necessity of
appointment of Debenture Trustee or creating a Debenture Redemption Reserve.
PCC/IPCC_Accounts_theory_______________________________________12
Ph: 08632242355 www.gntmasterminds.com
f. Premium amount at the time of conversion for the Partly Convertible Debenture (PCD)
should also be determined in advance and must be stated in the prospectus. Redemption
amount, period of maturity, yield on redemption for the PCDs or NCDs must be shown in
the prospectus.
g. The discount on the nonconvertible portion of the PCDs in case they are traded and
procedure for their purchase on spot trading basis must be disclosed in the prospectus.
h. Before roll over of any NCDs or nonconvertible portion of the PCDs fresh credit rating shall
be optioned within a period of six months to the due date of redemption, and communicated
to debenture holder before roll over and fresh trust deed shall be made.
i. Letter of information regarding all over should be vetted by SEBI with regard to the credit
rating, debenture holder resolution, option for conversion and such other item which SEBI
may prescribe from time to time.
j. In case, the nonconvertible portions of PCDs or NCDs are to be rolled over with or without
change in the interest rate, a compulsory option should be given to those debenture holders
who want to withdraw and in case from the debenture programme.
k. SEBI may prescribe additional disclosure requirement from time to time after due notice.
PARTNERSHIP ACCOUNTS:
1. Explain Garner Vs. Murray Rule applicable in case of partnership firms?
Ans:
a. Loss of Insolvent Partner: If upon dissolution, a Partner has become insolvent and the
debit Balance in his Capital Account is remaining unpaid, such loss should be borne by the
Solvent Partners in the proportion of their Capital at the date of dissolution.
b. Capital: The Capital for this purpose is the balance in Partners Capital Account before
adjusting therein the amount of profit or loss on the realization of assets.
c. Making Good the Realisation Loss: The loss on the realization should be contributed in
cash by the solvent partners and the profit, will not be taken into account for determining the
proportion in which the loss of insolvent partner should be borne by the solvent partners.
2. How to calculate Interest on Deceased partners loan as per Sec. 37 of Indian
Partnership Act?
Ans: Disposal of the amount due to the Retiring on Deceased Partner: In the absence of an
agreement in this regard, the outgoing partner at his option is entitled to receive either interest
@ 6% p.a. till the amount is paid off or a share of the profit which has been earned by using the
amount due to him. [Sec. 37] Which ever is lower.
Application on Section 37 of Indian Partnership Act, 1932
For e.g.: A, B & C were partners sharing profits and losses in the ratio of 2:2:1. C retired on 1
st
July, 2003 on which date the capitals of A, B and C after all necessary adjustments stood at Rs.
73,875, Rs. 63,875 and Rs. 42,250 respectively. A and B continued to carry on the business for
six months without settling the A/c of C. During the period of six months from 01.07.2003, a
profit of Rs. 20,500 is earned by the use of the firms property. State which of the two options
available under section 37 of the Indian Partnership Act, 1932 should be exercised by C.
Solution:
a. Share in the subsequent profits attributable to the use of his property
= Rs. 42,250 x 2,500 = Rs. 4,812
Rs. 1,80,000
b. Interest @ 6% p.a. on the use of his property = Rs. 42,250 x 6 x 6 = Rs. 1,267.50
12 100
C should exercise option. Since the amount payable to him under this option is more as
compared to the amount payable to him under option.
PCC/IPCC_Accounts_theory_______________________________________13
MEC/CEC,CA/CWA&B.Com By Mattupalli Associates for Master Minds
BANK ACCOUNTS:
1. Write shorts notes on Slip system of posting and double voucher system.
Ans: Slip system of posting: This system is used in the case of banking companies, in this
entries in the personal ledgers are made directly from vouchers, instead of being posted from
day book. Payinslips and cheques are used as slips. This become the basis of most of the
transactions directly recorded in the account of the customers. Lots of time and labour of the
bank staff is saved, because the slips are filled in by the customers themselves. The vouchers
entered into different personal ledgers are summarized on summary sheets every day, total of
which are posted to the different control accounts which are maintained in general ledger.
Double Voucher System: For the transaction not involving cash, two vouchers are prepared by
the bank one debit voucher and the other credit voucher. This system of maintaining two
vouchers are called double voucher system. The vouchers are sent to different clerks who make
entries in book under their charge.
2. Write shorts notes on Acceptances and Endorsements?
Ans:
a. A bank has a more acceptable credit as compared to that of its customers. On the basis of
this, it is often called upon to, accept or endorse bills on behalf of its customers.
b. In such a case, the bank undertakes a liability towards the party which agrees to receive a
bill in payment of a debt or agreed to discount the bill after the same has been accepted by
the bank.
c. As against this liability, the bank has a correspondence claim against the customer on whose
behalf it has undertaken to be a party to the bill, either as an acceptor or as an endorser.
d. The liabilities which are outstanding at the end of the year, and the corresponding assets
are disclosed as contingent liability in the financial statement.
e. To be on the safer side, usually the bank asks the customers to deposit a security equivalent
to the amount of the bill accepted on his behalf.
f. A record of the particulars of the bills accepted as well as of the securities collected from the
customers is kept in the Bill Accepted Register.
g. A bank may not treat this book as part of system of its accounts.
h. In these cases, no further record of the transaction is kept until the bill matures for payment.
i. If the bill, at the end of its term, has to be retired by the bank, and the amount cannot be
collected from the customer on demand. The bank reimburses itself by disposing of the
security deposited by the customer.
3. Write a short notes on Assets Classification borrowerwise in Bank Accounts?
Ans: The classification of advances as performing and nonperforming and is borrowerwise and
nonaccount wise. If one of the accounts of the borrower is nonperforming (NPA), then other
accounts which are otherwise performing, have to be classified as NPA only. The Reserve Bank of
India has made clear that for purpose of classification as NPA, the most adverse category in any
borrowers account should be adopted as a prudential measure.
For example, if any borrower has three days of facilities, one of which is classified as standard,
second substandard and the third is classified as doubtful., all the outstandings in the said
borrowers account should be classified as doubtful assets under prudential norms.
4. What do you mean by Standard assets, Substandard assets and Doubtful assets of a
bank?
Ans:
a. Standard Assets: Standard Assets are those assets which does not show any problem and
also does not carry more than normal risk attached to the business. They are not non
performing assets.
PCC/IPCC_Accounts_theory_______________________________________14
Ph: 08632242355 www.gntmasterminds.com
The provision of 0.25% is made on these standard assets as a matter of abundant caution,
thought there could be no risk of nonrecovery/default.
b. Substandard Assets: These are the assets which are classified as nonperforming assets
for a period not exceeding eighteen months. These assets have a well defined credit
weaknesses that prejuicide the liquidation of the debt and are characterized by the distinct
possibility that the bank will bear some loss, if deficiency are not corrected.
A provision of 10% of total outstanding should be made on such assets.
c. Doubtful Assets: The assets which remained a nonperforming assets for a period exceeding
18 months are called doubtful assets. These advances are so weak that its collection in full
is difficult. Depending on the age of the doubtful assets and the security, various percentage
of provisioning is recommended for doubtful assets.
5. What is the % of NPA provision to be made by banks in respect of fully secured
doubtful advances of more than 3 years old?
Ans: % of NPA provision for doubtful of advance of more than 3 years is 100%.
ELECTRICITY COMPANIES:
1. Write a short notes on Disposal of Surplus in Electricity Company?
Ans: Disposal of Surplus: The law intends to prevent an electricity undertaking from earning
unreasonable profits. That is why, it has defined Reasonable Return, Clear Profit, Capital Base.
Excess of Clear Profit over Reasonable Return is called as Surplus required to be credited to
Customers Rebate Reserve and Surplus upto 20% of Reasonable Return is to be disposed off as
follows:
a. 1/3
rd
of such surplus not exceeding 5% of Reasonable Return at the disposal of the
Company,
b. of the balance to be credited to Tarrifs, Dividend Control Reserve.
c. of the balance to be credited to Customers Rebate Reserves.
2. Write a short notes on Accounting treatment for replacement of an asset in Electricity
Company?
Ans: Journal Entries:
1. To Record the Total Current Cash Cost of Old Asset:
Asset A/c Dr. [with Current Cash Cost to be
Capitalised (as per Step 2)]
Replacement A/c Dr. [Current Cost of old Asset (as per Step 1)]
To Bank A/c (Current Cash Cost of New Asset)
2. To Record the Value of Old Materials reused:
Asset A/c Dr. (with the given value of old materials reused)
To Replacement A/c
3. To Record the sale proceeds of Old Materials sold:
Bank A/c Dr. (with the given Sale Proceeds of
Old Materials Sold)
To Replacement A/c
4. To Record the amount to be written off to Revenue:
Revenue A/c Dr. [with the amount to be written off to
Revenue A/c (as per Step 3)]
To Replacement A/c
Note: Hence, the total amount capitalized is calculated as follows:
A. Current Cash Cost to be Capitalized xxx
B. Add: Value of Old Materials reused xxx
C. Total amount Capitalized (A + B) xxx
PCC/IPCC_Accounts_theory_______________________________________15
MEC/CEC,CA/CWA&B.Com By Mattupalli Associates for Master Minds
3. Write short notes on Reasonable returns in electricity supply companies.?
Ans: Reasonable return in electricity supply companies:
The law seeks to prevent an electricity undertaking from earning to high a profit. For this
purpose, concept of Reasonable Return has been propounded. Reasonable Return is the
normal return which Electricity Company can earn. The following is the procedure to compute
the Reasonable Return and disposal of any surplus profits earned.
Computation of Reasonable Return:
Particulars Rs.
Yield on Capital Base = Capital Base X Standard Rate of Return (Note)
Add: Income on Investments other than Investment against
Contingencies Reserve
Add: % of Loans advanced by the Electricity Boards
Add: % of amount borrowed from State Government approved
Organisation/Institutions
Add: % of amount raised by the Issue of Debentures
Add: % on balance in Development Reserve
XXX
XXX
XXX
XXX
XXX
XXX
Reasonable Return XXX
Note: Standard Rate = Reserve Bank of India Rate + 2%.
The term Capital Base used above can be identified as:
a. The original cost of fixed assets available for use and necessary for the purpose of the
undertaking less contributions, if any made by the consumers for constructions of service
lines and also amounts written off.
b. The original cost of work in progress.
c. The cost of intangible assets.
d. The amounts of investments compulsory made against contingencies reserve and
e. The monthly average of the stores, materials supplies and cash and bank balances held at
the end of each month of the year of account.
4. Main features of double account system of presentation of financial information in
the case of public utility concern?
Ans: Double accounts system is the name given to the system of preparing the final accounts of
certain statutory companies formed by special Acts of Parliament, usually public utility
undertakings (for example Electricity Companies). This method is not a special method of
keeping accounts but it is a special method of presenting the accounts under the normal double
entry system.
In this system, separate accounts in respect of capital and revenue are prepared in order to
show clearly the capital receipts and the manner in which the amounts thereof have been
invested. The final accounts prepared under the double accounts system normally consists of:
a. Revenue Account.
b. Net Revenue Accounts.
c. Capital Account (Receipts and Expenditure).
d. General Balance Sheet.
The Revenue account is certain to the Profit and Loss Account of a company with some
exception. The Net Revenue Account resembles with appropriation portion of the Profit and Loss
Account of the Company. The Capital Account shows the total amount of capital raised and its
sources and also the manner and the extent to which this capital has been applied in the
acquisition of fixed assets for the purpose of carrying on the business. Other items are included
in the General Balance Sheet.
The Double Account system in its pure form does no longer exist but the statements submitted
to State Government by electricity companies generally follow the principle of double account
system. For presenting accounts to the shareholders, electricity companies normally follow
Schedule VI of the Companies Act, 1956.
PCC/IPCC_Accounts_theory_______________________________________16
Ph: 08632242355 www.gntmasterminds.com
5. Write short notes on Receipt and payment on capital account and General balance
sheet of a public utility?
Ans: Receipt and payment on capital account and General balance sheet of a public Utility:
Under the double accounts system, the balance sheet is split into two parts:
a. Receipts and expenditure (payment) on capital account and
b. General balance sheet.
The main object of the former is to show the total amount of capital raised with its sources and
the way and the extent to which this capital has been applied in the acquisition of fixed assets
for the purpose of carrying on the business of the undertaking. It thus discloses the receipt and
expenditure on capital account, that is the receipt from issue of shares, debentures and loans
and the expenditure out of such receipts on acquisition of and addition of fixed assets. The
receipt and expenditure on capital account is shown in a columnar form:
There are three columns:
a. One showing the amount at the commencement of the period.
b. Another disclosing the amount received or spent during the period.
c. The third showing the balance at the end of the period.
The general balance sheet contains other assets and liabilities and the balance of the receipt
and expenditure on capital account. It is drawn up in the usual way, showing on the liabilities
side, resources, depreciation fund, current liabilities and other credit balances and total of
receipts as per capital accounts, on the assets side total of expenditure as per capital account,
floating assets and other debit balances.
6. What is meant by Clear Profits?
Ans: Clear Profit: It is the difference between the total income and total expenditure including
specific appropriations. The following are the specific appropriations for computing clear profit.
a. Brought Forward Losses of Previous years to the extent permitted by State Government.
b. Taxes on Income and Profits
c. Amount written off in respect of Intangible Assets and New Issue Expenses (note)
d. Contribution to the Contingencies Reserve
e. Contribution towards arrears of Depreciation
f. Contribution to Development Reserve
g. Other Special Appropriations permitted by the State Government.
INSURANCE COMPANY ACCOUNT:
1. Computation of premium income, claims expense and commission expense in
the case of an insurance company?
Ans:
a. Premium: It is the consideration paid by insured to get his risk covered. The aggregate of
premium received together with premium receivable (outstanding) as reduced by premium
on reinsurance ceded, if any, is shown in the credit side of revenue account. Thus the net
figure of premium to be shown in revenue account is arrived in the following manner.
Premium Received (direct) Rs.
Add: Premium Received (Reinsurance accepted) Rs.
Add: Premium outstanding Rs. .
Add: Bonus in Reduction of premium Rs. .
Less: Premium of Reinsurance ceded Rs. .
Net Premium Rs. .
PCC/IPCC_Accounts_theory_______________________________________17
MEC/CEC,CA/CWA&B.Com By Mattupalli Associates for Master Minds
The reserve for unexpired risk and additional reserve are created on the net premium only.
b. Claims: The insurance company covers various risks for a consideration called Premium
and in case the risk covered falls with the insured, the insurance company is liable to
compensate. The amount paid as claim is written in the debit side of revenue account.
In addition to this, insurance companies, are required to incur expenditure as legal
expenses and medical expenses in connection with claim. This expenditure since directly
related to claim are basically incurred to reduce the liability of claim are also accounted for
under the head claims. Thus following shall be treatment of various adjustments under the
head Claim.
Claim:
Particulars Rs. Rs.
Claim paid during the year
Add: i Claim outstanding at, the closer of year,
ii Medical Expenses reclaims,
iii Legal expenses reclaims,
iv Survey expenses.
Less: i Claims outstanding at the beginning of the year
ii Re insurance claim recoveries.








c. Commission: When making final accounts of an Insurance Company is prepared terms
with respect to commission are:
a. Commission on direct business.
b. Commission on reinsurance ceded and
c. Commission on reinsurance accepted.
a. The commission paid by an insurance company for carrying out work by its agents is
called commission on Direct Business and shown in Direct side of Revenue Account.
b. Where reinsurance premium is payable then commission is recoverable in such
reinsurance business, the commission so paid by reinsurer is Commission on Re
insurance business ceded for the reinsured Company and is shown on Credit side of
revenues account.
c. There is Commission on Reinsurance Accepted for the insurance company accepting the
risk of other insurance company and is reflected in Debit side of revenue account. The
presentation is as follows:
Dr. Revenue Account Cr.
Debit Rs. Credit Rs.


Commission on direct business
Commission on reinsurance accepted
Commission on reinsurance
ceded
2. Write short notes on Reinsurance?
Ans:
a. When an Insurance Company wants to part with some risk on an insurance policy, a part of
the risk may be insured with some other insurer, is called, reinsurance ceded and original
company is called reinsured. In this case reinsured company surrender proportionate
premium to the other insurer and receive proportionate commission to reinsurance ceded.
b. In case of claim, original company pay to the policy holder and in turn receives
proportionate claim from the other company. For the insurer with whom risk has been
reinsured, this is called reinsurance accepted.
c. There are two types of reinsurances. (a) Fluctuative Reinsurance and (b) Treaty Re
insurance.
i. Fluctuative Reinsurance: In this type of reinsurance each risk is negotiated
separately. Reinsurance of each risk is affected by ceding company and accepted by the
PCC/IPCC_Accounts_theory_______________________________________18
Ph: 08632242355 www.gntmasterminds.com
accepting company. Both companies have free choice for offer and acceptance. Since
each transaction is negotiated separately, this involves large volume of work.
ii. Treaty Reinsurance: Under this type of reinsurance an agreement (Treaty) is entered
between insurance companies to offer reinsurance and accept reinsurance
compulsorily. The treaty may be for a specific type of risk or insurances of a specific
geographical area. It is mandatory for both, ceding company to offer and insurer to
accept within the limits of Treaty.
3. Write short notes on Reserve for Unexpired Risks in an insurance Company?
Ans: Insurance policies are usually issued for a period of 1 year. However, at the year end, risks
remain unexpired on most of the policies. Thus, total premium received cannot be taken as
income of the current year. Since, risk is not reduced with passage of time, the premium
relating to the next year is not calculated in proportion to unexpired period there is an
unexpired liability under various policies. In order to cover this unexpired risk a reserve is
created. This reserve is known as Unexpired Risk Reserve.
As per recommendation of the Executive Committee of the General insurance Council, the
reserve for unexpired risk has to be created at following rates:
a. For Marine Business 100% of Net Premium
b. For Other Business 40% of Net Premium
However, as per Income Tax Rules, a provision of 50% of net premium may be created for other
business.
The journal entry for creating provision at the end of year is:
Revenue A/c . Dr.
To Reserve for Unexpired Risk A/c.
Reserve for unexpired risk account is shown in Balance Sheet (liabilities side)
Next year, opening balance of Reserve for Unexpired Risk Account will be transferred to credit
side of revenue account by making following entry:
Reserve for Unexpired Risk A/c .Dr.
To Revenue A/c.
4. What are the accounting entries pertaining to reinsurance business ceded to and by
an insurance company? What are the corresponding commission entries?
Ans: When an insurance company wants to part with some risk on an insurance policy, a part
of the risk be insured with some other insurer. This is called Reinsurance ceded. The original
company is called Reinsured. In this case reinsured company surrenders proportionate
premium to the other insurer and receives proportionate commission to reinsurance ceded.
In case of claim, original company:
Pays to the policy holder and in turn recovers proportionate claim from the other company. For
the answer with whom risk has been reinsured, this is called reinsurance accepted.
A Insurance Company cedes reinsurance business to B Insurance Company. C Insurance
company further cedes reinsurance business to A Insurance company. Accounting entries
pertaining to reinsurance business ceded to and by A Insurance Company as follows:
1. Reinsurance Premium (or reinsurance ceded ) A/c Dr.
To B Insurance Company A/c
(Being premium on reinsurance business ceded to B insurance co. recorded
2. C Insurance Company A/c Dr.
To Reinsurance Premium (or reinsurance accepted) A/c
(Being premium on business ceded by C insurance company recorded)
3. B Insurance company A/c Dr.
To claims (On reinsurance ceded) A/c
(Being claim receivable from B company for part of insurance business ceded)
PCC/IPCC_Accounts_theory_______________________________________19
MEC/CEC,CA/CWA&B.Com By Mattupalli Associates for Master Minds
4. Claims (on reinsurance accepted) A/c Dr.
To C Insurance Company A/c
(Being claims on reinsurance business accepted from Z company recorded)
5. B Insurance Company A/c Dr.
To Commission (on reinsurance ceded) A/c
(Being commission due on reinsurance business ceded to B insurance company recorded)
6. Commission (on reinsurance accepted) A/c Dr.
To C Insurance Co. A/c
(Being commission due on reinsurance business ceded to C company debited)
5. Explain the significance of average clause in a Fire Insurance Policy?
Ans: In the interest of the business, business units take a fire insurance policy to indemnify
itself against the loss of stock and other assets resulting from the fire.
A fire insurance policy generally includes an average clause to discourage the under insurance
of stock or any other asset.
The impact of this clause is that:
a. If the value of stock or any other asset insured on the date of fire is more than the amount
of policy taken, the full value of stock or any other asset destroyed does not become payable
to the business unit.
b. But, insurance company agrees to pay the proportion of the loss which the amount of policy
taken bears to the total value of stock or any other assets on the date of the fire.
Following formula is used for computing average clause:
= Value of stock destroyed x Value of Insurance Policy
Value of stock on the date of fire
Thus, Insurance Company accept the insurance claim in a proportionate reduction to the
actual loss and stock or any asset on the date of such loss.
SHARE CAPITAL:
1. Describe the conditions, which have to be fulfilled by a Joint Stock Company to buy
back its equity shares?
Ans: According to Section 77A of the companies Act, 1956 a joint stock company has to fulfill
the following conditions to buyback its own equity shares:
a. There must be an authorization in the articles for the buyback.
b. A special resolution must be passed in general meeting of the company for authorizing the
buyback and it must be completed within 12 months of passing S.R.
c. The buyback should not exceed 25% of the total paidup capital and freereserves of the
company in particular financial year.
d. All the shares of the company must be fully paidup.
e. The ratio of the debt owned by the company is not more than twice the capital and its free
reserves after such buy back.
f. The buyback is made out of the freereserves or out of the proceeds of the fresh issue.
g. The buyback of the shares listed on any recognized stock exchange is in accordance with
the regulation made by the SEBI on this behalf.
2. What are Sweat Equity Shares? What are the conditions, which must be fulfilled by a
Joint Stock Company to issue these shares?
Ans: The Companies (Amendment) Act, 1999 has introduced with the help of Section 79A a new
type of equity shares called Sweat Equity Shares. Sweat equity shares are the equity shares
issued by a company to its employees or director at a discount or for consideration other than
PCC/IPCC_Accounts_theory_______________________________________20
Ph: 08632242355 www.gntmasterminds.com
cash for providing knowhow or making available right in the nature of intellectual property
right or value addition by whatever name called.
When the following conditions are fulfilled by the company, than only a company may issue
sweat equity shares:
a. Sweat equity shares will be issued only when it is authorized and a special resolution is
passed by the company in general meeting.
b. The resolution specifies the number of shares, current market price, the consideration if any
and the class or classes of directors or employees to whom such equity shares are to be issued.
c. Not less than one year has, at the time of the issue, elapsed since the date on which the
company was entitled to start the business.
d. The sweat equity shares of company, whose equity shares are listed on a stock exchange are
issued in reference with the regulation made by the Securities and Exchange Board of India
on this behalf.
e. In case of company whose equity shares are not listed on any stock exchange, the Sweat
equity shares are issued on reference with the guidelines as may be prescribed.
3. Write short notes on Dividend on partly paid shares?
Ans:
a. As per the provisions given in the articles of the company, in case of partly paidup shares,
the dividends are payable either on the nominal, called up or the paid up amount of shares.
b. In the absence of any such provision, Table A would be applied.
c. In this case the amount to be paid as dividend will be calculated on the amount paidup on
the shares and at the time of calculation, the date on which the amount were paid must be
taken into account.
d. Calls paid in advance do not rank for payment of dividend.
e. But the interest may be paid on such calls, the rate of interest is 6% p.a as per Table A
different rates may be prescribed by the articles of the company.
f. According to Section 93, of the companies Act, 1956 a company may if so authorized by its
article, pays a dividend in proportion to the amount paid on each share, where a larger
amount is paid on some shares than on others.
g. But where the articles are silent and Table A has been excluded, the amount of dividend
payable will have to be calculated on the nominal amount of shares.
h. According to Clause 88 to Table A dividends are to be declared and paid according to the
amounts paid or credited as paid on the shares in respect where of the dividend is paid.
i. But in case nothing is paid upon any of the shares of the company, dividends may be
declared and paid as per the nominal amount of the shares.
INTERNAL RECONSTRUCTION:
1. What is meant by Internal Reconstruction?
Ans: In case of internal reconstruction, the companys existing financial structure is
reorganized without dissolving the existing company and without forming a new company.
Taking a wider meaning of the term Internal Reconstruction. It includes:
a. Alteration of Share Capital under Section 94 to 97.
b. Reduction of Share Capital under Section 100 to 105.
c. Variation of Shareholders Rights under Section 106.
d. Scheme of Compromise/Arrangement under Sections 391 to 393 and 394A.
PCC/IPCC_Accounts_theory_______________________________________21
MEC/CEC,CA/CWA&B.Com By Mattupalli Associates for Master Minds
PREPARATION OF FINANCIAL STATEMENTS:
1. Which parties are Interested in Financial Statements?
Ans: The users of financial statements include present and potential investors, employees,
lenders, supplier and other trade creditors, customers, governments and their agencies and the
public. They use financial statements in order to satisfy some of their information needs. These
needs include the following:
a. Investors: The providers of risk capital are concerned with the risk inherent in, and return
provided by, their investments. They need information to determine whether they should
buy, hold or sell. They are also interested in information which enables them to assess the
ability of the enterprise to pay dividends.
b. Employees: Employees and their representative groups are interested in information about
the stability and profitability of their employers. They are also interested in information
which enables them to assess the ability of the enterprise to provide remuneration,
retirement benefits and employment opportunities.
c. Lenders: Lenders are interested in information which enables them to determine whether
their loans, and the interest attaching to them, will be paid when due.
d. Suppliers and other trade Creditors: Suppliers and other creditors are interested in
information which enables them to determine whether amounts owing to them will be paid
when due. Trade creditors are likely to be interested in an enterprise over a shorter period
than lenders unless they are dependent upon the continuance of the enterprise as a major
customer.
e. Customers: Customers have an interest in information about the continuance of an
enterprise, especially when they have a longterm involvement with, or are dependent on,
the enterprise.
f. Governments and their agencies: Governments and their agencies are interested in the
allocation of resources and therefore, the activities of enterprises. They also require
information in order to regulate the activities of enterprises and determine taxation policies,
and to serve as the basis for determination of national income and similar statistics.
g. Public: Enterprises affect members of the public in a variety of ways. For example,
enterprises may make a substantial contribution to the local economy in many ways
including the number of people they employ and their patronage of local suppliers. Financial
statements may assist the public by providing information about the trends and recent
developments in the prosperity of the enterprise and the range of its activities.
While all of the information needs to these users cannot be met by financial statements,
there are needs which are common to all users. As providers of risk capital to the enterprise,
investors need more comprehensive information than other users. The provision of financial
statements that meet their needs will also meet most of the needs of other users that
financial statements can satisfy.
2. Give the Four qualitative Characteristics which the financial statements should observe?
Ans:
a. Understandability: Financial Statements should be readily understandable by the users.
This means that all required information should be disclosed, clearly and properly.
b. Relevance: Financial Statements should contain only relevant information. Information,
which is likely to influence the economic decisions of the users, is said to be relevant.
Relevance of an item of information is related to its materiality.
c. Reliability: Financial Statements should be reliable, i.e. free from material error and bias.
This means that the transactions and events in Financial Statements are
i. Are faithfully represented.
ii. Reported in terms of its substance and economic reality, rather than merely their legal form.
iii. Reported in a neutral fashion, i.e. free from bias.
PCC/IPCC_Accounts_theory_______________________________________22
Ph: 08632242355 www.gntmasterminds.com
iv. Reported based on the principle of prudence.
d. Comparability: Financial Statements should be useful for interfirm comparison (i.e. between
different Firms in the same industry), and intrafirm comparison (i.e. within the Firm for
different periods, branches/divisions, products, etc.). Comparability can be achieved only if
the financial effect of any change in accounting policies is disclosed properly.
e. True and Fair View: Financial Statements are required to show an true and fair view of the
performance, financial position and cash flows of the enterprise.
3. One of the Characteristics of Financial Statements is neutrality Do you agree with
this statement?
Ans:
a. Yes. One of the essential characteristics of Financial Statement is Neutrality.
b. It means the Financial Statements are free from Biasimpartial.
c. In order to make the Financial Statements more ReliableNeutrality is more important. The
Financial Statements prepared and reported should be from Material misstatements and bias.
d. Hence Neutrality is one of the important characteristics of Financial Statements.
4. Write the Procedure for Calculation of Managerial Remuneration (or) How to calculate
Profits as per Sec. 349 for the sale of calculating Managerial Remuneration?
Ans: Calculation of Net Profits [Sec. 349]: Sec. 349 lays down the manner in which net
profits for the purposes of calculation of managerial remuneration shall be calculated.
The various items which are required to be included or excluded or deducted or not to be
deducted as per Sec. 349 are given below:
Items to be included in/excluded from Profits for Computing Managers Remuneration
Items to be included in profits Items to be excluded from profits
a. Bounties and subsidies received
from any Government or any
public authority constituted or
authorized in this behalf by any
government.
a. Premium on shares or debentures.
b. Profit on sale of forfeited shares.
c. Profits of capital nature including
those from the sale of the undertaking
of the company.
d. Profits of capital nature from the sale
of any immovable property or fixed
assets.
Items to be / Not to be deducted from Profit for Computing Managers Remuneration
Item to be deducted from Profits Item not to be deducted from Profits
a. All the usual working charges
b. Directors remuneration
c. Bonus of Commission paid or payable to
any person employed or engaged by the
company
d. Excess Profit Tax and Business Tax
e. Interest on Debentures, Mortgages,
Loans and Advances.
f. Expenses of Repairs not of Capital
Nature
g. Contribution to Charitable and other
funds, not directly related to the
business of the company not exceeding
Rs. 50,000 or 5% of the average net
profit during the three financial years
immediately preceding, whichever is
greater. However, these limits can be
exceeded with the consent of the
company in general meeting
a. Incometax and super tax paid by
the company or any other tax on the
income of the company not falling
under Clause (d) above.
b. Any Compensation, Damages or
Payments made voluntarily and not
paid due to any legal liability.
c. Capital Losses.
PCC/IPCC_Accounts_theory_______________________________________23
MEC/CEC,CA/CWA&B.Com By Mattupalli Associates for Master Minds
h. Depreciation to the extent specified in
Sec. 350
i. Past losses arising after 1
st
April, 1956 to
the extent not already deducted in any
subsequent year
j. Any compensation or damages to be paid
due to any legal liability including the
one arising from a breach of contract
k. Any sum paid by way of insurance to
cover any liability discussed in above
clause.
l. Bad debts written off or adjusted during
the accounting year.
5. Write a short notes on Transfer of Profit to Reserve Rules?
Ans:
a. Yes, a Company can transfer more than 10% of its profits to reserves. In such an event a
minimum dividend has to be declared as stated below:
b. Minimum dividend rate should be equal to average rate of dividend declared in preceding
three years.
c. Where bonus shares have been issued in the preceding three years, the distribution of
dividend, a minimum distribution of dividend equal to the average amount (quantum and
not the rate) of the dividend declared for the three years is ensured.
d. However, the minimum distribution in the above two cases need not be ensured if net profits
after tax have fallen by at least 20% of the net profits of the average net profits after tax of
the two preceding financial years.
e. Where no dividend is declared the amount proposed to be transferred to its reserves from
the current profits must be less than the average amount of dividends declared to the
shareholders over the three preceding financial years.
ACCOUNTS IN COMPUTERISED ENVIRONMENT:
1. What are the advantages and disadvantages of outsourcing the accounting functions?
Ans:
a. Advantages are:
i. Organisation can concentrate on their core area Less number of human resource is
required.
ii. Organisation is able to utilized the expertise knowledge and experience of the
outsourcing agency.
iii. It is economical for the organisation.
iv. Labour turnover does not effects the functioning of accounts department.
v. Accounts are maintain and stored in the hand of professionals.
b. Disadvantages are:
i. Outsourcing agency is unable to meet the standard desirable.
ii. Various hidden costs are involved, which was not initially envisaged.
iii. There is a fear of confidentiality loss and security of accounts.
iv. Delay in obtaining services from third party.
v. Organisation Looses their control from various financial activities.
PCC/IPCC_Accounts_theory_______________________________________24
Ph: 08632242355 www.gntmasterminds.com
2. Explain the factors to be considered before selecting the prepackaged accounting
software?
Ans: It is very difficult task for an organisation is choose appropriate accounting software from
the bundle of software available in the market. Some basic criteria should be considered while
selecting the software.
a. Fulfillment of business needs: Buyers try to match his own requirement with the available
softwares.
b. Easy to use: Such software which is easily operative should be selected.
c. Provides maximum reports: Some software packages are available in the market which
might provide extra reports or such report as they wants.
d. Goodwill of the Vendor: A stable vendor with good past records will always be preferred
because their continuous support is essential for any software.
e. Cost comparison: First analyse various software the select most economic software.
f. Regular update: Vendor normally provides regular updates to take care of the changes of
law as well as add new feature to the existing software. So, select the vendor whose past
record in this context is good.
3. What are the advantages and disadvantages of customized accounting packages?
Ans:
a. Advantages are:
i. Such softwares suitable match with the organizational structures.
ii. It covers all functional areas of the organizations.
iii. The input screens can be tailor made so that is matches with the input documents for
data entry.
iv. Reports are available as per the specification of the organisation.
v. Various tools such as Barcode can used as input devices.
b. Disadvantages are:
i. Development of such software is a time taking process.
ii. Of input specifications are incomplete or improper resulting in a defecting or
inappropriate system.
iii. Lack of documentation.
iv. Inadequate control measures.
v. Regular undates are not possible.
vi. It is costlier than prepackaged accounting.
vii.Less reliability.
viii. Gestation period is very high.
4. What are the advantages and disadvantages of an ERP package ?
Ans:
a. Advantages are:
i. Large volume of information are available through such package.
ii. It is an integrated package so it reduces the possibility of duplication of data entry.
iii. It is a generalised package which covers most of the common functionalities of any
specific module.
iv. Reports or ERP are standardized across industry and are generally acceptable to the
user.
PCC/IPCC_Accounts_theory_______________________________________25
MEC/CEC,CA/CWA&B.Com By Mattupalli Associates for Master Minds
v. Various reports are available in such package. These reports are standardized and
generally acceptable to the user.
b. Disadvantages are:
i. It is less flexible: In which user may have to modify their business procedure at times
to be able to effectively use the ERP.
ii. Implementation is very difficult: Many of the consultants doing he implementation of
the ERP may not be able to fully appreciate the business procedure to be able to do a
good implementation of an ERP.
iii. It is very expensive: ERP are normally priced at an amount which is often beyond the
reach of small and medium sized organisation. However, there are some ERP coming into
the market which are moderately priced and may be useful to the small businesses.
iv. It is a very complex software: Generally an ERP package has large number of options
to choose from. Further the parameter settings and configuration market it a little
complex for the common users.
The End

S-ar putea să vă placă și